Project Engineering Primer for Chemical Engineers 9811906599, 9789811906596

This book discusses financial, managerial and engineering aspects associated with project engineering. The book is a tex

128 43 4MB

English Pages 252 [246] Year 2022

Report DMCA / Copyright

DOWNLOAD PDF FILE

Table of contents :
Preface
Acknowledgements
Contents
About the Author
List of Figures
List of Tables
1 Introduction
1.1 Preamble
1.2 Engineering, Science and Chemical Engineering
1.3 Project and Project Engineering
1.3.1 Characteristics of a Project
1.4 Different Parties Involved in a Chemical Engineering Project
1.5 Categories and Phases of a Chemical Engineering Project
1.6 Life Cycle of a Chemical Engineering Project
1.6.1 Pre-project Work
1.6.2 Starting the Project
1.6.3 Starting and Carrying Out Engineering Work
1.7 Project Engineering and Process Engineering
1.8 Duties and Responsibilities of Project Engineer—Various Perspectives
1.9 Engineering and Environmental Ethics in Project Engineering
1.10 Risk and Reliability Analysis
References
2 Pre-project Work
2.1 Preamble
2.2 Feasibility Study—A Brief Overview
2.3 Preparation of Bid Documents—A Guideline
2.4 Codes and Standards
2.5 Site Selection
2.5.1 Availability of Raw Materials
2.5.2 Location Concerning the Market
2.5.3 Transportation Facilities
2.5.4 Availability of Workforce and Labours
2.5.5 Availability of Utilities—Water, Fuel and Power
2.5.6 Climatic Conditions
2.5.7 Environmental Impacts Including Waste Disposal Facilities
2.5.8 Site Characteristics Concerning Potential Hazards
2.5.9 Taxation and Legal Aspects
2.5.10 The Political Situation and Local Community Consideration
2.6 Environmental Impact Assessment (EIA)
2.7 Optimization in Design
2.7.1 Trade-Offs
2.7.2 Objective Function and Constraints
2.7.3 Degree of Freedom
2.7.4 Mathematical Programming Techniques for Optimization
2.7.5 Optimization of a Single-Variable Problem-Problem Statement
2.7.6 Application of Optimization in Process Design
2.8 Scaling Up and Scaling Down
2.8.1 Scaling Factors
2.9 Safety and Hazard Issues
2.9.1 Toxicity
2.9.2 Flammability, Fire and Explosion
2.9.3 Ionizing Radiation
2.9.4 High Pressure and Vacuum
2.9.5 Temperature Deviation
2.9.6 Noise
References
3 Basic Engineering
3.1 Preamble
3.2 Process Drawings
3.2.1 Block Flow Diagram (BFD)
3.2.2 Process Flow Diagram (PFD)
3.2.3 Piping and Instrumentation Diagram (P&ID)
3.2.4 Plot Plans or Layouts
3.3 Design and Selection of Equipment
3.4 Safety, Hazard and Environmental Considerations in Design
3.5 HAZOP Analysis
References
4 Detailed Engineering
4.1 Preamble
4.2 Drawing and Documents for a Project
4.3 Specification or Data Sheets for Equipment
4.3.1 Specification for an Atmospheric Storage Tank
4.3.2 Specification for a Shell and Tube Heat Exchanger
4.3.3 Specification for a Centrifugal Pump
4.4 Worked Out Examples on Pump Heads
4.4.1 Pressure Drop Calculation Sheet for Pumps
4.5 Specification of the Instruments and Control Valves
4.6 Line and Valve List or Schedule
4.7 Equipment List
4.8 Engineering Change Notice
4.9 Evaluation of Quotations from Vendors—Procurement Assistance
4.10 Plant Commissioning and Start-up
4.11 Operation and Maintenance Manual
References
5 Financial Aspects of Project Engineering
5.1 Preamble
5.2 Cash Flow
5.2.1 Capital Investment
5.2.2 ISBL Plant Cost
5.2.3 Offsite Cost or OSBL Investment
5.2.4 Depreciation Charges and Other Investment Incentives
5.3 Estimation of Capital Investment
5.3.1 Cost Indexes
5.3.2 Cost Components of Capital Investment
5.3.3 Estimation of Purchased Equipment Cost
5.3.4 Methods of Estimating Capital Investment
5.4 Methods for Calculation of Depreciation Charges
5.5 Economic Analysis and Profitability Study
5.5.1 Methods for Calculating Profitability
5.6 Alternative Investments
5.7 Replacement
5.8 Worked Out Examples
References
6 Management Aspects of Project Engineering
6.1 Preamble
6.2 Project Planning
6.3 Project Scheduling
6.4 Project Scheduling and Progress Control
6.4.1 Bar Chart
6.4.2 Gantt Chart
6.4.3 Network Analysis
6.5 Project Crashing and Cost-Time Trade-Off
6.6 Risk Management
References
Appendix A Case Study—Developing PFD and P&ID
A.1 Preamble
A.2 Example-I
A.2.1 Preliminary Process Flow Diagram (PFD)
A.2.2 Preliminary Piping and Instrumentation Diagram (P&ID)
A.3 Example–II
A.3.1 Preliminary Process Flow Diagram (PFD)
A.3.2 Preliminary Piping and Instrumentation Diagram (P&ID)
Annexure I Multiple Choice Questions with Answers
Annexure II Project Engineering and Similar Courses—Syllabi (Collected from Open Sourced Websites of the Respective Institutions)
Index
Recommend Papers

Project Engineering Primer for Chemical Engineers
 9811906599, 9789811906596

  • 0 0 0
  • Like this paper and download? You can publish your own PDF file online for free in a few minutes! Sign Up
File loading please wait...
Citation preview

Sampa Chakrabarti

Project Engineering Primer for Chemical Engineers

Project Engineering Primer for Chemical Engineers

Sampa Chakrabarti

Project Engineering Primer for Chemical Engineers

Sampa Chakrabarti Department of Chemical Engineering University of Calcutta Kolkata, India

ISBN 978-981-19-0659-6 ISBN 978-981-19-0660-2 (eBook) https://doi.org/10.1007/978-981-19-0660-2 © The Editor(s) (if applicable) and The Author(s), under exclusive license to Springer Nature Singapore Pte Ltd. 2022 This work is subject to copyright. All rights are solely and exclusively licensed by the Publisher, whether the whole or part of the material is concerned, specifically the rights of translation, reprinting, reuse of illustrations, recitation, broadcasting, reproduction on microfilms or in any other physical way, and transmission or information storage and retrieval, electronic adaptation, computer software, or by similar or dissimilar methodology now known or hereafter developed. The use of general descriptive names, registered names, trademarks, service marks, etc. in this publication does not imply, even in the absence of a specific statement, that such names are exempt from the relevant protective laws and regulations and therefore free for general use. The publisher, the authors and the editors are safe to assume that the advice and information in this book are believed to be true and accurate at the date of publication. Neither the publisher nor the authors or the editors give a warranty, expressed or implied, with respect to the material contained herein or for any errors or omissions that may have been made. The publisher remains neutral with regard to jurisdictional claims in published maps and institutional affiliations. This Springer imprint is published by the registered company Springer Nature Singapore Pte Ltd. The registered company address is: 152 Beach Road, #21-01/04 Gateway East, Singapore 189721, Singapore

For the budding Chemical Engineers

Preface

After spending 4 years working out various complicated mathematical problems on heat, mass and momentum transfer, when a young chemical engineer lands into a project engineering job, s/he feels lost in the various documents of a project. In most cases, s/he fails to correlate her or his bookish training with the practical field of construction of a process plant. S/he has to do many things that were beyond the scope of ‘chemical engineering’ as it was known to her or him so far. Project Engineering is the bridge between the theoretical basis of the process and the practical construction or implementation of the theoretical knowledge. Project Engineering teaches how to obtain a tangible outcome from a design calculation and its underlying theory. It shows how a ‘process’ becomes a ‘plant’.

Fig 1. Interdisciplinary nature of project engineering

vii

viii

Preface

Project Engineering stands on the confluence of three disciplines—process design, financial and management skills. Of these three, process design is the area of chemical engineers, whereas the other two are from the other two disciplines. Though costing and finance, as well as management, are two full-fledged disciplines by their own respective capacities, construction and operation of a plant are not possible without some knowledge of these two areas. In most bachelor’s degree programmes of Chemical Engineering in India, a course is dedicated to Project Engineering. There are a few seminal textbooks on plant design and economics. However most of the textbooks emphasize the design aspects; the next priority is given to the cost estimation and financial aspects, but the management aspects are grossly neglected though planning, scheduling and monitoring are inseparable parts of the construction of a chemical process plant. Again, there are a few books exclusively on project management, where only the management aspects are described without elucidating the process design or economics for obvious reasons. In the books on design and economics, even in the design part, process drawings are generally not illustrated in detail so that a student can become interested to construct a small process flow diagram (PFD), piping and instrumentation diagram (P&ID) or an equipment layout by her/himself. It is seldom mentioned that specification sheets of purchased equipment are to be prepared using P&ID, PFD along with the design of the particular equipment. The duties and responsibilities of a project engineer are generally described without pointing out the difference in the roles of project engineers belonging to different parties of a project. The book aims to bridge the above-mentioned gaps in the available bibliography of project engineering, especially for budding chemical engineers. The volume of the book has been kept minimum so that the youngsters are not scared by the size of the book and also to keep the book handy. Generally, for design purposes, students refer to the standard books on the respective topics rather than a project engineering book. So process calculations for various equipment are not included. In Chap. 1, a background of project engineering is described with special emphasis on its interdisciplinary nature. Different phases and various parties involved in a project are discussed. It is mentioned that the duties and responsibilities of a project engineer or project manager vary according to where s/he belongs to. Engineering and environmental ethics are nowadays emphasized much. This has been discussed with the ethical principles of a few professional bodies of Chemical Engineers. In Chap. 2, pre-project works are introduced. A few considerations and tasks are there before one thinks of launching a project. Some of them are feasibility study, environmental impact assessment, preparation of enquiry documents for basic and detailed engineering, deciding the production capacity with scaling up or down and being aware of the safety issues. This chapter informs the student about these groundworks.

Preface

ix

Chapter 3 is dedicated to some aspects of basic engineering. The essences of basic engineering are material and energy balance for the whole plant as well as for the individual equipment. Based on these balances, a process flow diagram (PFD) is developed. Based on the PFD, detailed engineering is done. Process Design for the equipment and preliminary sizing of the equipment are also involved. There are many authentic books for material and energy balance; many more books are there for the design of process equipment like heat exchangers or distillation columns. So such process designs have not been included here. Instead, the outcome of the above as basic engineering like the process drawings, HAZOP study report and safety considerations of design have been highlighted with examples wherever possible. Thumb rules for the selection and design of equipment may also be helpful, though different organizations may have their own thumb rules. Chapter 4 is on how the information generated in the basic engineering phase is further developed and detailed for the construction of a real-life plant. Preliminary Piping and Instrumentation Diagrams (P&ID), constructed in the basic engineering phase, are updated by incorporating information received from the actual manufacturers, specifications are modified based on the equipment layout and so on. Fabrication and site activities start with information generated in the detailed engineering phase. These sets of information are compiled in the form of several universally understandable documents. An Appendix is added at the end where two simple worked out examples of developing PFD and P&ID have been compiled with the steps and explanations. The financial aspects associated with project engineering are described in Chap. 5. Financial Management is a separate discipline, but some of the costing and finance should be understood by a project engineer. Project cash flow and its components, choosing among alternative options for investing for a piece of equipment or replacing existing equipment are some of those aspects. In the course of knowing such features, a few accounting calculations like interest or depreciation calculation may be necessary. These are explained with simple worked out examples. Similarly, though Project Management is a branch of management discipline, a few management skills are required to construct, monitor and implement a chemical process plant. Some of such techniques are the critical path method (CPM), programme evaluation and review technique (PERT), Gantt chart and bar chart. Risk management is also a critical area for a project. These management techniques are discussed with examples in Chap. 6. In Annexure-I, 50 multiple-choice questions (MCQ) are compiled (with the correct answer) from all the chapters for quiz purposes. In Annexure-II, syllabi for the relevant courses in different institutes in India have been compiled for ready reference and comparison with the coverage of this book.

x

Preface

The book is dedicated to my dear students—not only of the University of Calcutta but also of all other institutes. This book is meant only for a preliminary introduction to Project Engineering for the undergraduate students whereby the readers are expected to be encouraged to read further. Nowadays, various software like ASPEN and MS PROJECT are indeed available for detailed engineering and project management, but software cannot be a substitute for engineering judgement; it can only supplement it and the use of software without understanding the basis is a dangerous abandonment of professional responsibility. This book is expected to explain the rationale for using such tools and software. If after reading this book, the students feel interested to work as Project Engineers, I shall consider my effort to be successful. Kolkata, India

Sampa Chakrabarti

Acknowledgements

During my service in different consultancy organizations, I came across a good number of very efficient supervisors working under whom was great experience in terms of learning with pleasure. Mr. S. K. Mukherjee was my first boss, who is also an alumnus of my alma mater. He taught me a lot. I gratefully acknowledge the affectionate guidance of Late Mr. A. K. Dutta Majumdar and Late Mr. A. Ganguli during my service with M/s. Krebs & Cie (India) Ltd. While I was working with M/s. Development Consultants (P) Ltd., I was guided and advised by Late Mr. Partha Pratim Bhattacharya and Mr. Goutam Roy who induced me with detailed engineering of a petrochemical plant. Late Mr. Jayanta Sarkar, who was an Instrumentation Engineer, taught me many aspects of instrumentation and control that are required for P&ID. Dr. A. R. Ghosal, Director (Technical), M/s Development Consultants (P) Ltd., is still associated with my department and always showers his blessings on me. I am indebted to all of them. My husband, Mr. Dipankar Chakrabarti, being a corporate executive, explained many of the management and financial techniques. I am more than thankful to him as well as to my daughter Dr. Sampurna Chakrabarti. I am grateful to my Ph.D. supervisor and mentor, Prof. Binay Kanti Dutta, for always encouraging my academic endeavours. Professor Sekhar Bhattacharjee, who was my senior colleague in the Department of Chemical Engineering, always inspired and encouraged me. He runs a tutorial website for the students of Chemical Engineering, and I took help from the same. His comments enhanced the quality of the book. No amount of gratitude is enough for him. Dr. Prantik Banerjee, my former Ph.D. student and currently Assistant Professor, Adamas University, downloaded many materials for me. Dr. Anirban Roy, my other student, Assistant Professor, Maulana Abul Kalam Azad University of Technology, has kindly rendered help in editing. I am thankful to them. I am fortunate to have my alma mater as my workplace; I am grateful to the Department of Chemical Engineering, University of Calcutta, and to my colleagues there.

xi

xii

Acknowledgements

I am thankful to the publication team of Springer Nature, Singapore, for a special mention of Mr. Aninda Bose, Senior Editor, and Mr. Suresh Dharmalingam, Editor, for publishing this book. I acknowledge the support of my family and friends without which I could do nothing. I know, despite utmost care, there are mistakes in the book. I await bits of advice from erudite peers to rectify them. Sampa Chakrabarti

Contents

1 Introduction . . . . . . . . . . . . . . . . . . . . . . . . . . . . . . . . . . . . . . . . . . . . . . . . . . . 1.1 Preamble . . . . . . . . . . . . . . . . . . . . . . . . . . . . . . . . . . . . . . . . . . . . . . . . . 1.2 Engineering, Science and Chemical Engineering . . . . . . . . . . . . . . . 1.3 Project and Project Engineering . . . . . . . . . . . . . . . . . . . . . . . . . . . . . 1.3.1 Characteristics of a Project . . . . . . . . . . . . . . . . . . . . . . . . . . 1.4 Different Parties Involved in a Chemical Engineering Project . . . . 1.5 Categories and Phases of a Chemical Engineering Project . . . . . . . 1.6 Life Cycle of a Chemical Engineering Project . . . . . . . . . . . . . . . . . 1.6.1 Pre-project Work . . . . . . . . . . . . . . . . . . . . . . . . . . . . . . . . . . 1.6.2 Starting the Project . . . . . . . . . . . . . . . . . . . . . . . . . . . . . . . . . 1.6.3 Starting and Carrying Out Engineering Work . . . . . . . . . . 1.7 Project Engineering and Process Engineering . . . . . . . . . . . . . . . . . . 1.8 Duties and Responsibilities of Project Engineer—Various Perspectives . . . . . . . . . . . . . . . . . . . . . . . . . . . . . . . . . . . . . . . . . . . . . . 1.9 Engineering and Environmental Ethics in Project Engineering . . . . . . . . . . . . . . . . . . . . . . . . . . . . . . . . . . . . . . . . . . . . . . 1.10 Risk and Reliability Analysis . . . . . . . . . . . . . . . . . . . . . . . . . . . . . . . References . . . . . . . . . . . . . . . . . . . . . . . . . . . . . . . . . . . . . . . . . . . . . . . . . . . . .

1 1 2 3 3 5 6 7 7 7 7 9

11 14 15

2 Pre-project Work . . . . . . . . . . . . . . . . . . . . . . . . . . . . . . . . . . . . . . . . . . . . . . 2.1 Preamble . . . . . . . . . . . . . . . . . . . . . . . . . . . . . . . . . . . . . . . . . . . . . . . . . 2.2 Feasibility Study—A Brief Overview . . . . . . . . . . . . . . . . . . . . . . . . 2.3 Preparation of Bid Documents—A Guideline . . . . . . . . . . . . . . . . . . 2.4 Codes and Standards . . . . . . . . . . . . . . . . . . . . . . . . . . . . . . . . . . . . . . . 2.5 Site Selection . . . . . . . . . . . . . . . . . . . . . . . . . . . . . . . . . . . . . . . . . . . . . 2.5.1 Availability of Raw Materials . . . . . . . . . . . . . . . . . . . . . . . . 2.5.2 Location Concerning the Market . . . . . . . . . . . . . . . . . . . . . 2.5.3 Transportation Facilities . . . . . . . . . . . . . . . . . . . . . . . . . . . . 2.5.4 Availability of Workforce and Labours . . . . . . . . . . . . . . . . 2.5.5 Availability of Utilities—Water, Fuel and Power . . . . . . . . 2.5.6 Climatic Conditions . . . . . . . . . . . . . . . . . . . . . . . . . . . . . . . .

17 17 18 21 24 25 25 25 26 26 26 27

10

xiii

xiv

Contents

2.5.7

Environmental Impacts Including Waste Disposal Facilities . . . . . . . . . . . . . . . . . . . . . . . . . . . . . . . . . . . . . . . . . 2.5.8 Site Characteristics Concerning Potential Hazards . . . . . . 2.5.9 Taxation and Legal Aspects . . . . . . . . . . . . . . . . . . . . . . . . . 2.5.10 The Political Situation and Local Community Consideration . . . . . . . . . . . . . . . . . . . . . . . . . . . . . . . . . . . . . 2.6 Environmental Impact Assessment (EIA) . . . . . . . . . . . . . . . . . . . . . 2.7 Optimization in Design . . . . . . . . . . . . . . . . . . . . . . . . . . . . . . . . . . . . . 2.7.1 Trade-Offs . . . . . . . . . . . . . . . . . . . . . . . . . . . . . . . . . . . . . . . . 2.7.2 Objective Function and Constraints . . . . . . . . . . . . . . . . . . . 2.7.3 Degree of Freedom . . . . . . . . . . . . . . . . . . . . . . . . . . . . . . . . . 2.7.4 Mathematical Programming Techniques for Optimization . . . . . . . . . . . . . . . . . . . . . . . . . . . . . . . . . . . 2.7.5 Optimization of a Single-Variable Problem-Problem Statement . . . . . . . . . . . . . . . . . . . . . . . . . . . . . . . . . . . . . . . . . 2.7.6 Application of Optimization in Process Design . . . . . . . . . 2.8 Scaling Up and Scaling Down . . . . . . . . . . . . . . . . . . . . . . . . . . . . . . . 2.8.1 Scaling Factors . . . . . . . . . . . . . . . . . . . . . . . . . . . . . . . . . . . . 2.9 Safety and Hazard Issues . . . . . . . . . . . . . . . . . . . . . . . . . . . . . . . . . . . 2.9.1 Toxicity . . . . . . . . . . . . . . . . . . . . . . . . . . . . . . . . . . . . . . . . . . 2.9.2 Flammability, Fire and Explosion . . . . . . . . . . . . . . . . . . . . 2.9.3 Ionizing Radiation . . . . . . . . . . . . . . . . . . . . . . . . . . . . . . . . . 2.9.4 High Pressure and Vacuum . . . . . . . . . . . . . . . . . . . . . . . . . . 2.9.5 Temperature Deviation . . . . . . . . . . . . . . . . . . . . . . . . . . . . . 2.9.6 Noise . . . . . . . . . . . . . . . . . . . . . . . . . . . . . . . . . . . . . . . . . . . . References . . . . . . . . . . . . . . . . . . . . . . . . . . . . . . . . . . . . . . . . . . . . . . . . . . . . .

27 27 27 27 28 30 31 31 32 32 33 33 40 43 45 46 47 48 48 49 49 57

3 Basic Engineering . . . . . . . . . . . . . . . . . . . . . . . . . . . . . . . . . . . . . . . . . . . . . . 59 3.1 Preamble . . . . . . . . . . . . . . . . . . . . . . . . . . . . . . . . . . . . . . . . . . . . . . . . . 59 3.2 Process Drawings . . . . . . . . . . . . . . . . . . . . . . . . . . . . . . . . . . . . . . . . . 60 3.2.1 Block Flow Diagram (BFD) . . . . . . . . . . . . . . . . . . . . . . . . . 60 3.2.2 Process Flow Diagram (PFD) . . . . . . . . . . . . . . . . . . . . . . . . 61 3.2.3 Piping and Instrumentation Diagram (P&ID) . . . . . . . . . . . 65 3.2.4 Plot Plans or Layouts . . . . . . . . . . . . . . . . . . . . . . . . . . . . . . . 88 3.3 Design and Selection of Equipment . . . . . . . . . . . . . . . . . . . . . . . . . . 96 3.4 Safety, Hazard and Environmental Considerations in Design . . . . . 101 3.5 HAZOP Analysis . . . . . . . . . . . . . . . . . . . . . . . . . . . . . . . . . . . . . . . . . . 101 References . . . . . . . . . . . . . . . . . . . . . . . . . . . . . . . . . . . . . . . . . . . . . . . . . . . . . 106 4 Detailed Engineering . . . . . . . . . . . . . . . . . . . . . . . . . . . . . . . . . . . . . . . . . . . 4.1 Preamble . . . . . . . . . . . . . . . . . . . . . . . . . . . . . . . . . . . . . . . . . . . . . . . . . 4.2 Drawing and Documents for a Project . . . . . . . . . . . . . . . . . . . . . . . . 4.3 Specification or Data Sheets for Equipment . . . . . . . . . . . . . . . . . . . 4.3.1 Specification for an Atmospheric Storage Tank . . . . . . . . .

109 109 110 111 111

Contents

4.3.2 Specification for a Shell and Tube Heat Exchanger . . . . . . 4.3.3 Specification for a Centrifugal Pump . . . . . . . . . . . . . . . . . . 4.4 Worked Out Examples on Pump Heads . . . . . . . . . . . . . . . . . . . . . . . 4.4.1 Pressure Drop Calculation Sheet for Pumps . . . . . . . . . . . . 4.5 Specification of the Instruments and Control Valves . . . . . . . . . . . . 4.6 Line and Valve List or Schedule . . . . . . . . . . . . . . . . . . . . . . . . . . . . . 4.7 Equipment List . . . . . . . . . . . . . . . . . . . . . . . . . . . . . . . . . . . . . . . . . . . 4.8 Engineering Change Notice . . . . . . . . . . . . . . . . . . . . . . . . . . . . . . . . . 4.9 Evaluation of Quotations from Vendors—Procurement Assistance . . . . . . . . . . . . . . . . . . . . . . . . . . . . . . . . . . . . . . . . . . . . . . . . 4.10 Plant Commissioning and Start-up . . . . . . . . . . . . . . . . . . . . . . . . . . . 4.11 Operation and Maintenance Manual . . . . . . . . . . . . . . . . . . . . . . . . . . References . . . . . . . . . . . . . . . . . . . . . . . . . . . . . . . . . . . . . . . . . . . . . . . . . . . . .

xv

113 113 114 121 121 124 126 126 127 128 132 134

5 Financial Aspects of Project Engineering . . . . . . . . . . . . . . . . . . . . . . . . . 5.1 Preamble . . . . . . . . . . . . . . . . . . . . . . . . . . . . . . . . . . . . . . . . . . . . . . . . . 5.2 Cash Flow . . . . . . . . . . . . . . . . . . . . . . . . . . . . . . . . . . . . . . . . . . . . . . . . 5.2.1 Capital Investment . . . . . . . . . . . . . . . . . . . . . . . . . . . . . . . . . 5.2.2 ISBL Plant Cost . . . . . . . . . . . . . . . . . . . . . . . . . . . . . . . . . . . 5.2.3 Offsite Cost or OSBL Investment . . . . . . . . . . . . . . . . . . . . 5.2.4 Depreciation Charges and Other Investment Incentives . . . . . . . . . . . . . . . . . . . . . . . . . . . . . . . . . . . . . . . . . 5.3 Estimation of Capital Investment . . . . . . . . . . . . . . . . . . . . . . . . . . . . 5.3.1 Cost Indexes . . . . . . . . . . . . . . . . . . . . . . . . . . . . . . . . . . . . . . 5.3.2 Cost Components of Capital Investment . . . . . . . . . . . . . . . 5.3.3 Estimation of Purchased Equipment Cost . . . . . . . . . . . . . . 5.3.4 Methods of Estimating Capital Investment . . . . . . . . . . . . . 5.4 Methods for Calculation of Depreciation Charges . . . . . . . . . . . . . . 5.5 Economic Analysis and Profitability Study . . . . . . . . . . . . . . . . . . . . 5.5.1 Methods for Calculating Profitability . . . . . . . . . . . . . . . . . 5.6 Alternative Investments . . . . . . . . . . . . . . . . . . . . . . . . . . . . . . . . . . . . 5.7 Replacement . . . . . . . . . . . . . . . . . . . . . . . . . . . . . . . . . . . . . . . . . . . . . . 5.8 Worked Out Examples . . . . . . . . . . . . . . . . . . . . . . . . . . . . . . . . . . . . . References . . . . . . . . . . . . . . . . . . . . . . . . . . . . . . . . . . . . . . . . . . . . . . . . . . . . .

135 135 135 137 138 138 141 142 142 143 144 145 149 151 154 162 162 163 178

6 Management Aspects of Project Engineering . . . . . . . . . . . . . . . . . . . . . . 6.1 Preamble . . . . . . . . . . . . . . . . . . . . . . . . . . . . . . . . . . . . . . . . . . . . . . . . . 6.2 Project Planning . . . . . . . . . . . . . . . . . . . . . . . . . . . . . . . . . . . . . . . . . . . 6.3 Project Scheduling . . . . . . . . . . . . . . . . . . . . . . . . . . . . . . . . . . . . . . . . 6.4 Project Scheduling and Progress Control . . . . . . . . . . . . . . . . . . . . . . 6.4.1 Bar Chart . . . . . . . . . . . . . . . . . . . . . . . . . . . . . . . . . . . . . . . . . 6.4.2 Gantt Chart . . . . . . . . . . . . . . . . . . . . . . . . . . . . . . . . . . . . . . . 6.4.3 Network Analysis . . . . . . . . . . . . . . . . . . . . . . . . . . . . . . . . . . 6.5 Project Crashing and Cost-Time Trade-Off . . . . . . . . . . . . . . . . . . . . 6.6 Risk Management . . . . . . . . . . . . . . . . . . . . . . . . . . . . . . . . . . . . . . . . . References . . . . . . . . . . . . . . . . . . . . . . . . . . . . . . . . . . . . . . . . . . . . . . . . . . . . .

179 179 180 181 183 184 186 187 196 199 204

xvi

Contents

Appendix A: Case Study—Developing PFD and P&ID . . . . . . . . . . . . . . . . 205 Annexure I: Multiple Choice Questions with Answers . . . . . . . . . . . . . . . . . 213 Annexure II: Project Engineering and Similar Courses—Syllabi (Collected from Open Sourced Websites of the Respective Institutions) . . . . . . . . . . . . . . . . . . . . . . . . . . 223 Index . . . . . . . . . . . . . . . . . . . . . . . . . . . . . . . . . . . . . . . . . . . . . . . . . . . . . . . . . . . . . 229

About the Author

Dr. Sampa Chakrabarti is a Professor of Chemical Engineering at the University of Calcutta, India, a Fellow of the Institution of Engineers India (IEI) and a life member of the Indian Institute of Chemical Engineers (IIChE). She teaches Separation Processes, Project Engineering and Environmental Science and Engineering. Her research interest includes pollution control by advanced oxidation, nanoparticles and smart surfaces. She worked as visiting researchers in the State University of New York at Buffalo and Washington University in St. Louis, USA. She holds two Indian patents on ceramic pigment and on solar reactor. She published two books—one reference book on solar photocatalysis (jointly published by TERI, India and CRC press) and another text book for post graduate level on solid waste (with TERI, India). She had published more than 85 papers and chapters in reputed international journals, edited books and conferences. In addition, she has published books and popular science articles in Bengali for spreading scientific awareness among common people. She has 10 years of professional experience in Project Engineering with reputed engineering organizations of India like Development Consultants Pvt. Ltd., Kolkata and Batliboi and Company, Mumbai in addition to her teaching and research experience in the University of Calcutta.

xvii

List of Figures

Fig. 2.1 Fig. 2.2 Fig. 2.3 Fig. 3.1 Fig. 3.2 Fig. 3.3 Fig. 3.4 Fig. 3.5 Fig. 3.6 Fig. 3.7 Fig. 3.8 Fig. 3.9 Fig. 3.10 Fig. 3.11 Fig. 3.12 Fig. 3.13 Fig. 3.14 Fig. 3.15 Fig. 3.16 Fig. 3.17 Fig. 3.18 Fig. 3.19 Fig. 3.20 Fig. 3.21 Fig. 3.22 Fig. 3.23 Fig. 4.1 Fig. 4.2

Economic pipe diameter . . . . . . . . . . . . . . . . . . . . . . . . . . . . . . . . . Plot for optimum outlet temperature of cooling water . . . . . . . . . Levels of process similarity . . . . . . . . . . . . . . . . . . . . . . . . . . . . . . Block flow diagram for a complex plant . . . . . . . . . . . . . . . . . . . . Block flow diagram for a simple process . . . . . . . . . . . . . . . . . . . . Sample PFD . . . . . . . . . . . . . . . . . . . . . . . . . . . . . . . . . . . . . . . . . . . Sample part of process P&ID . . . . . . . . . . . . . . . . . . . . . . . . . . . . . Sample part of tank area P&ID . . . . . . . . . . . . . . . . . . . . . . . . . . . Sample part of utility (instrument air) plant P&ID (referred to for special control P&ID, Fig. 3.10) . . . . . . . . . . . . . . . . . . . . . Sample part of the utility distribution P&ID . . . . . . . . . . . . . . . . . Typical segment of interconnecting line P&ID . . . . . . . . . . . . . . . Sample part of auxiliary P&ID . . . . . . . . . . . . . . . . . . . . . . . . . . . . Sample special control P&ID for a compressor (referred to in Fig. 3.6) . . . . . . . . . . . . . . . . . . . . . . . . . . . . . . . . . . . . . . . . . . Typical line numbering system . . . . . . . . . . . . . . . . . . . . . . . . . . . . Symbols of P&ID explained . . . . . . . . . . . . . . . . . . . . . . . . . . . . . . A typical level control loop . . . . . . . . . . . . . . . . . . . . . . . . . . . . . . A typical pressure control loop . . . . . . . . . . . . . . . . . . . . . . . . . . . Typical temperature and flow control loops . . . . . . . . . . . . . . . . . Typical cascaded level and flow control loop . . . . . . . . . . . . . . . . Typical ratio control of two flows . . . . . . . . . . . . . . . . . . . . . . . . . Alarm and trip: a together b separate . . . . . . . . . . . . . . . . . . . . . . Typical P&ID showing control scheme of a distillation column . . . . . . . . . . . . . . . . . . . . . . . . . . . . . . . . . . . . . . . . . . . . . . . Typical Master plot plan (Indicative only—not to scale) . . . . . . . a–e Basic types of layouts . . . . . . . . . . . . . . . . . . . . . . . . . . . . . . . Typical unit plot plan (indicative only) . . . . . . . . . . . . . . . . . . . . . Typical 3D model of a chemical plant [8] . . . . . . . . . . . . . . . . . . . Sample specification sheet for storage tank . . . . . . . . . . . . . . . . . . Sample specification sheet for shell and tube heat exchanger . . .

36 40 44 60 61 66 67 68 69 69 70 71 72 73 77 77 78 78 79 80 80 82 90 93 94 95 112 114 xix

xx

Fig. 4.3 Fig. 4.4 Fig. 4.5 Fig. 4.6 Fig. 5.1 Fig. 5.2 Fig. 5.3 Fig. 6.1 Fig. 6.2 Fig. 6.3 Fig. 6.4 Fig. 6.5 Fig. 6.6 Fig. 6.7 Fig. 6.8 Fig. 6.9 Fig. 6.10 Fig. 6.11 Fig. 6.12 Fig. 6.13 Fig. 6.14 Fig. A.1 Fig. A.2 Fig. A.3 Fig. A.4

List of Figures

Sample specification sheet for centrifugal pump . . . . . . . . . . . . . Figure for Problem 4.1 . . . . . . . . . . . . . . . . . . . . . . . . . . . . . . . . . . Figure for Problem 4.2 . . . . . . . . . . . . . . . . . . . . . . . . . . . . . . . . . . Figure for Problem 4.3, [Legend: GT-gate valve, HE-heat exchanger, OR-orifice and CV-control valve] . . . . . . . . . . . . . . . . Schematic of cash flow . . . . . . . . . . . . . . . . . . . . . . . . . . . . . . . . . . Classification of the methods for profitability analysis . . . . . . . . . Cash flow diagram against time [3] . . . . . . . . . . . . . . . . . . . . . . . . Levels of work breakdown structure (WBS) . . . . . . . . . . . . . . . . . Example of WBS—construction of a building . . . . . . . . . . . . . . . Example of Bar chart of an R&D project . . . . . . . . . . . . . . . . . . . Example of order control by Gantt chart . . . . . . . . . . . . . . . . . . . . Example of machine control by Gantt chart . . . . . . . . . . . . . . . . . Network for Example 6.1 . . . . . . . . . . . . . . . . . . . . . . . . . . . . . . . . Gantt chart for Example 6.2 . . . . . . . . . . . . . . . . . . . . . . . . . . . . . . Critical path network for Example 6.2 . . . . . . . . . . . . . . . . . . . . . . Critical path network for Example 6.3 . . . . . . . . . . . . . . . . . . . . . . Normal critical path network of problem 6.5 . . . . . . . . . . . . . . . . Crashed CPM network for Example 6.5 . . . . . . . . . . . . . . . . . . . . Risk management cycle—a schematic . . . . . . . . . . . . . . . . . . . . . . Likelihood-Consequence matrix for risk assessment . . . . . . . . . . Bow-tie diagram for cause and consequence of an event . . . . . . . PFD of Example-I:DM Water cooling plant section—See Fig. A.1 and Table A.1 . . . . . . . . . . . . . . . . . . . . . . . . . . . . . . . . . . Preliminary P&ID of a DM Water cooling plant section—See Fig. A.2 . . . . . . . . . . . . . . . . . . . . . . . . . . . . . . . . . . . PFD for Exampe-II: Boiler Feed Water pumping section—See Fig. A.3 and Table A.2 . . . . . . . . . . . . . . . . . . . . . . . P&ID for Example-II: Boiler Feed Water pumping section—See Fig. A.4 . . . . . . . . . . . . . . . . . . . . . . . . . . . . . . . . . . .

115 116 117 120 136 155 156 182 183 185 186 187 189 191 192 194 197 198 200 201 201 206 209 210 211

List of Tables

Table 1.1 Table 1.2 Table 2.1 Table 2.2 Table 3.1 Table 3.2 Table 3.3 Table 3.4 Table 3.5 Table 3.6 Table 3.7 Table 3.8 Table 3.9 Table 3.10 Table 3.11 Table 3.12 Table 3.13 Table 3.14 Table 3.15 Table 3.16 Table 4.1 Table 4.2 Table 4.3 Table 4.4 Table 4.5 Table 4.6 Table 4.7 Table 4.8

Duties of project engineers belonging to various teams . . . . . . . The code of ethics adopted by IIChE [9] . . . . . . . . . . . . . . . . . . Examples of maximization and minimization problems in the design . . . . . . . . . . . . . . . . . . . . . . . . . . . . . . . . . . . . . . . . . Comparing scaling up by number and by size . . . . . . . . . . . . . . Symbols (for PFD and P&ID) of a few equipment [3] . . . . . . . Symbols of a few valves and piping materials . . . . . . . . . . . . . . Stream specification in PFD . . . . . . . . . . . . . . . . . . . . . . . . . . . . . Codes generally used for pipeline numbering . . . . . . . . . . . . . . Symbols generally used for types of pipeline and instrumentation . . . . . . . . . . . . . . . . . . . . . . . . . . . . . . . . . . . Recommended velocities for a few fluids in pipes . . . . . . . . . . . Recommended colour codes for pipelines . . . . . . . . . . . . . . . . . Recommended symbols for instrument position . . . . . . . . . . . . Instrument identification letter/legend . . . . . . . . . . . . . . . . . . . . . Operating range and applicability of pumps . . . . . . . . . . . . . . . . Operating range and applicability of heat exchangers . . . . . . . . Separation processes and corresponding governing properties . . . . . . . . . . . . . . . . . . . . . . . . . . . . . . . . . . . . . . . . . . . . Operating range and applicability of separation processes . . . . Different types of dryers—a brief introduction . . . . . . . . . . . . . Influence of safety and hazard issues on design . . . . . . . . . . . . . Guide words and explanations for HAZOP analysis . . . . . . . . . Pressure drop calculation sheet for centrifugal pump . . . . . . . . Pressure drops in pipelines . . . . . . . . . . . . . . . . . . . . . . . . . . . . . . Equivalent lengths of various fittings [4] . . . . . . . . . . . . . . . . . . Process data to specify for a pressure gauge . . . . . . . . . . . . . . . . Process data to specify for a pressure gauge . . . . . . . . . . . . . . . . Sample line list . . . . . . . . . . . . . . . . . . . . . . . . . . . . . . . . . . . . . . . Sample valve list . . . . . . . . . . . . . . . . . . . . . . . . . . . . . . . . . . . . . . Typical format for engineering change notice (ECN) . . . . . . . .

10 13 31 43 63 64 65 73 74 74 75 76 76 96 98 99 100 100 102 104 122 123 123 124 124 125 126 127 xxi

xxii

Table 4.9 Table 5.1 Table 5.2 Table A.1

Table A.2

List of Tables

Format for comparative statement for procurement . . . . . . . . . . Cost of various components as fractions of FCI [1] . . . . . . . . . . Value of the exponent for different process equipment . . . . . . . Stream details for DM Water cooling plant section (generally such a table is placed below the PFD on a drawing sheet) . . . . . . . . . . . . . . . . . . . . . . . . . . . . . . . . . . . . Stream Details for Boiler Feed Water pumping section . . . . . . .

129 143 145

207 210

Chapter 1

Introduction

For chemical engineers, Project Engineering is the journey of transforming a chemical process into a chemical plant where raw materials are converted to finished products in an industrial scale. This transformation needs various parties and different types of works including engineering design, financial analysis and managerial activities. This chapter introduces the various aspects of the construction of a plant as well as major duties of project engineers belonging to different parties who install and commission a process plant starting from conceptualization. Learning Objectives • To understand what are Project, Project Engineering and Process Engineering. • To be aware of the various phases of a chemical engineering project. • To know about the duties and responsibilities of a project engineer.

1.1 Preamble Chemical engineering is one of the branches of engineering that produces finished products by converting various raw materials according to the rules of chemistry using engineering principles of various unit operations. The facility where various unit operations are combined is called a chemical process plant. Design, construction and commissioning of such process plant require simultaneous application of engineering and management skills. Some financial aspects are also to be included. This combined assignment is referred to as ‘Project Engineering’. Basically, a project engineer has to get the job done. There are many parties associated with a particular project and duties of a project engineer vary according to the party s/he is working for. While discharging her/his duties as a professional, a project engineer should not forget the ethical principles s/he has to follow including her/his obligations to the environment.

© The Author(s), under exclusive license to Springer Nature Singapore Pte Ltd. 2022 S. Chakrabarti, Project Engineering Primer for Chemical Engineers, https://doi.org/10.1007/978-981-19-0660-2_1

1

2

1 Introduction

1.2 Engineering, Science and Chemical Engineering Engineering is the skill to apply the results of basic science. They aim at producing some product or service that is useful and economical in a finite and acceptable time scale. Engineering is a profession not only of imagination but also of bringing into reality a new product or service, which can accomplish a specific target robustly and economically following all safety measures for humankind and environment. Engineers conceive, design, implement and operate solutions for certain problems. Engineers generally assemble standard components in more or less standard processes to produce a product or service. Engineering Science is the application of scientific principles by which engineered artefacts or services are produced. A good example is Thermodynamics, which was developed to understand the operation of a steam engine whereas the steam engine was developed without the help of thermodynamics. In case of applied science, scientific principles are used to understand the natural processes and thereby a real-world problem is solved. Though generally, engineers take the initiative, Applied Sciences and Engineering are not synonymous. Chemical Engineering is, however, not a branch of Chemistry but a category of Engineering. This broad discipline involves physical, chemical or biological transformation of matter or energy into various forms useful for human civilization. The transformation should be economical in using finite resources and the process should not compromise with the conservation of environment. The term was coined by George E Davis in 1904 to describe the use of mechanical equipment in the chemical industry. The subject was developed in the early twentieth century as a subject comprising basically of Unit Operations and Unit Processes. The book entitled Principles of Chemical Engineering, published by Walker, Lewis, and McAdams (1923) is considered to be one of the first books on the basic principles of Chemical Engineering [1]. Chemical engineers upscale the processes developed in the laboratories for the commercial production of commodities and then continuously keep on maintaining and improving those processes [2]. Chemical engineers actually transform raw materials into useful products like chemicals, fuels, food, medicines and biomaterials, in commercial scales by designing and maintaining equipment and processes. Moreover, they should also take care of the environmental impacts while maintaining the optimum profits and production rates. The journey from conception to production of a product is generally done in the form of a Project. When the product is a chemical, the work of design to construction is a chemical engineering project [3].

1.3 Project and Project Engineering

3

1.3 Project and Project Engineering In the sixteenth century, the word Project was first used by the architects, the original Latin word literally meant throw forward that is a trajectory having relation with a particular space and time. The process implies a particular point of departure or starting point that is used as a base from which something is thrown forward towards a predefined goal. Oxford dictionary defines “Project” as ‘An individual or collaborative enterprise that is carefully planned and designed to achieve a particular aim’. The project management body of knowledge (PMBOK) defines a project as ‘A temporary endeavour undertaken to create a unique product, service or result’. Since by definition, a project is temporary in nature, it must have definite starting and ending points. The end of a project is reached when the objectives or targets of a project are achieved or it is understood that the objectives cannot be accomplished with the available and finite resources; sometimes a project ends when the requirement of the project does not exist anymore, for example, the product may become obsolete/unwanted or the owner may not afford the funds [4].

1.3.1 Characteristics of a Project Projects have purpose: Since the projects have defined aims and objectives to achieve, they have definite purposes. Projects are aimed at analyzing needs and solving a particular problem by which a long-lasting social impact can also be achieved. Projects are realistic: The aim of a project should be achievable within realistic time and resources, hence all the practical aspects should be considered. Projects are limited by time and space: They have a beginning and an end and are implemented in a specific place, context and time frame. Projects are complex: Different types of planning and implementation skills are necessary for all the parties involved in the project, to complete it. Hence, a project is multidimensional and complex in nature. Projects are collective: No project is possible by any individual, it is carried out by group activities and interactions between several groups. Projects are unique: No two projects can be exactly the same. All projects address a specific problem in a specific context, place and time. In that sense, all projects are unique. Projects are adventures: Since all projects are unique, they always involve some uncertainty and risk due to some unknown parameters. Projects should be assessed: Projects should be planned in several sub-parts and should be broken into measurable components that can be quantitively evaluated.

4

1 Introduction

Projects are made up of stages: Projects should have distinct and identifiable stages [4]. A project is completed by performing different activities. For example, the construction of a house is a project. It consists of ground excavation, foundation, building of walls, roof casting, fixing doors and windows, painting, plumbing and electrical work and so on. A project consumes different types of resources including personnel, materials, money and time. Thus, in addition to the conventional definitions, a project can be defined as an organized programme of a group of previously planned but unique activities that must be completed using the available resources within the scheduled time limit. A project engineer is fully responsible for everything that has to be done within the assigned area to accomplish the objectives of the project. Total responsibility includes planning each activities and controlling them. Priorities should be given to keep it within the stipulated time schedule, to limit the expenditure within budget and to use estimated manhours. So Project Engineering means to get the assigned job done within the allotted resources. A Project Engineer should be conversant with the management skill along with the engineering skills. Basic knowledge of the engineering principles of all disciplines like electrical, civil and instrumentation are necessary for a Project Engineer to co-ordinate the interface engineering. The scope of management includes not only scheduling and monitoring but also financial and fund management. Since a Project Engineer is responsible for completion of a project, s/he has to comply with a management checklist to ensure that s/he is taking care of all the aspects of a project. Duties and responsibilities of a project engineer can broadly be classified into the following categories: • • • • • • • • • • • •

Feasibility study and cost estimate Setting standards and design basis Planning and controlling Complying with safety and environmental standard Identifying, assessing, and minimizing risks Achieving and maintaining quality standards in procurement and construction Complying with a planned schedule and preparing a progress report Controlling costs within the budget Controlling interfaces between various parties including change management Solving technical and commercial issues Construction Commissioning and start-up for production.

As it is apparent from the above list, the duties of a project engineer can be divided into three aspects—Engineering, Finance and Management. Before elaborating all the points let us know about the various types, phases and parties involved in a chemical engineering project. Duties of a Project Engineer or Project Manager depend on the party whom that particular Project Engineer belongs

1.4 Different Parties Involved in a Chemical Engineering Project

5

to. So instead of generalizing the duties and responsibilities of a Project Engineer, we should discuss his/her duties from the various perspectives of the different players or participants associated with a chemical engineering project.

1.4 Different Parties Involved in a Chemical Engineering Project If one is aware of the different stakeholders participating in a chemical engineering project, it would be easier to understand the division of the phases of a project among the various parties. Generally, the following parties are present in a Chemical Engineering project. Owner: A person, group or company who owns and funds the plant. Owner is referred to as Client by the other parties appointed by him/her/them. Feasibility consultant: An agency that studies the feasibility of the project from the scratch. Basic Engineering consultant: A person, group or company who supplies the technical know-how or the basic engineering package. Detailed Engineering consultant: A company employed by the client or owner to detail out the engineering and generate necessary drawings and documents based on the basic engineering package. Management consultant: Sometimes a management consultant may be employed by the owner to monitor or control the progress of work. Different tools and software packages may be used to prepare the progress report. However, for small projects, project engineers of the owner and the respective parties jointly monitor the status of the project from time to time. Vendors: Various fabricators and suppliers for the bought-out equipment and instruments required for the plant. The contractor for construction: A company who constructs or erects the plant with the equipment and instrument procured by the owner based on the detailed engineering drawings and documents provided by the detailed engineering consultant. Government Departments and statutory bodies: For the construction of a chemical plant, approvals or permissions are required from several government departments and statutory bodies such as Ministry of Industries and Pollution Control Board. Turnkey Contractor: Sometimes the owner does not want to be involved too much in the project to appoint different consultants and contractors separately to get the project done. Instead, they want a single contractor who would design, procure and install the whole plant and hand over to them so that they can only ‘turn the key’ for starting the plant. This type of contract is also known as Engineering, Procurement and Construction (EPC) contract.

6

1 Introduction

1.5 Categories and Phases of a Chemical Engineering Project Depending on the uniqueness involved, chemical engineering projects can be categorized into three types as below: • Additions and/or modifications of the existing plant are usually carried out by the design group of the plant itself. • Modification for new production capacity to meet growing market demand of the product or sale of established processes by contractors. It involves only minor changes in the existing design, with repetition of the major part. • Entirely new processes that are developed or scaled-up from the bench-scale laboratory research, through pilot plant to a commercial scale. Though the process is new, most of the unit operations and process equipment are based on established designs. For this last type, the first step will be to construct a rough block diagram showing the main stages in the process and to list the primary objectives and major constraints for each stage. The types of unit operations and equipment to be used should be considered out of experience. The design of a chemical manufacturing process plant can be divided into two broad phases as described below: PHASE-I The first phase of a chemical engineering project includes initial selection of the process to use, selection, specification and design of equipment and finally issuing of the process flow sheets covered under the umbrella of Process Design. Generally in a consultancy organization, Phase I is done by a group of chemical engineers constituting a Process Design Group. This group is responsible for developing process drawings like PFD and P&ID. PHASE-2 In the second phase of a chemical project, the activities are the detailed engineering design of mechanical, structural, civil and electrical works including the design and specification of the ancillary and utility services. Respective specialist design groups of different disciplines are responsible for each of such activities. Other non-technical specialist groups are responsible for cost estimation, purchase and procurement of equipment and materials. The steps in the design process cannot be separated and the sequence of events cannot be firmly fixed. Among the design groups of different disciplines, there should be a continuous communication, which is the essence of the interdisciplinary nature of Project Engineering. A project manager is generally a chemical or mechanical engineer and is usually responsible for the co-ordination of the whole project [4].

1.6 Life Cycle of a Chemical Engineering Project

7

1.6 Life Cycle of a Chemical Engineering Project 1.6.1 Pre-project Work The requirement of a particular project or Problem is identified by the owner. Many possible alternative ideas for solutions are proposed and a Feasibility Consultant is appointed by the owner to see whether any of the proposed ideas will be worth implementing a solution. If the proposed solution is found technically and economically feasible, then the owner prepares a rough specification of the problem to invite proposals for solutions from several basic engineering package providers or basic engineering consultants. Preliminary cost estimation is also done. After receiving technical and commercial bids from different basic engineering consultants, the engineering and management team of the owner compares the offers and ultimately finalizes one Basic Engineering consultant to work for them. From this point onwards, the project starts for the owner and for the basic engineering consultant [5, 6].

1.6.2 Starting the Project Once the basic engineering consultant is fixed, the scope of work is mutually agreed between the owner and them. Code, standard and design basis is decided. A design specification is prepared by the Basic Engineering Consultant to decide a suitably detailed engineering consultant. A notice inviting tender or NIT is released to invite bidders for Detailed Engineering work. After receiving technical and commercial bids from different detailed engineering consultants, the engineering and management team of the owner compares the offers and ultimately finalizes one detailed engineering consultant to work for them. From this point onwards, a project starts for the detailed engineering consultant.

1.6.3 Starting and Carrying Out Engineering Work Drawings and documents The detailed engineering consultant prepares a draft piping and instrumentation diagram (P&ID) based on the process flow diagram (PFD) and the preliminary P&ID developed by the basic engineering consultant. The equipment layout is also developed based on the rough sizing provided by the basic engineering package. Based on the preliminary design data from the basic engineering consultant, specifications for the fabricated and bought out items are prepared by the detailed

8

1 Introduction

engineering consultant. For the preparation of specification, some calculations are required in addition to the design calculations provided by the basic engineering consultant. It may be noted that the design of several items of different categories (like equipment, electrical items, instruments, structural) is taken care of by the respective specialized groups belonging to the detailed engineering consultant. Interactions between such groups lead to revisions of the documents. Based on the specifications, quotations are invited for the bought-out items from qualified vendors. For fabricated items, if fabricated by the owners, general arrangement (GA) drawings are to be prepared by the detailed engineering consultant. Otherwise, the vendor prepares the same. At this stage, more accurate cost estimation is done and compared with the preliminary cost estimate. A move of capital investment is to be taken. Procurement If procurement assistance is within the scope of work of the detailed engineering consultant, they compare the quotations received from several vendors and recommend one vendor to the owner for purchase of the item. Selection of the vendor is based on the technical merit of the item offered and its price. When a particular vendor is selected for supplying a particular item, that point onward, the project starts for the vendor. Incorporation of vendor’s data in the documents and drawings On receiving a purchase order, the vendors provide respective data and drawings to the detailed engineering consultants so that those can be incorporated in the P&ID and piping layout drawings. Bill of materials (BOM) for piping, electrical or instrumentation work will also be influenced by the vendor drawings/ documents. Approval of statutory bodies Before starting the work to install or erect a chemical plant, permission should be taken from Government Departments (the department will depend on the nature of the chemical) and statutory bodies like pollution control boards or local municipal authorities. Applications for such permissions often need project descriptions or drawings as attachments. These documents should be prepared by the project engineers. Starting construction at the plant site After the design is fixed, a site office is mobilized with sufficient funds and manpower. Facilities like site office, labourer’s housing, storage for equipment and construction machinery and site security have to be established. First, civil construction starts and then mechanical equipment, electrical equipment and instruments are installed.

1.7 Project Engineering and Process Engineering

9

Pre-commissioning, commissioning and start-up After the erection or installation of equipment, pipelines and instruments with their connections, the pre-commissioning is planned. Commissioning means to put the plant into operation and pre-commissioning is the preparation for it. In this stage, the equipment and interconnections are cleaned and tested for the tolerance at specified temperature and pressure. Engineers from all parties are involved in pre-commissioning. However, the demarcation between pre-commissioning and commissioning is not very sharp. Commissioning of the plant is accompanied by a written operation and maintenance (O&M) manual to be prepared by the project engineering group. O&M manual for the vendors is also part of this manual. Operators of the owner company are to be trained to operate the plant.

1.7 Project Engineering and Process Engineering Process engineering is the knowledge, skill or expertise required to design, optimize, analyze, develop, construct and operate the processes in which the raw materials change its shape, physical state or chemical nature to yield a useful product. Process engineering is generally applied to design and modify processes in the economically significant industrial sectors. Process Engineering should not be confused with Industrial chemistry. Because the scope of Industrial Chemistry is limited within the study of the chemical products and their manufacturing processes rather than its design or development. The Process Engineer draws the process flow diagram (PFD), which is a schematic visualization of the industrial process. For each line or stream in the PFD, a process engineer determines the temperature, pressure, flow rate and chemical composition of the corresponding flow stream. To do this, the process engineer has to visualize what is happening within a piece of particular equipment or pipeline, therefore should be able to analyze it chemically and to have appropriate measuring instrument or technique to determine the qualitative and quantitative characteristics of that particular flow. In the life cycle of a chemical project described in the previous section, Process Engineers are responsible for preparing the basic engineering package comprising basic material and energy balance, PFD and preliminary design/specification of the equipment and instruments including the preliminary P&ID. In the two phases of the chemical engineering projects described before, the first phase is generally carried out by the process engineers. Process Engineers are generally not involved in the cost estimations, project planning, scheduling or monitoring. These are generally done by the Project Engineers. On the contrary, Project Engineers sometimes have to be involved in the process design especially for small projects or companies, where separate process groups cannot be afforded [3].

10

1 Introduction

1.8 Duties and Responsibilities of Project Engineer—Various Perspectives By now it must be clear that a project is looked into different perspectives by the different parties involved in it. So for each of them, starting and endpoint of the same project are different. For example, for a Project Engineer from the basic engineering consultant company, completion of the basic engineering package is the endpoint of the project whereas for a Project Engineer, working for a detailed engineering consultant, that may be the starting point of the project. Therefore to indicate a general list for the duties and responsibilities of a project engineer may be over-simplification. Duties of the project engineer of the respective parties of a project are tabulated below Table 1.1. It may be observed from the above table that though all are the duties of a project engineer, all of the duties are not to be done by the same project engineer. Duties of a particular person working as a Project Engineer will depend on the party for whom he or she is working.

Table 1.1 Duties of project engineers belonging to various teams Sl. no

Duty

Party

1

Preparation of feasibility report including site selection

Feasibility consultant

2

Initial cost estimate

Owner and feasibility consultant

3

Preparation of pre-bid documents for the project Defining the scope of work and process Defining battery limit Utility specification Defining code standard and design basis including SHE issue

Owner

4

Preliminary estimation of cost and Management consultant man-hour Planning and scheduling of the project as a whole

5

Basic Engineering package Material and energy balance and process flow diagram (PFD) Development of preliminary P&ID from PFD Preliminary equipment layout Utility specification, flow and distribution Preliminary sizing and specification of equipment and instruments

Basic Engineering consultant

(continued)

1.9 Engineering and Environmental Ethics in Project Engineering

11

Table 1.1 (continued) Sl. no

Duty

6

Detailed engineering Detailed engineering consultant Drawing and document schedule Equipment list and equipment layout Updating P&ID Specification of equipment and instrument BOM and specification for piping materials including valves Fabrication drawings Piping layout drawing Civil, electrical and instrument layout drawings

Party

7

Environmental Impact Analysis (EIA)—HAZOP and HAZAN studies Risk and reliability analysis

Detailed engineering consultant

8

Procurement Enquiries and follow-up vendors Raise technical queries to vendors Preparation of comparative statements Recommendation and placement of orders

Owner or detailed engineering consultant

9

Interface engineering Freezing basic engineering package Updating P&ID with vendor data Design change report and its circulation

Detailed engineering consultant

10

Application for approval of government departments and statutory bodies Detailed cost estimate

Owner with the help from the detailed engineering consultant

11

Supply of equipment and instrument as per Vendors order to the site

12

Construction of plant as per approved design

Construction contractor or site office of the owner

13

Pre-commissioning and commissioning

All the parties

14

Operation and maintenance manual for the whole plant including start-up and shutdown protocol

Basic and detailed engineering consultant along with the vendors

15

Project monitoring and status updating from time to time

Management consultant and/or owner with respective parties

1.9 Engineering and Environmental Ethics in Project Engineering The word ‘ethics’ originated from the Greek word ‘ethikos’ referred to distinctive character, spirit or attitude. Our social existence lies in the person-to-person or face-to-face relationship between at least two persons. Ethics is usually practised in communities, in the plural. Ethics or moral philosophy is therefore a systematic

12

1 Introduction

endeavour to understand moral concepts and justify moral principles and theories. Ethics establish principles of right behaviour or in other words, action guidelines of individual or group in a society [4]. Though some people equate the practices of morality and law, basically they are not the same. Some aspects of morality are not covered by the law. For example, telling lie is condemned in every society, though there is no general law to punish a liar according to penal code. Ethics is also different from etiquette. Etiquette determines what is polite behaviour rather than what is right behaviour in a deeper sense. For example, a person may not know the table manners during a formal dinner—he lacks etiquette, but if a person, who is very smart with table manners, behaves badly with the waiter, will be considered as a person with good etiquette but poor ethics [7]. Engineering ethics refer to the personal conduct of engineers so that they can uphold and advance the aspects like integrity, honour and dignity of the engineering profession. The conduct or behaviour of an engineer has obligations to the employer and/or client, colleagues and co-workers, the public and to the environment. No engineering activity should be conducted by an engineer unless the engineer has complied with the ethical codes of conduct. Various professional bodies have adopted several codes of conducts for their members. American Institute of Chemical Engineers (AIChE), Indian Institute of Chemical Engineers (IIChE) and Institute of Chemical Engineers (IChemE) are three such institutes that have distinct ethical guidelines. If we look into the ethical guidelines of AIChE [8], we will see that as Chemical Engineering professional, one has to continuously update her/his professional knowledge, at the same time s/he has to take care of the environmental and safety issues of the plant. Moreover, s/he should maintain high moral and ethical standard while discharging her/his duties. The basic ethical codes of conduct for the engineers as enunciated by most of the professional bodies are more or less the same. The ethical codes of conduct specified by IIChE are reproduced in the following Table 1.2 with the permission of IIChE. In all cases, the basic and common ethical points are: • • • • • •

Honesty, sincerity and integrity Respect to client, employer/employee, colleagues and subordinates Responsibility for environment Responsibility for safety and health of the public Updating knowledge and skill The welfare of society as a whole.

It shows that despite the responsibility to design, construct and to operate a chemical plant, a Project Engineer has an obligation to save and protect the environment as well. Chemical Engineering projects are generally hazardous to the environment, in some form or other. Setting up a plant on a piece of land where raw materials and products both are chemicals, must impart some adverse effects to the surrounding environment. However, an industry produces some useful products, generates employment and improves civic amenities of the locality.

1.9 Engineering and Environmental Ethics in Project Engineering

13

Table 1.2 The code of ethics adopted by IIChE [9] “I AM, BY PROFESSION, A CHEMICAL ENGINEER. In my profession, I take great pride, but without vain glory; to it, I owe solemn obligations that I am eager to fulfil. As a member of the engineering community, I am here to create a new world that never has been and see the world that has been given to me by my predecessors. I have been taught to conserve material and energy by which I will live to welcome the next generation and hand over the heritage to the fledgling members of my profession As a chemical engineer, I will participate and advance none but honest enterprise and be both a benefactor and beneficiary. To him that has engaged my services, as employer and client, I will give the sincerest of my performance, integrity and fidelity. I know my efforts will be a drop in ocean but ocean shall we have through collective wisdom and knowledge of all members of my profession When called, my skill and knowledge shall be given without reservation and prejudice for the good of not only humankind but also other members of the biosphere. From special capacity springs the obligation to use it well in the service of humanity; I accept the challenge that this implies. Well-equipped now, I am ever eager to serve my profession as a member of the Indian Institute of Chemical Engineer. I shall never close my eyes to new ideas and knowledge and change any erroneous practices that might have served my profession to-date Zealous of the high repute of my calling, I will strive to protect the interests and the good name of any engineer that I know to be deserving, but I will not shrink, should duty dictate, from disclosing the truth regarding anyone that, by an unscrupulous act, has shown himself unworthy of the profession Since antiquity, human progress has been conditioned by the genius of my professional forebears. By them have been rendered usable to mankind Nature’s vast resources of material and energy. By them have been vitalized and turned to practical account the principles of science, engineering and the revelations of technology. Except for this heritage of accumulated experience, my efforts would be feeble and lost. My vision has been sharpened as a result of this and it is my bounden duty to protect the environment and be in harmony with nature. I will strive hard to overcome ignorance and prejudice and bestow the benefit of my knowledge on the younger generation so that they improve upon my mistakes to make this world a better place to live. I dedicate myself to the dissemination of engineering knowledge, and, especially to the instructions of younger members of my profession in all its arts, ethos and traditions I am seized of the great challenge which our profession is going to face in future and that is to use our skills to improve the quality of life-foster employment, advance economic and social development and protect the environment. This challenge encompasses the essence of sustainable development. I will work with others to make the world a better place for future generations As a chemical engineer, it will be my endeavour to design processes and products which are innovative, energy-efficient and cost-effective, make the best use of scarce resources and ensure that waste and adverse environmental impact are minimized; achieve the biggest standards of safety in making and using products of all kinds; provide the processes and products which give the people of the world, shelter, clothing, food and drink, and which keep them in good health; work with other disciplines to seek solutions; engage in honest and open dialogue with the public on challenges presented by the manufacture of the products which the public requires; promote research to allow the profession to respond fully to global demands; encourage the brightest and best young people into the profession; and promote lifelong professional development. I acknowledge that this challenge cannot be met by my efforts alone, but this does not lessen the responsibility to pursue it. Therefore I must co-operate with other fellow colleagues and recognize each other’s efforts in striving to meet this challenge I have found a new faith that traverses all narrow boundaries of nations, castes and creeds and it envisions me to look at society afresh. To my fellows I pledge, in the same full measure I ask of them, integrity and fair dealing, tolerance and respect, and devotion to the standards and the dignity of our profession and the Institute; with the consciousness, always, that our special expertness carries with the obligation TO SERVE HUMANITY WITH COMPLETE SINCERITY”

14

1 Introduction

A responsible and ethical Project Engineer should therefore try to minimize the (adverse) environmental impacts. Some of the actions that can be taken are: • Conducting environmental impact analysis (EIA) very rigorously and following its recommendations. • HAZOP and HAZAN analysis as well as safety audit should be performed at various stages. • Selection of comparatively barren land for the plant site so that local agriculture is not affected. • Providing effective effluent treatment plants for gaseous, liquid and solid effluents of the plant. • Plantation of new trees and landscaping as compensation of the trees cut for the plant site—for most of the industries, about 33% of the area of the proposed project should be under plantation according to the current environmental rules of India. • Organization of several awareness programmes on health and environmental issues among the local residents [10].

1.10 Risk and Reliability Analysis Loss prevention and safety are two important aspects of a chemical process plant. ’Loss’ includes not only the loss of lives or money but also that of the environment or production time. Therefore, the risk and reliability of a plant should be managed efficiently. Summary The commodities we use every day are mostly manufactured by chemical engineers. They transform raw materials into useful products in an economical way. The facilities where these commodities are produced are chemical process plants. ‘Projects’ are collective and novel endeavours to establish or augment such a plant. There are several stages of a chemical project where the duties and responsibilities are clearly defined for a project engineer working for a particular stakeholder of the project. However, there is a subtle difference between the project and the process engineer. Project engineers should obey certain ethical rules while maximizing production or profit. They should not compromise safety, work ethics or environment while striving for their professional excellence. Exercise 1. 2. 3. 4.

A company decides to construct a 100 TPD cement plant at Purulia within 2 years from now. Is this a project? Justify your answer. Describe the characteristics of a project. What is the difference between a Project Engineer and a Process Engineer? What are the major stages of a Project in its life cycle? Introduce briefly the sub-components of the detailed engineering stage.

1.10 Risk and Reliability Analysis

5. 6.

7.

15

What are the different phases of a chemical plant project? Point out the major differences between the two phases. Two project engineers, one belongs to the company that is providing the basic engineering package and the other belongs to the company that is conducting the detailed engineering. What would be the differences in the duties and responsibilities of the two project engineers? What are the essences of the ethical principles of a design (chemical) engineer? Describe three universal ethical principles for the project engineer. Why project or design engineers should strictly comply with not only the engineering ethics but also the environmental ethical guidelines?

References 1. Moran, S. (2015). An applied guide to process and plant design (Illustrated ed.). ButterworthHeinemann. 2. IDC Technologies. (2003). Practical fundamentals of chemical engineering. Pty 2003, Ver 1.2. 3. Dal Pont, J.-P. (2012). Chemical engineering and process engineering. In J.-P. Dal Pont (Ed.), Process engineering and industrial management (Ch. 4). Wiley-ISTE. ISBN: 978-1-848-213265. 4. Sinnot, R. K., & Towler, G. (2009). Coulson and Richardson’s chemical engineering series. In Chemical engineering design (Vol. 6). Elsevier (Butterworth-Heinemann). 5. Turton, R., Shaeiwitz, J. A., Bhattacharyya, D., & Whiting, W. B. (2018). Part of the international series in the physical and chemical engineering sciences series. In Analysis, synthesis, and design of chemical processes (5th ed.). Pearson. ISBN-13: 978-0-13-417740-3. 6. Watermeyer, P. (2002). Handbook for process plant project engineers. Professional Engineering Publishing Limited. 7. Pojman, L. P. (1998). Environmental ethics—Readings in theory and application (2nd ed.). Wadsworth Publishing Company. 8. American Institute of Chemical Engineers. Retrieved October 30, 2021, from https://www. aiche.org/about/governance/policies/code-ethics 9. Indian Institute of Chemical Engineers. Retrieved October 30, 2021, from https://www.iiche. org.in/code_ethics.php 10. Grubbe, D. L. (2015). Ethics-examining your engineering responsibility. CEP-February 2015. American Institute of Chemical Engineers.

Chapter 2

Pre-project Work

Before starting the design and construction of a chemical process plant, a few groundworks are necessary which are covered under Pre-project Work. For example, without knowing the optimum production rate or the optimum number of effects in an evaporator train, one cannot start designing a process plant; or one should not proceed towards designing a plant unless it is feasible. Feasibility study, environmental impact assessment, scaling up or down and optimization are some of the pre-project works described in this chapter. Learning objectives: • To know about the considerations to be made before a project starts. • To be aware of the importance of the pre-project works.

2.1 Preamble As described in the previous chapter, pre-project work is the first step towards the installation of a process plant. Pre-project work generally includes feasibility study, environmental impact analysis (EIA), initial cost estimate and preparation of bid document or notice inviting tender (NIT) for basic and detailed engineering, supply and construction work. In this chapter, outlines of the feasibility study report, environmental impact assessment and preparation of bid document will be discussed. The initial cost estimate does not come directly into the purview of the Project Engineer. However, it will be discussed in Chap. 5 of financial aspects. One more thing that is important for designing is optimization. The design engineer has to optimize many of the design aspects based on which the plant is built. Production rate and pipe diameter are two such examples. Sometimes, the designer has to make a trade-off between two options during freezing a design. Some optimum quantities not only form the basis of design but also influence the cost of the plant. A very brief overview

© The Author(s), under exclusive license to Springer Nature Singapore Pte Ltd. 2022 S. Chakrabarti, Project Engineering Primer for Chemical Engineers, https://doi.org/10.1007/978-981-19-0660-2_2

17

18

2 Pre-project Work

of the application of optimization on process design is included as one of the preproject works; however, mathematical optimization technique is out of the scope of the present discussion.

2.2 Feasibility Study—A Brief Overview The study on which management or investor depends for deciding their investment is the feasibility study. The study report must contain the following: Definition of the idea. Developing the process application of the idea. Evaluating the process proposals technically and economically. Reviewing the current proposal and testing, re-designing, optimization and reevaluation of the alternative proposals or possibilities. Feasibility studies are therefore structured ways to assess the technical, social, financial and environmental viability or practicality of a project and are used to make an informed decision about whether the project should be implemented. General characteristics (a)

(b)

A feasibility study should provide all data necessary for an investment decision. The commercial, technical, financial, economic and environmental prerequisites for an investment project should therefore be defined and critically examined based on alternative solutions already reviewed in the pre-feasibility study. A feasibility study should be carried out only if the necessary financing facilities, as determined by the studies, can be identified with a fair degree of accuracy. Possible project financing should be considered as early as the feasibility study stage because financing conditions have a direct effect on total costs and thus on the financial feasibility of the project. The following topics should be covered in a standard feasibility study report.

Chapter I: Executive Summary For the convenience of preparation and interpretation, the report should begin with a brief executive summary outlining the assessed and assumed project data, conclusions and recommendations which would afterwards be covered in the body of the study report. The executive summary should concentrate and cover all critical aspects of the study such as the degree of reliability of data in the business environment, project input and output, margin of error, uncertainty and risk, in market-forecast, supply and technological trends. The executive summary should have the same structure as the body of the study report.

2.2 Feasibility Study—A Brief Overview

19

Chapter-II: Project Background and Basic Data • Name and address of project promoter. • Project background. • Objective, strategy, geographical area, market share, cost leadership, differentiation and market niche of the proposed project. • Project location concerning the product market or market of resources (e.g. raw materials). • Economic and industrial policies supporting the proposed project. Chapter-III: Market Analysis and Marketing Concept • Summary of the results of market research—business environment, target market, market segmentation, distribution channels, competition and life cycles. • Annual data on demand and supplies in terms of quantities and prices, prediction of demand and supply. • Explanation and justification of marketing strategies for achieving project objectives. • Outline of marketing concept—projected marketing cost, projected sales programme and revenues. • Impact of marketing on raw materials, supplies, location, environment, production capacity, technology, etc. Chapter IV: Raw Materials and Supplies • Description of the general availability of raw materials, processed components, factory supplies, spare parts and supplies for social and external needs. • List of annual supply requirements. • Availability and possible strategy for critical inputs. Chapter-V: Location, Site and Environment • Description and location and plant site with special reference to ecological and environmental impact, socio-economic policies, incentives and constraints, infrastructural conditions and environment. • Summary of critical aspects and justification of the choice of location and site. • Outline of the significant cost relating to location and site. Chapter-VI: Engineering and Technology • Overview or outline of the production process along with the production capacity. • Description and justification of the technology selected, review of its availability and possible advantages and disadvantages, life cycle, transfer of technology, training, risk control, cost, legal aspects, etc. • Description of the layout and scope of the project. • Summary of main plant items with their availability and cost. • Description of major civil engineering works.

20

2 Pre-project Work

Chapter-VII: Organization and Overhead Cost Description of basic organizational structure and management along with measures required. Chapter-VIII: Human Resources • Description of socio-economic and cultural environment related to project requirements. • Description of human resource availability recruitment and training needs, justification of employing foreign experts, if any and the extent required for the project. • Skills required for the key persons, total number and cost for total employment. Chapter-IX: Implementation Planning and Budgeting • Duration of plant erection and installation. • Duration of project start-up and running-in period. • Identification of critical actions for timely implementation. Chapter-X: Financial Analysis and Investment Appraisal • • • • • • • • • • • • • • • • • • • • •

Summary of criteria governing investment appraisal. Total investment cost. Data for indigenous and foreign investments. Land and site preparation cost. Civil and structural works and their cost. Plant machinery and equipment cost. Utilities, auxiliary and service plant equipment and their cost. Incorporated fixed assets. Pre-production expenditure and capital cost. Net working capital requirements. Total cost of products sold. Operating cost, depreciation charges, marketing costs, finance cost and project financing. Source of finance. Impact of cost of financing and departmental service on a project. Public policy on financing. Investment appraisal—key data. Discounted cash flow and pay-off period. Yield against the total and equity capital investments. If joint venture, the yield for parties. Environmental impact as well as financial or economic implications on the local and national economic scenario. Uncertainty and risks; strategy for risk management; the impact of the projected future scenario on the financial feasibility. National economic evaluation.

2.2 Feasibility Study—A Brief Overview

21

Chapter-X: Conclusions • Major advantage of the project. • Major drawbacks of the project. • Chances of implementation of the project. Cost estimation is the heart of such a report; it is the investor’s concern. There are several methods of estimating cost without detailed engineering. However, for an ongoing project, the preliminary cost estimates should be revalidated from time to time based on the updates of the detailed engineering and procurement status [1].

2.3 Preparation of Bid Documents—A Guideline Once the decision of establishing a plant is taken, enquiries are floated to various parties for basic engineering package, detailed engineering, supply of equipment and instruments and plant construction. Consultants, contractors or vendor would quote their prices based on the technical and commercial specifications in the enquiry or bid document. Now, what is a bid? According to Webster’s dictionary, a bid is ‘to offer (a price) whether for payment or acceptance’. To explain, it is the price required by a particular contractor to do a certain job. Sometimes, it is synonymous with the quotation which is a formal statement setting out the estimated cost for a particular job or service. Again, tender is an offer or proposal made for acceptance, such as an offer of a bid for a contract. Hence, it may be concluded that the tender is the request or invitation for an offer by the purchaser, whereas the bid or offer is the supplier’s proposal and price in reply to such tender. On being notified, many contractors or consultants may purchase the bid document and participate in the bidding procedure, but the one to get the job must offer a price based on the optimum technical and commercial specifications. Differences are there depending upon the work involved, but a basic standard guideline for preparing a bid document (or notice inviting tender, NIT) where the mandatory requirements are given is as follows: Letter Inviting Bid: This is like a covering letter that contains the essence of the instructions, technical and commercial specifications in a nutshell. Some of the essential information that should be included in the letter are as follows: Name of the work. Definitions of various parties and abbreviations. Schedule. Parts of bid, if any—Sometimes bids are requested in two parts: one technical part with commercial conditions without the price and the other is the price part. If a bidder qualifies in the technical bid, then only their price part is opened. So it is to be specified whether the purchaser wants a two- or single-part bid.

22

2 Pre-project Work

Dates of submission, technical query and opening of the bid—There may be some queries from the bidder regarding the bid document before submission of the bid. So a particular date is fixed before the date of submission to clear such queries. Estimated cost and earnest money deposits—A very rough estimate is indicated for the whole work to make the bidders aware of the financial load involved. Earnest money deposit or EMD is a certain amount of deposit to prove that the bidder is serious about the work and eager to do it if the contract is awarded to them. After the finalization of the successful contractor or vendor, the EMD of all the bidders is refunded. Contact person contact details Instruction to the bidders (ITB): This is the guideline following which the bidder would prepare their bids and calculate their prices. The following are some of the mandatory instructions that should be included in the ITB: Eligibility criteria for bidding—It is expected that the bidder has the experience of doing a similar type of work. The bidder should also have sufficient manpower and financial strength to undertake the project. All these contribute to the eligibility of the bidder. As proof of eligibility, a bidder should submit the list of work, bio-data of their key personnel for the project and their tax return certificates. Definitions and abbreviations. Integrity Contract—The bidder is exposed to many technical and financial data of the owner. It is expected that the bidder (and the successful contractor) would maintain confidentiality about such data. For this, an undertaking has to be signed by the bidders. This is the integrity contract. Validity of Bid—The bidder calculates the cost of his work or supply at a particular time. If the finalization of the contract takes much time and there is an escalation in the market, the cost may be increased. Hence, the contractor cannot supply the items or carry out the job at the quoted price. Hence, a bidder should specify up to which time they can do the work without increasing the cost. That period is the validity of the bid. Beyond that point, the offer would not be valid and a fresh cost calculation is necessary. Cost of Bidding—Cost of the bid document and the EMD. Format for Bidders query—Queries from the bidders should be in the same format to enable the owner/purchaser to clarify. Hence a format is required. Exceptions and Deviations—All specifications by the purchaser may not be followed. Sometimes, a bidder can suggest a better alternative. The deviations and exceptions from the bidder’s wish list should be enlisted and declared with proper justification. Bid Evaluation Criteria—Basis or criteria on which bids would be evaluated should clearly be stated so that the bidder can prepare their bids accordingly.

2.3 Preparation of Bid Documents—A Guideline

23

Commercial Terms and Conditions—Certain commercial terms are important for the calculation of price for a job or supply. Some important ones are as follows: Definition of terms. Scope of work. Applicable taxes and charges—It is an important component and affects the price. Terms of payment—Here the breakups of the total payment should be specified. For example, a certain fraction of the total price is generally paid in advance along with the purchase order or contract. After that, different fractions are paid according to the progress of work. These terms or schedule of payment are specified here. Responsibilities and liabilities. Insurance. Applicable government laws and jurisdiction—It should be specified that in case of any legal procedure, under which jurisdiction the case will be sent. For example, the plant is going to be constructed in town A, whereas the head office of the company is in city B. Then it should be specified that the legal procedures would be made in the court of A or B. Penalty and forced Majeure conditions—If the performance is not satisfactory, or delayed, the contractor should be penalized. The details of the penalty should be specified. Similarly, there are some exceptional emergency conditions like natural calamity where the contractor should be exempted from penalty. These are called forced Majeure conditions which should be specified. Disputes—Despite contractual agreements, there may be some disputes during the execution of a project. Possible disputes and the mode of their settlements may be indicated in this segment. Guarantees—Both performance guarantee and bank guarantee are to be clarified here. Price schedule—Item-wise price schedules with breakups for taxes are to be specified here. Schedule—Various milestones to be completed are to be indicated here preferably in the form of a bar chart. Termination of contract—There are conditions when the owner/purchaser may decide to terminate the contract. Those conditions are to be specified. Technical Specifications—These are the most important information based on which the overall project will be worked out. Cost calculation will be influenced by the design, and design will be based on the technical specification. Some of the important technical information are: Definition of battery limit—A Battery Limit is the physical demarcation of boundaries indicating the scope of works for a particular unit, facility, system as well as for a particular contractor or subcontractor. For costing, it is very important to know the scope of work, which in turn depends on the battery limit. Scope of work for various parties involved—As described earlier, there are many parties involved in a project. A sketchy scope of work for all the parties with detailed

24

2 Pre-project Work

scope for the particular work should be included to understand the interfaces with the other parties. Battery limit conditions for process streams—Physical and chemical conditions for the inlet (feeds, catalysts) and outlet (product/by-product) streams should be specified at the battery limit. This would be required for the process flow diagram which is the technical basis of the project. Battery limit conditions for utilities—Like the process streams, battery limit conditions for the utilities like steam, cooling water, process water, instrument air, etc. are to be specified for plant and equipment design. Processing objectives and process description with tentative flow diagrams and plot plans should be furnished. Capacity and turndown ratio—The rate of production is an important parameter whereby the magnitude of the plant is determined. Turndown ratio is the range of the operational capacity of a plant and can be determined by the maximum is to the minimum capacities. Specification of feed and desired product—These would help the design process. Code, standard and design basis—There are different codes and standards for materials and designs based on which the specification (and thereby the cost) changes. A particular purchaser may opt for a particular code and standard. That should be specified since the cost may vary to comply with a particular standard. This will be described in more detail in the next section. Specific design guidelines—There are different bases of design that influence the size or specification of equipment. For example, one designer may allow a safety factor of 1.3 and the other may allow 1.2 for the thickness of the shell of particular equipment. The purchaser/owner should specify specific design guidelines if any. Safety and environmental aspects—Safety and environmental norms to be complied with should be specified. Tolerance limit for the effluents to be discharged—Most of the chemical plants discharge solid, liquid and gaseous waste streams into the environment after treatment. The maximum limits of various pollutants that can be discharged as per local pollution control authorities should be specified. Evaluation criteria for technical bid—There are certain key points based on which the technical bids are evaluated. Weightages on each of such key points are to be specified to enable the bidder to prepare their bids properly.

2.4 Codes and Standards A code is a set of rules or models that are recommended by experts in the respective fields. Codes describe the ideal case or situation and hence a code can be adopted as law. On the other hand, a standard is the definition, explanation and a set of guidelines for achieving the ideal situation described in the code. Both code and standard ensure uniform quality of a product or service irrespective of the manufacturer or provider. The cost of the product or service is also dependent

2.4 Codes and Standards

25

on the code and standard. The more stringent is the standard, the more the cost will be. Standards are usually published by companies, professional bodies, organizations or countries. When the standards are adopted by government agencies, they become codes. Examples of code may be International Building Code and ASME Boiler and Vessel Code. Standards may be Indian Standard by BIS or British Standard (BS) and so on. American Society for Mechanical Engineers (ASME), Bureau of Indian Standard (BIS), American National Standard Institute (ANSI), American Petroleum Institute (API), British Standards (BS), German Institute for Standardization (DIN), International Organization for Standardization (ISO) and Institute of Electrical and Electronics Engineers (IEEE) are a few familiar organizations or associations for stipulating codes and standards [2].

2.5 Site Selection Though site selection is a part of the feasibility report, it has immense importance for a chemical process plant and therefore is explained separately. The location influences the profitability and provision for future expansion. The principal factors to consider are as follows.

2.5.1 Availability of Raw Materials Price and availability of raw materials are one of the important factors that influence the selection of a site location. For example, plants for bulk chemicals should be located near to the source of its major raw material since the raw material requirement is large and if its source is far from the plant, a huge shipping cost for raw materials should be borne; this would affect the cost of production.

2.5.2 Location Concerning the Market Location of the raw materials and product markets are determining factors for the location of a plant site. If a plant produces a product having high volume and low price (for example, cement or fertilizers), then carrying the product to market is not preferred, hence such plants should be located near the primary selling market. In contrast, products of low volume and high price, such as pharmaceuticals, can be carried to a faraway market. Again, for refinery, which has too many products to carry

26

2 Pre-project Work

individually to the primary market, it is convenient to locate it near the consumer market—whereas crude oil can be transported more conveniently.

2.5.3 Transportation Facilities Raw materials and finished products should be transported from a plant. During construction, equipment should be transported to the site as well as engineers and management staff should have to travel to the site. Hence, a site should be easily accessible by at least two of the major transport facilities among rail, road and air. Water transport within India is less common, but sometimes it is also considered. The proximity of highway, railroad, seaport and airport is a major consideration for site selection.

2.5.4 Availability of Workforce and Labours Though experts and engineers may come from outside, unskilled labourers are generally hired locally. Semi-skilled labourers suitable for further training should also be available near the site. Electricians, fitters and welders are needed not only during construction but also for maintenance of an operating plant. So it should be considered whether an adequate pool of labourers is available near the site. Local labour laws and trade union customs should also be considered.

2.5.5 Availability of Utilities—Water, Fuel and Power A huge quantity of water is required in most of the chemical plants for cooling, washing and steam generation. Fuel is required for boilers or furnaces. So the source of water and supply of fuel are two major factors in the choice of a plant site. Raw water for various uses may be drawn from a river or wells, whereas treated water can be purchased from a local authority. If a large quantity of solid or liquid fuel is required for the plant, the location of the plant should be near the source of supply of the fuel on the economical ground. Whether the plant will purchase grid power or generate power will be determined by the local cost of power. A competitively priced fuel must be available on-site for steam and power generation.

2.5 Site Selection

27

2.5.6 Climatic Conditions Cost of a plant may be increased by adverse climatic conditions. For example, if the ambient temperature of a plant is very low, additional heating, steam tracing or insulation may be required. Similarly in a hard seismic zone or high winds structures should be made stronger. A corrosive environment may need special protective paint on the pipeline or equipment.

2.5.7 Environmental Impacts Including Waste Disposal Facilities Results of the Environmental Impact Assessment (EIA) is a very important parameter for selecting a site location. Especially if there is a reserve forest or heritage monument around, those can be affected by the construction and operation of the plant. Locally available waste disposal facilities should also be considered. This includes sewage canals and solid waste management facilities. Hazardous wastes to be generated from the plant should be treated by an authorized facility.

2.5.8 Site Characteristics Concerning Potential Hazards If the area is prone to earthquake, volcanic eruption or forest fire, the place should be either avoided or special care should be taken.

2.5.9 Taxation and Legal Aspects Law and tax rules differ from country to country, even from region to region. A site should be selected based on the most convenient tax rules and legal aspects.

2.5.10 The Political Situation and Local Community Consideration Some places are inherently unstable in terms of political situations. That may affect construction and supply and may also increase the project cost. Similarly, the local community sometimes becomes hostile about a new plant in their locality. If the plant

28

2 Pre-project Work

site displaces a section of residents, proper compensation should be provided; that will contribute to the capital investment. All such factors should be considered.

2.6 Environmental Impact Assessment (EIA) Environmental Impact Assessment is the systematic examination and analysis of unintended but likely consequences of a proposed project or programme, with the view to reduce or mitigate negative impacts and maximize on positive ones. More detailed deliberations can be obtained in the books specifically covering environmental impacts. Here a very sketchy introduction of the assessment is presented. The various steps for an EIA are as follows: • To identify all environmental factors in the project area which will be influenced by all the activities of the proposed project under consideration. For this, the environmental conditions and parameters should be recorded before starting the project (baseline information) and the change in the environmental conditions after the proposed plant should be forecast using suitable environmental models and tools. • To identify alternative approaches including no action plan, their levels of impacts and economic considerations. • To identify the tools and methodologies to be adopted in the EIA. • To describe the environmental setting, to collect data and the factors associated with the proposed project. The collection of data will be influenced by the method chosen. • To evaluate control measures for the unwanted impacts. • To encourage public participation in evaluating the impact of the proposed project and to provide information. The framework of EIA may be summarized as follows: • Studying the effects of a proposed action on the environment. • Comparing various alternatives where the best combination or economic and environmental cost and benefits can be obtained. • Predicting the change of the environment that would result from the proposed action. • Selecting and deciding the optimum action among the alternatives. The essence of all the methodologies for EIA lies in impact identification, measurement and interpretation. Once the information for impact is generated, it should be passed on to the user. The various methodologies for an EIA study are described very briefly as follows: Ad Hoc Method—Ad hoc methods are for a rough assessment of total impact giving the broad areas and general nature of possible impacts. It does not define specific

2.6 Environmental Impact Assessment (EIA)

29

parameters to study and therefore cannot provide effective guidance for the assessment or remediation. Basically, it is a planning tool and a range of pollution or economic effects are estimated. Map Overlays Method—This method is particularly useful for a preliminary site selection among various alternative probable sites in a particular region. Deciding the highway route and corridors are mainly facilitated by such a method. A particular region may have different maps—political, natural, social, etc. In the map overlay method, several maps printed on transparencies are overlaid to identify a suitable site where the interference of the proposed project is the least with the existing environment. Previously transparent maps were used; nowadays, it is possible digitally with the help of software and geographical information system (GIS). The main disadvantage of such a method is it needs sophisticated skills and skilled personnel as well as cost for accurate data. Checklist Method—Checklists are a comprehensive list of the various environmental impacts and their indicators that are used to describe the possible consequences of a proposed project. There are four types of checklists—Simple, Descriptive, Questionnaire, Threshold concern and Scaling Checklist. A simple checklist just indicates the environmental parameters with no guidance for assessment or interpretation. Other checklists are comparatively detailed. But the checklist method as a whole is too general and cannot indicate interactions between the impacts themselves. Network Method—A project may have many subsequent or secondary environmental impacts in addition to/as a result of the various primary impacts. Again, the primary impact was caused by a particular sub-action of the total project. A network method illustrates or traces all such higher order impacts. Hence, this method indicates remediation measures also. The main disadvantage is its complexity and over-specification of the higher order impacts making the assessment too pessimistic. For investigating higher order environmental impacts, the Sorenson network method may be considered as one of the best choices. It identifies feasible mitigation measures. The structure/content of the network must be predefined for a particular EIA. However, its application is effective only adequate data are available and if the reference network is relevant to the local environment. Method of Matrix—Matrices are two-dimensional tables that demonstrate the impacts due to interactions between activities and environmental components. Qualitative or quantitative estimates of the impacts can be entered into the cells of the matrix. There may be five types of matrices—Simple Matrix, Time-dependent matrix, Magnitude Matrix, Quantified Matrix (Leopold Matrix) and Weighted Matrix. The matrix method quantifies the impact. Leopold matrix is the most familiar quantitative matrix. Combination (Computer-Aided) Method—This method combines network and matrix methods along with standard government data and information systems. Eventually, it is based on software and computer networks. Instead of a numerical system,

30

2 Pre-project Work

potential impacts are identified on a need-to-consider scale, using A, B and C as indicators [3]. Formats for EIA report may vary for different consultants, but some of the very essential information is tabulated as follows: • • • • • • • • • • • • • • • • • • • •

Executive Summary, Introduction, Project Description, Project Cost and Implementation Details, Process Description, Raw Material Details, Description of Environment, Baseline Environmental Conditions, Ambient Air Quality, Noise Level, Water Quality, Soil Quality, Biodiversity, Baseline socio-economic conditions, Anticipated Environmental Impact and Management Plan, Construction Phase, Operation Phase, Environmental Monitoring Programme, Risk Analysis and Conclusion and Final Recommendation.

2.7 Optimization in Design Preliminary design of the plant and the equipment is done during pre-project work for costing purposes and the designs are further refined during basic engineering. Finally, a detailed Engineering contractor freezes the design. Designing a process plant is nothing but finding out the optimum solution to a problem. The optimization can be carried out mathematically, using various software tools or based on the experience and judgement of the design engineer. Most of the optimization problems have many sub-problems to optimize; it is interesting that optimal solutions to each sub-problem may not lead to the overall optimal solution. In engineering process design, a major criterion for optimality generally is reduced to consideration of cost or profit. The economic performance of a design is influenced by several factors—type of processing techniques, equipment and their arrangement as well as the physical parameters and operating conditions. In most cases, major considerations of optimization are cost and economic criteria, but sometimes, factors other than cost can determine optimum conditions.

2.7 Optimization in Design

31

2.7.1 Trade-Offs Enhancement of the performance of a plant is generally achieved at the expense of increased fixed capital or operating cost. It should be decided whether the extra cost should at all be incurred or not. Sometimes, two costs compete and any one of them should be selected. For example, if process heat is to be recovered to save the cost of energy, the cost of heat exchanger surface should be borne—here a trade-off between the two costs should be made. In places where energy is costly, the heat energy recovery must be done; however, where energy is not very costly and cost of equipment is high, one may opt for saving the capital cost of equipment rather than cost of energy [4]. Some common trade-offs encountered in the design of chemical plants include. • • • •

More separation equipment and operating cost versus lower product purity. More recycle cost versus increased feed and waste. More heat recovery versus cheaper heat exchanger network. Higher reactivity at high pressure versus more expensive reactor reactors and higher compression cost. • Fast reactions at high temperature versus product degradation. • Marketable by-products versus more plant expenses. • Cheaper steam and electricity versus more offsite capital cost.

2.7.2 Objective Function and Constraints An optimization problem generally involves the maximization or minimization of a variable quantity called ‘objective’ and the expression of that quantity in terms of the influencing parameters is called ‘objective function’ [5]. For chemical engineering designs, the objective should be a measure of how effectively and economically a design meets the customer’s need. A few typical design optimization objectives are as follows (Table 2.1). The first step in formulating the optimization problem is to state the objective as a function of a finite set of variables, sometimes referred to as the decision variables, which may be independent but are usually related to each other by many constrained equations. Table 2.1 Examples of maximization and minimization problems in the design

Maximization

Minimization

Net present value of the project

Project expenditure

Return on investment (ROI)

Cost of production

Reactor production capacity per unit volume

Total annual cost

Yield of the main product

Waste generation

32

2 Pre-project Work

The optimization problem can therefore be considered as maximization or minimization of the objective function subject to a set of constraints or boundary conditions. An additional issue during formulating the objective function is the quantitative estimation of uncertainty. Economic objective functions are generally highly sensitive to the price estimates used for feeds, raw materials and energy and also to the estimates of project capital cost. These costs/estimated costs generally have substantial errors. Sometimes, the design engineer uses other objectives than the original problem and works on the sub-components of the design. However, optimum solutions for the sub-units may not always result in the optimum solution of the whole process—the optimization process is not additive. The purpose of constraint is to limit the parameter space. Equality constraints are from the ‘balanced’ equation and inequality constraints are from the ‘limit’ equation.

2.7.3 Degree of Freedom If the problem has n variables and me equality constraints, then it has degrees of freedom of (n − me ): If n = me , If me > n,

No degree of freedom. n variables can be determined by solving me equations. The problem is over-specified.

Generally, me < n and n − me is the number of adjustable independent parameters for finding optima.

2.7.4 Mathematical Programming Techniques for Optimization There are no generalized systematic programming practices that can be applied to all designs. Again, mathematical methods are out of the scope of this book. However, the commonly applied processes are just pointed out here before going to the application of optimization in design. Many programming methods have been developed for the various kinds of optimization situations most commonly encountered. The programmes are classified according to several criteria: According to the range of mathematical relations. Linear integer programming (LIP)—all variables should be an integer. Mixed Integer Linear Programming (MLIP). According to the need for constraints.

2.7 Optimization in Design

33

Stochastic programming—useful for dealing with uncertain processes or evaluating process flexibility. Less applicable for design optimization. Bound constrained programming—these are situations where some variables or relations are artificially bound. Linear programming. Non-linear programming. Linear Programming may be done by Analytical method. Graphical method.

2.7.5 Optimization of a Single-Variable Problem-Problem Statement Minimize (or maximize) Subject to

z = f (x)

x ≥ xL x ≤ xU

(2.1) (2.2) (2.3)

Analytical Method: 1. 2. 3.

= 0 at minima (or maxima). So z should be differentiated with respect to x and the value should be made equal to 0 to find out x’s. 2 At all values of x, dd xz2 should be evaluated. When f  (z) is more than 0, that point is the minima whereas if f  (z) is less than 0, it reaches the maxima. The function z should be evaluated at all x’s, x L and x U . dz dx

Graphical Method: The function z should be evaluated at different arbitrary points between x L and x U and a plot of z versus x should be made. The lowest (or highest) point should be located on the plot. That is the value of the function z at minima (or maxima) [5].

2.7.6 Application of Optimization in Process Design I.

Optimum Production Rate

A very important variable for the operation of a plant is its rate of production or production per unit time. In fact, the plant is sized based on this production rate.

34

2 Pre-project Work

Production rate depends on many factors such as the number of hours in operation per day, week month or year; the load on the equipment and market demand. From an analysis of the costs involved under different situations along with other factors influencing the particular plant, it is possible to determine the optimum rate of production or economic lot size. During the determination of the optimum production rate, it may be observed that the optimum production rate can be determined with respect to the minimum cost of production as well as with respect to the maximum profit. The total production cost per unit time may be divided into two components— operating cost and organizational cost. Operating costs are generally functions of the quantity or rate of materials produced and include expenses like labour, raw materials, power, heat, supplies and similar items. Organizational costs are generally independent of the quantity or rate of production—this includes expenses for directive personnel, physical equipment and other services and facilities that must be maintained irrespective of the quantity of production. For convenience, operating costs are generally considered based on one unit of production. Then this cost can be divided into two components • minimum expenses for raw materials, labour, power, etc. that remain constant and must be paid for each unit of product as long as any material is produced. • extra expenses due to an increase in the rate of production (super-production cost). These costs include extra expenses caused by an overload of power facilities, additional labour requirements or decreased efficiency of conversion. Super-production rate can be represented as m Prn Where Pr is the rate of production units/time; m and n are constants. Now let the constant part of the operating cost be h, organizational cost per unit time be OC ; then the cost of product per unit is x T . So, x T = h + m Prn +

OC Pr

(2.4)

So total product cost per unit time for Pr quantity/units of product is C T = x T × Pr = (h + m Prn +

OC )Pr Pr

(2.5)

Again, let the profit per unit product be yr and s be the selling price. Then,  yr = s − x T = s − h +

m Prn

OC + Pr



If R is the profit per unit time (that is for Pr units of production), then

(2.6)

2.7 Optimization in Design

35



R = yr × Pr = (s − h − m Prn −

OC )Pr Pr

(2.7)

Now it is pertinent to know the rate of production for which • the cost of production per unit product is the minimum Or • the profit per unit time (that is for Pr quantity of production) is the maximum. For the first case, let us differentiate x T in Eq. (2.4) with respect to Pr . d xT should be equal to 0 at minima. Solving for Pr , we get d Pr 1  OC  n+1 Pr = m×n this is the optimum production rate for which cost of per unit production is minimum. For the second case, we should differentiate R (Eq. 2.7) with respect to Pr and put the derivative equal to 0 to get  Pr =

s−h m(n + 1)

 n1 (2.8)

So, this is the optimum production rate for which profit per unit time is the maximum. Now it is the choice of the owner to decide which production rate they would set up the plant for. II.

Economic Pipe Diameter

Investment for piping and pipe fittings is a significant portion of the total investment in a chemical plant. For any given set of flow conditions, the use of increased pipe diameter results in higher fixed cost and lower pumping cost due to less frictional resistance inside the pipe. Hence, an optimum or economic pipe diameter has to be determined so that the total cost is minimum (Fig. 2.1). For flow of incompressible fluid through a pipe of constant diameter, the mechanical work added to the system, W s , is Ws =

2 f v2 L(1 + J ) +B Di

(2.9)

where f = Fanning’s friction factor, v = average linear velocity, L = length of the pipe, J = frictional loss due to fittings expressed as a fraction of loss in a straight pipe, Di = inside diameter of the pipe and B = constant that takes care of all other factors of mechanical energy balance.

36

2 Pre-project Work

Fig. 2.1 Economic pipe diameter

Since the turbulent flow is the most common in a chemical plant, economic pipe diameter in the turbulent flow only is discussed here. For turbulent flow with Reynolds number Re ≥ 2100, Fanning’s friction factor can be estimated by f =

0.04 Re0.16

(2.10)

Combining this to the mechanical work equation, with a suitable conversion factor, cost of pumping, C pump , is C pump =

0.84 0.16 μc × K (1 + J )Hy 1.248 × 10−4 q 2.84 f ρ

Di4.84 E

+ B

(2.11)

where C pump = pumping cost in USD per year per m pipe length, qf = flow rate in m3 /s, ρ = fluid density in kg/m3 , μC = fluid viscosity in Pa s, K = cost of electrical energy required for pumping USD/KWh, E = efficiency of pump and motor expressed in fraction, H y = hours of operation per year and B’ = Constant independent of Di . Cost of pipe and fittings (C pipe ) has two components—cost of new pipelines including piping materials and installation charges and cost of maintenance of the installed pipeline. For most pipes, purchase cost for pipes is

2.7 Optimization in Design

37

 C pp = X

Di 0.0254

n (2.12)

and annual cost for the installed piping system is  Cpipe = (1 + F)X

Di 0.0254

n KF

(2.13)

where Cpipe = Cost of installed piping system in USD per year per m of pipe length, F = Ratio of the total cost for fittings and installation to the purchase cost of new pipes, X = Purchase cost of new pipe per m of pipe length in USD per m, if the diameter is 0.0254 m (1 inch pipe) and K F = Annual fixed charges including maintenance expressed as a fraction of the initial cost for the completely installed pipe. Now the total annual cost for the piping system can be obtained by adding pumping cost (Eq. 2.11) and fixed cost (Eq. 2.13). Taking a derivative of the total annual cost with respect to the pipe diameter Di and setting it equal to 0 gives the economic pipe diameter. For turbulent flow, economic pipe diameter depends on qf , ρ and μC as well as on the cost of pipe, cost of its installation and pump efficiency.  Di,opt =

0.16 × K (1 + J ) × H 6.04 × 10−4 × (0.0254)n × q 2.84 × ρ 0.84 × μC y f n(1 + F) × E × K F

1  (4.84+n)

(2.14) where for steel pipes, n = 1.5 if Di ≥ 0.0254 m and = 1.0 if Di > 0.0254 m. Putting the above values for n, we get for turbulent flow in steel pipes,  Di,opt =

q 0.448 f

×ρ

0.132

×

μC0.025

1.63 × 10−6 × K (1 + J ) × Hy (1 + F)X × E × K F

for Di ≥ 0.0254 m and

0.158 (2.15)

38

2 Pre-project Work

 Di,opt = q 0.487 × ρ 0.144 × μC0.027 f

1.53 × 10−5 × K (1 + J ) × Hy (1 + F)X × E × K F

0.171 (2.16)

for Di < 0.0254 m. However, it is also observed that the economic pipe diameter is less sensitive to most of the quantities because of the small values of the exponents. If the standard values of the operating parameters like J, Hy, X or K F are known, the expressions for economic pipe diameter can be further simplified. The estimates are sufficiently accurate and generally on the safe side for design purposes. Again, the expression for the economic pipe diameter can be further refined considering the effects of taxes and the cost of capital or return on investment. More accurate expressions for frictional loss due to pipe fittings can also be used. But all these are possible at a later stage of the detailed design. III.

Cooling Water Flow Rate in a Condenser

Let us consider a condenser with water as the cooling medium to carry out a certain duty. If the change in water temperature is small, the flow rate of water is increased and if the change in water temperature is large, the flow rate of water can be kept low. Both of the options have their consequences. The use of more water would reduce the heat transfer area, thereby the initial fixed cost of the heat exchanger is reduced. But this high water flow rate asks for the treatment and recirculation cost (operating cost) for water. An economic balance between the costs of high water flow rate—small surface area and low water flow rate-large surface area—determines the optimum flow rate of cooling water to the condenser. Now a general case is considered in which heat should be removed from a condensing vapour at a given rate of q˙ kJ/s. The vapour condenses at a constant temperature T cond °C; let the cooling water inlet temperature be T 1 °C. If m˙ is the flow rate of cooling water in kg/s, C p is the heat capacity and T 2 °C is the outlet temperature of cooling water from the condenser, then q˙ = mC ˙ p (T2 − T1 ) = U ATL M =

U A(T2 − T1 )

cond −T1 ) ln (T (Tcond −T2 )

(2.17)

Here, U = Overall heat transfer coefficient and A = Heat transfer area. Again m˙ =

q˙ C p (T2 − T1 )

(2.18)

2.7 Optimization in Design

39

Design considerations set q˙ and T 1. The heat capacity of water is generally 4.2 kJ/kg K. So the flow rate of water m˙ is fixed if T 2 is fixed. Now let us see how to determine the optimum T 2 . The annual cost of cooling water is given by m˙ Hy Ccw =

q˙ Hy Ccw C p (T2 − T1 )

(2.19)

where Hy = operating hours per year and Ccw = Cost of cooling water per unit weight. Annual fixed charges for the condenser are AK F CC A where CC A = Installed cost of the condenser per square meter of the heat transfer area and K F = annual fixed charges including maintenance expressed as a fraction of the installed cost of the condenser. Assuming all other costs to be constant, the total annual operating cost for the condenser is q˙ Hy Ccw + AK F CC A C p (T2 − T1 )

(2.20)

Substituting A from the first equation, we get the annual operating cost as follows:

CC O



cond −T1 ) q˙ K F CC A ln (T (Tcond −T2 ) q˙ Hy Ccw + = C p (T2 − T1 ) U (T2 − T1 )

(2.21)

Differentiating CC O with respect to T 2 and setting the derivative = 0, we get (Tcond − T2,opt ) U Hy Ccw (Tcond − T1 ) − 1 + ln = (T cond − T2,opt ) (Tcond − T1 ) K F CC A C p

(2.22)

With other quantities known, T 2,opt can be found out from the above equation. Generally, a trial-and-error or computer programming is required to solve for T 2,opt ; however, a semi-logarithmic plot below (Fig. 2.2) may also be used. IV.

Optimum Conditions in Cyclic Operation

Many processes are carried out via cyclic operations, which involve periodic shutdowns for discharging, cleanout or reactivation. This type of operation occurs when the product is produced by a batch process or when the rate of production decreases with time, as in the operation of a plate and frame filtration unit. In a true batch process/operation, no product is obtained until the unit is shut down for discharging. In semi-continuous cyclic operations, the product is delivered

40

2 Pre-project Work

Fig. 2.2 Plot for optimum outlet temperature of cooling water

continuously while the unit is in operation, but the rate of delivery decreases with time. Analysis of cyclic operations can be carried out conveniently by using the time for one cycle as a basis. The following type of relationship can be developed: Total annual cost = x cycles per year × {(Operating cost + shutdown cost)/cycle} + annual fixed cost. Annual production rate = (total production/cycle) × (no. of cycles/year). Cycle/year = {(Operating + shutdown time)/year}/{(operating + shutdown time)/cycle}. or, {(Operating + shutdown cost)/year}/{(operating + shutdown cost)/cycle} [6].

2.8 Scaling Up and Scaling Down The issue of scale-up or scale-down is often encountered in process plant design. The technology available and tested for a different capacity may sometimes have to be scaled up or down according to the requirement. Many processes are successful in a laboratory or bench scale, but only a few of them become industrially feasible. Scaling up needs the skill of chemical engineering rather than, or in addition to, the knowledge of chemistry [7].

2.8 Scaling Up and Scaling Down

41

Scaling up is often required for achieving one or more of the following objectives: • • • •

Introduction of new process. Growth in the demand in the market. Reduction in making expensive errors in a new design and its operation. Attaining economic feasibility.

Compared to scaling up, scaling down is rather uncommon. It is generally done to characterize a process in detail which is not possible on a manufacturing scale. One unit operation can be studied at a time. Bioreactors and chromatographic columns are the best examples of a scale-down approach. In chemical projects, scaling up is the most critical issue [8]. In nature, the possible range of size of animals is determined by the factors like the ability of an animal to support its own weight, the adequacy of blood circulation, metabolic rate and change in body temperature. Data compiled on animals of various sizes show that the metabolic rate is a major factor limiting the size of the animal. With the increase in size, the weight is increased more than the strength to bear the weight. All these observations are applicable for scaling up a process plant as well. Rate of reaction, fluid flow and increase in temperature are the main factors to consider for scale-up. Mechanical strength of large equipment or robustness of the process is also important issues [9]. Scaling up generally proceeds through the following stages: • Bench scale or laboratory scale—When a new product or technology is invented in the laboratory, its study is an early stage tool for a scale-up process. Typical capacity is 0.001–0.1 kg/h. • Pilot scale—It is the first view of the continuous processing generated out of the bench-scale process which is generally a batch one. Typical capacity is 10– 100 kg/h. • Demonstration scale—Here, the process flow sheet closely resembles the commercial operation on a reduced scale. The typical capacity is 100–1000 kg/h. • Commercial scale (typical capacity > 1000 kg/h) is preceded by a demonstration scale. There are three categories of scale-up: • Well-defined, easy and quantifiable (examples are distillation process, heat exchangers and absorption process). • Difficult, but quantifiable (an example is a reactor). • Very difficult and rarely quantifiable (examples are particulate processes). Common problems of scale-up are: • Extended reaction time that is, a low rate of reaction—leading to poor productivity. • Interacting operational problems—one leading to another. For example, excessive swelling of reactants leading to the ejection of the reaction mixture from the reactor.

42

2 Pre-project Work

• Poor selectivity and low yield. These problems are generally caused by the interaction of the chemistry of the process with the physical variables of the same. It should be kept in mind that chemical rate constants are scale-independent but physical parameters are not. Factors that most commonly interact with the chemistry of the process and cause poor performance of the scaled-up process may include processing time, heat transfer, power input, gas desorption, phase separation, mixing time and mass transfer rates in the two-phase system. The most common reason for poor selectivity on scale-up is the failure to assess the stability of products and reactants over the exposure time likely in a commercial scale manufacturing condition. Reactants and products may have to be exposed to the experimental conditions for longer periods during the higher scale manufacturing process compared to the laboratory scale. Another issue lies in the scale-up of heat transfer. Scaling up by a certain factor in a jacketed reactor results in a reduction of the surface area to volume ratio which means that heating and cooling via jacket is much slower on the full scale. Hence, the temperature difference between the vessel wall and contents should be increased, but this may result in overheating of the reaction mixture near the wall especially for viscous fluids causing degeneration. One more risk is an exothermic reaction, delayed exotherms and runaway reaction. There are several general aspects of scale-up: Economical aspects: When considered financially, it looks more attractive to build one big piece of equipment rather than multiple small parallel units. Generally, the investment cost (I C ) increases with the production capacity (PC ) according to the 0.6th rule. That is IC ∝ (P C )0.6

(2.23)

It follows from IC ∝ ar ea(L 2 ) and PC ∝ volume(L 3 ). The exponent 0.6 is called the Degression coefficient and changes from case to case. It depends on the type of equipment and the definition of ‘standard size’. As long as this Degression Coefficient is less than 1.0, increasing size is preferred compared to increase in number. Scaling up by number and by size can be compared as follows (Table 2.2). It is generally suggested that scaling up should be done first by size till the degression coefficient reaches 1.0, then by number. • Aspects of energy, raw materials and environment. • Vulnerability of production. • Logistics – Transportation of parts. – Supply of raw materials. – Discharge of products.

2.8 Scaling Up and Scaling Down Table 2.2 Comparing scaling up by number and by size

43

Scale-up by number

Scale-up by size

• The design process is fast

• The design process is slow

• Construction is easy

• Construction is difficult

• Less operational problem

• More operational problem

• Repair and maintenance is easy

• Repair and maintenance is difficult

• Logistics problem for distributed system

• Logistics problem for large size

• Problem of interference

• Risk of bulk handling

• Social aspects – Employability. – Labour conditions. • Mechanical and other technical considerations. • Start-up, management and maintenance considerations. • Safety and hazard issues.

2.8.1 Scaling Factors Let two different scales be considered—model scale (M-scale) and prototype scale (P-scale) along with one physical quantity X. A scaling factor can be defined as k X = XX MP . For example, for a cylindrical reaction vessel, scaling factors of the height and diameter can be given as k H = HHMP and k D = DDMP . Three kinds of physical quantities require special attention: • Dimension (length)—One method of scaling is geometrical similar scaling. All dimensions change by the same factor. If the scaling factor of length is kL , then for all areas, the scaling factor for area kA should be kL 2 and the scaling factor of volume kV should be kL 3 . • Material properties—Properties of materials like densities or specific heat capacities do not change with scale, hence for identical materials, those scaling factors are 1.0. • Physical constants like gas constants also do not change with scale, hence those scaling factors are by definition, 1.0. Scale-up is designing using a ‘foreknowledge’. Most often this foreknowledge is a result of laboratory experiments or experience in a similar industrial process at a different scale. At the M-scale, information is available that we want to make use of at the Pscale. This is possible only if there are certain analogies between the two processes at M-scale and P-scale. This analogy is called similarity.

44

2 Pre-project Work

Fig. 2.3 Levels of process similarity

There are two types of similarity: • Equipment or apparatus similarity where dimensions of the equipment are being scaled up with a common scale-up factor. • Process similarity where certain process variables are scaled up from process considerations (e.g. rate of production or energy consumption). In such cases, process requirement at process similarity is the primary consideration rather than the dimensional scale-up. In the case of process similarity, ‘similarity’ between model and prototype means that deformation, flow, temperature and concentration profiles of model and prototype are congruent in dimensionless form. Various levels of process similarity are given in Fig. 2.3. For scaling up mixers, three types of similarities may be considered—geometric, kinematic and dynamic. A few points about similarity-based scale-up may be summarized as follows: • At each level of process similarity, it is possible to indicate the conditions by using equal numbers of dimensionless numbers at both P- and M-scales. • Not all dimensionless numbers are important at all scales. Regime-analysis is the method whereby it is determined which dimensionless group is important—which should be kept constant and which may be allowed to vary. The groups which have to be kept constant in both scales form the scale-up criterion together. • For a gas–liquid reactor, the same mass transfer coefficients should be maintained in the equipment of the two scales. That is, gas phase film coefficient, liquid phase physical coefficient and reactive coefficient should be the same for the two

2.8 Scaling Up and Scaling Down

45

reactors. Also, there should be the same ratio of the specific interfacial area and liquid hold-up. • For scaling up of a fermenter or similar aerobic process reactors, the dissolved oxygen (DO) level should be determined for the smaller equipment. Conditions of the larger equipment should be adjusted to have the same DO level. Both the reactors should have the equivalent oxygen transfer coefficient. Other factors are shear, mixing time and Reynold’s number.

2.9 Safety and Hazard Issues Before starting a project, one should be aware of the potential hazards of the process. All manufacturing processes are hazardous to some extent but in chemical processes, there are additional special hazards associated with the toxic or corrosive chemicals used and process conditions like high temperature, high or low pressure. Hence, a chemical engineer should keep a strict vigil to the safety issues and loss prevention throughout the life cycle of the project—from its inception to the last day of production. According to the type of safety, processes are classified into intrinsically safe and extrinsically safe (where safety features are engineered) ones. An intrinsically safe process does not cause (or very little) danger under all possible deviations from the designed operating conditions. In reality, all chemical processes are dangerous or unsafe in some form or other when the process conditions deviate from the design conditions. However, a designer should always select an intrinsically safe process as far as practicable and economic. The term ‘Engineered Safety’ covers the provision in the design of control system, alarm trips, pressure relief devices, automatic shutdown systems, duplication of key equipment services, fire-fighting equipment, sprinkler system and blast walls to contain any fire or explosion. Hence, some additions and alterations of the process design become necessary not for the functionality of the process or design but for the safety. Miscellaneous design modifications for safety will be discussed under basic and detailed design. Here, let us become aware of the potential hazards of a chemical process plant. Generally, the following points are considered as hazards in chemical plants [10]: • • • • • •

Toxicity, Flammability, fire and explosion, Ionizing radiation, High pressure and vacuum, Temperature deviation and Noise. Let us explain the characteristics of such hazards briefly.

46

2 Pre-project Work

2.9.1 Toxicity Most of the products manufactured in a chemical plant need chemicals as raw materials—most of the chemicals being harmful to living organisms. The degree of hazard will depend on the inherent toxicity of the material—and also on the extent of exposure. Both toxicity and duration matter together; short exposure to a very toxic material may be less dangerous than long exposure to a less toxic material. The difference between short-term (acute) and long-term (chronic) toxicity and the same between safety hazard and industrial health and hygiene hazard may also be noted. A highly toxic material that causes immediate injury (such as phosgene or chlorine) would be classified as a safety hazard. A material whose effect is only apparent after long exposure at low concentrations (such as vinyl chloride) would be classified as industrial health and hygiene hazard. The permissible limits and the precautions to be taken care of to ensure the safe limits are very difficult for these two classes of toxic materials. The inherent toxicity of a material is measured by tests on animals. It is usually expressed as the lethal dose at which 50% of the test animals are killed, LD50 (lethal dose fifty) value. The dose is expressed as the quantity in mg of the toxic substance per kg of the bodyweight of the test animal. There is no generally accepted definition of what can be considered toxic and non-toxic. World Health Organization (WHO) classifies toxicity into four categories as below. The system is based on LD50 determination in rats, by oral solid agent ingestion. Class I a: extremely hazardous

LD50 at 5 mg or less/kg bodyweight

Class I b: highly hazardous

5–50 mg/kg

Class II: moderately hazardous

50–2000 mg/kg

Class III: slightly hazardous

2000 mg/kg

However, these values of permissible limits may differ for liquid oral agents and dermal agents. It may be noted that the definitions are applicable only to short-term (acute) effects. As mentioned, for long-term exposure, duration or frequency should also be considered as well to decide the permissible limits. For this, the basis is generally the threshold limit value (TLV). The TLV is defined as the concentration to which it is believed the average worker could be exposed to, day by day, for 8 h a day, 5 days a week, without suffering harm. It is expressed in ppm for vapour and gases and in mg/m3 for dust and mists. The set of regulations in force is ‘Control of Substances Hazardous to Health Regulations (COSHH)’—This applies to any hazardous substances in use in any workplace. The employer should assess to evaluate the risk to health and establish what precautions are needed to protect employees. A written record of the assessment should be kept and details made available to employees.

2.9 Safety and Hazard Issues

47

The designer should be concerned more with the preventive aspects of the use of hazardous substances during the design process.

2.9.2 Flammability, Fire and Explosion Fire and explosion are the two most common chemical plant accidents. Organic solvents and liquid fuels are the major sources of fire and explosions in the chemical industry. The process design and project engineer should therefore be aware of the nature of the process leading to fire and explosion, flammability characteristics of the materials and the measures for reducing the possibility of fire and explosions. Fire is nothing but rapid, exothermic oxidation or combustion of an ignited fuel. Hence, the essential elements for a fire to occur are fuel, oxidizer and source of ignition. Of these, sufficient quantities of fuel and oxidizers are required. These three elements hypothetically form a triangle called a fire triangle. If any of the sides of such a triangle is missing, there will be no fire. Fuels may be solid (plastic, fibre, wood dust, metal dust), liquid (organic fuels and solvents) and gases or vapours (CO, H2 , organic gases or vapours). Oxidizers also can be solids (metal peroxides, ammonium nitrite), liquids (H2 O2 , HNO3 , HClO4 ) and gases (O2 , F2 , Cl2 ). Sparks, flames, static electricity or heat can be sources of ignition. Dust explosion is a comparatively less familiar phenomenon—for dust explosion, analogous to fire triangle, dust explosion pentagon should be considered where the extent of mixing and confinement are additional pre-conditions. The difference between fire and explosion lies in the rate of release of energy during the combustion/oxidation process. In an explosion, energy may be released in microseconds. Fire can result from the explosion and vice versa. The damage due to the explosion is caused by the pressure or shock wave. A project engineer should be acquainted with a few terminologies relevant to fire and explosion. Auto-Ignition Temperature (AIT)—By definition, AIT is a fixed temperature above which a flammable mixture is capable of extracting enough energy from the environment to self-ignite. Flashpoint (FP)—The flash point of a liquid is the lowest temperature at which it gives off enough vapour to form an ignitable mixture with air. At the flashpoint, the vapour will burn, but only briefly; vapour produced is inadequate for maintaining combustion. Flashpoint is higher at higher pressure. Fire point: It is the lowest temperature at which a vapour above a liquid burns continuously if ignited. Fire point temperature is higher than flash point temperature. Flammability Limits: A mixture of flammable vapour and air will ignite and burn over a well-specified range of composition. It will not burn if the volume per cent of the flammable vapour is below a certain value (lower flammability limit or LFL) or above another particular value (upper flammability limit or UFL). Below LFL, the

48

2 Pre-project Work

combustible component is too low to generate a flame and above UFL, the oxygen in the air is not adequate to support combustion. For safety, the vapour-air mixture should be out of the flammability range. An explosion can be of various types. A deflagration is an explosion where the resulting shock waves move through the air with a speed less than the sonic speed, whereas detonation is the explosion releasing shock waves at a supersonic speed. A confined explosion occurs within a building or vessel. An unconfined explosion occurs in the open air as a result of the release of flammable gas in the air. If the explosive vapour is diluted by air to make the composition of the gas below LFL before ignition occurs, there will be no explosion. When an explosion occurs, it is highly destructive in terms of spread area and severity. Boiling liquid expanding vapour explosion (BLEVE) generally occurs when an external heat source heats the contents of a storage tank full of a volatile liquid. The vessel ruptures and explosive vapour is generated from the liquid. If the vapour cloud is ignited, BLEVE occurs. Very fine powder of metal and other solids are very rapidly oxidized in the air with a high rate of release of energy. This is called a dust explosion. Fine particles of iron or aluminium and iron, even dust of flour sometimes explodes in the presence of air. Sources of Ignition: The third component required for the onset of a fire, other than the fuel and air, is the source of ignition. The source may be a spark from electrical equipment, static electricity, process flames or miscellaneous small sources like cigarette lighters or matches, petrol or diesel vehicles or exhaust, welding equipment and so on.

2.9.3 Ionizing Radiation For a few non-destructive testing of equipment or for level or density measuring instruments, small quantities of radioactive isotopes are used in process industries. Radiation from such isotopes is also dangerous and should be covered by government rules for handling radioactive materials.

2.9.4 High Pressure and Vacuum Overpressure, that is, a pressure exceeding the system design pressure, is one of the most serious hazards in chemical plant operations. Failure of a vessel and associated piping can give rise to a sequence of disasters. Underpressure or vacuum is also equally dangerous. Generally underpressure means a vacuum inside a closed vessel where the outside pressure is atmospheric. As a result, the vessel may collapse. As an example, it may be noted that if the pressure in a 10 m-diameter tank falls to 10 millibars below the

2.9 Safety and Hazard Issues

49

external pressure, the total load on the roof of the tank will be around 80 kN or 8157.73 kgf. It shows that a slight drop in the inside pressure can become fatal.

2.9.5 Temperature Deviation High temperature, more than the equipment design temperature, can cause the failure of the structure and result in a disaster. Either the loss in control of the reactor (internal factor) or open fire (external factor) may cause high temperature. Very low ambient temperatures or prolonged periods of intense cold can cause pipes to freeze and then burst as the melting chemical contents expand. Heavy ice can cause structural damage to equipment and brake pipes to release toxic or hazardous chemicals into the environment.

2.9.6 Noise Noise is unwanted sound. It may be considered as a wrong sound in the wrong place at the wrong time. The degree of ‘unwantedness’ is highly subjective. Long exposure to high noise levels can cause permanent hearing damage. At lower levels, noise is a distraction that causes irritation and fatigue. Noise measurements are reported in terms of decibels. The hearing mechanism responds to changes in sound pressure in a relative rather than an absolute manner. So decibel is dB = 10 log

x y

(2.24)

x → pressure, power or intensity. y → respective reference. The above-mentioned potential plant hazards are taken care of by modification of design and also by good engineering and management practices. During the preproject stage, one has to be aware of the potential hazards of the process they are proposing. Subsequent steps like basic and detailed engineering should be guided by the knowledge of the hazards [10]. Some of the design features resulting from the safety requirements are elaborated under the Basic Engineering chapter. Worked Out Examples Example 2.1 (Optimum production rate) A company produces certain small equipment at a rate of N r units per day. The variable cost per equipment is Rs. 500 + 0.1N r 1.2 . Total daily fixed charges are Rs.

50

2 Pre-project Work

25,000 and the other expenses are Rs. 50,000. If the selling price per equipment is Rs. 1100, determine the following: (a) (b) (c) (d) (e)

the optimum rate of production at the minimum production cost per equipment. daily profit when the production cost per equipment is the minimum. the optimum production rate at which the daily profit is maximum. amount of the maximum daily profit. The criterion of the break-even point whereby production rate can be found out.

Solution Total production cost per equipment: C T = 500 + 0.1Nr1.2 +

(50,000 + 25,000) Nr

C T = 500 + 0.1Nr1.2 +

75,000 Nr

Differentiating C T with respect to N r and setting the derivative equal to 0, we get Nr2.2 = 625,000 which gives Nr = 430 Hence, 430 units should be produced per day for minimum production cost per unit. Ans (a) Daily profit: Production cost per unit: 500 + 0.1 × 4301.2 + 75,000 = 500 + 144.6 + 174.42 = Rs. 430 819. Total production cost for 430 units: Rs. 352,170. Total selling price for 430 units: Rs. 473,000. Daily profit is Rs. 120,830. Ans (b) Daily profit is given by   75,000 1.2 R = 1100 − 500 + 0.1Nr + × Nr Nr 



Simplifying: R = 600Nr − 0.1Nr2.2 − 75,000

2.9 Safety and Hazard Issues

51

Differentiating R with respect to N r and setting the derivative equal to 0, we get Nr1.2 = 2727.273 which gives Nr = 729 So 729 units should be produced per day for getting maximum daily profit. Ans(c) Hence, maximum daily profit = Rs. [600 × 729 − 0.1 × 7292.2 − 75,000]. Rs. 163,790.2511. Ans (d) At break-even point, 500 + 0.1Nr1.2 +

75,000 = 1100 Nr

Solving for N r , production rate at break-even point can be determined. Ans(e) By trial-and-error method, it was observed that 133 units should be produced to break-even. Example 2.2 (Optimum cooling water) A condenser condenses at a rate of 2500 kg/h. The condensation temperature is 75 °C. The heat of condensation of vapour is 465 kJ/kg. The cost of cooling water at 25 °C is Rs. 10/m3 . The overall heat transfer coefficient at optimum conditions is 0.284 kJ/m2 s K. Cost for the installed condenser is Rs. 30,000/m2 . Annual fixed charges including maintenance are 20% of the initial investment. Heat capacity of water is 4.2 kJ/kg K. If the condenser operates 6000 h per year, determine the optimum cooling water flow rate in kg/h. Solution Given: U = 0.284 kJ/m2 s K. Hy = 6000 h/y. K F = 20% that is 0.2. C p = 4.2 kJ/kg K. C A = Rs. 10,000/m2 . Cw = Rs. 10/m3 that is Rs. (10/1000)/kg = Rs. 0.01 /kg. The following equation is used: (Tcond − T2,opt ) U Hy Ccw (Tcond − T1 ) − 1 + ln = (T cond − T2,opt ) (Tcond − T1 ) K F CC A C p The RHS is calculated as

52

2 Pre-project Work

0.284 × (6000 × 3600) × 0.01 0.20 × 4.2 × 30000 = 2.434 Using trial and error by a computer program or by using Figure 2 …, optimum outlet temperature of cooling water T 2,opt = 65 °C. So the optimum cooling water flow rate is calculated as follows: q˙ C p (T2 − T1 ) 2500 × 465 = 6919.6 kg/h = 4.2 × (65 − 25)

m˙ =

Ans.

Example 2.3 (Optimum number of effect of evaporator) A multiple-effect evaporator is being used to evaporate 2,00,000 kg/day of water from a solution. The total initial cost for the first effect is Rs. 18,00,000 and each additional effect would cost Rs. 15,00,000. The expected service life of the evaporator system is 10 years and the salvage value after 10 years is zero. Depreciation is calculated by the straight-line method (ref to Chap. 5). Fixed charges other than depreciation are 15% of the equipment cost. Steam economy, that is, kg of water evaporated per kg of steam is 0.85 × no. of effects. Steam cost is Rs. 0.30/kg steam. Annual maintenance cost is 5% of the initial investment for equipment. Other costs are independent of the number of effects. The evaporator unit operates 300 days per year. Determine the optimum number of effects for a minimum annual cost. Solution: Let the number of effects = x. Overall cost of equipment = V = 1,800,000 + (x − 1) 1,500,000. Annual depreciation = {1,800,000+(x−1)1,500,000}−0 depreciation calculated by the 10 straight-line method. Refer to Chap. 5. = 1,800,000 + (x − 1)1,500,000

(i)

Annual fixed charges other than depreciation: 0.15 × V = 0.15[1,800,000 + (x − 1)1,500,000] = 270,000 + (x − 1)225, 000

(ii)

Annual maintenance charges: 0.05 × V = 0.05[1,800,000 + (x − 1)1,500,000] = 90,000 + (x − 1)75, 000

(iii)

2.9 Safety and Hazard Issues

53

Annual cost for steam: Steam economy =

kg water evaporated = 0.85 × x kg steam used

Kg of steam required in a year = (kg water evaporated per day × 300 days/year/0.85 × x Annual cost of steam = (200,000 × 300 × 0.30)/0.85 × x = 21,176,470/x

(iv)

Now total annual cost = (i) + (ii) + (iii) + (iv) C T = 90,000 + 450,000x + At minimum annual cost, So,

dC T dx

21, 176, 470 x

should be equal to 0.

21,176,740 dC T = 450,000 − =0 dx x2 Hence x 2 = 47.05 and x = 6.85 ≈ 6. The number of effects would be 6 for the minimum total annual cost. Ans. Example 2.4 (Optimum reflux ratio) The annual fixed charges (A) and the annual utilities cost (B) of a distillation column are expressed in terms of the reflux ratio as follows: A = 650R 2 − 1796R +

1 + 1280 (R − 1.15)

B = 60 + 62R Write down the objective function and the equation to determine the optimum reflux ratio for minimum annual operating cost. Total annual operating cost (T) = Annual fixed charges (A) + Annual utilities cost (B)

54

2 Pre-project Work

1 + 1280 + 60 + 62R (R − 1.15) 1 T = 650R 2 − 1734R + + 1340 (R − 1.15)

T = 650R 2 − 1796R +

Ans.

This is the objective function for optimization. For optimum cost, ddTR = 0; so the equation is 1300R − 1734 −

1 =0 (R − 1.15)2

Ans.

Example 2.5 (Optimal total cost) Total cost of an equipment C T (lakh rupees) in terms of the operating variables x and y are given as follows: C T = 2x +

120,000 + y + 10 xy

What will be the optimal total cost? Solution   1 120,000 ∂C T =2+ − 2 +0+0 ∂x y x x 2 y = 60,000

(i)

  120,000 1 ∂C T =0+ − 2 +1+0 ∂y x y x y 2 = 120, 000 Putting value of x from (ii) into Eq. (i), y3 =

120,0002 = 240,000 60,000

y = 62.144 120,000 x= = 31.072 (62.144)2 So,

(ii)

2.9 Safety and Hazard Issues

55

120,000 + 62.144 + 10 31.072 × 62.144 = 196.434 lakh rupees

C T = 2 × 31.072 +

Ans.

Example 2.6 (Cycle time for minimum total cost) In a batch process, an organic chemical is produced. Each cycle needs 1.5 h charging/discharging time in addition to the operating time of 0.06Pr hours per cycle (Pr = kg of product produced per batch). The operating cost per hour is Rs. 1200 and the charging/discharging cost per hour is Rs. 1000. Annual fixed cost can be given by Rs. 300 Pr . Maximum operating time per day may be 12 h at 300 days per year. Annual production is 50,000 kg. Annual costs for raw materials and miscellaneous heads are Rs. 1,000,000. (a) (b) (c)

Determine the optimum cycle time for a minimum total cost per year. The total annual cost at the optimum cycle time. How much time is available for plant maintenance?

Solution. (All costs are in Rs.) Operating cost per cycle: 1200 × 0.06Pr + 1000 × 1.5 72Pr + 1500 Number of cycles per year: 50,000 Pr Now, based on the problem statement, total annual cost is given by C = (72Pr + 1500) × C = 3,600,000 +

50,000 + 300Pr + 1,000,000 Pr

75,000,000 + 300Pr + 1,000,000 Pr

C = 4,600,000 +

75,000,000 + 300Pr Pr

Differentiating C with respect to Pr and putting the derivative equal to 0 at minimum cost, we get Pr2 = 250,000

56

2 Pre-project Work

and then Pr = 500 kg per batch Now cycle time is 0.06×500+1.5 = 31.5 h. Annual cost: Rs. 49,00,000. × 31.5 = 3150 h. In a year, total operational hours: 50,000 500 Given that available plant hours in a year is 300 × 12 = 3600 h, time available for maintenance is 3600–3150 = 450 h.

Ans (a) Ans (b) Ans (c)

Summary After conceptualization, there are certain considerations and tasks by which it is decided whether the project should be taken up. In this chapter, a few such aspects are discussed. Optimum production rate and selection of proper site are two very important criteria. A formal feasibility report is critical for both construction and finance. Safety aspects are also to be considered. Based on all such analyses, the basic and detailed design of the plant are done. Exercise 1. 2.

3. 4.

5.

6.

What is the basic information required in a feasibility report for a chemical process plant? What should be the chapters of a feasibility report? What is a bid document? Why is it required in the pre-project stage and for whom? How can you distinguish between a feasibility report and a bid document? What are the main considerations for an eligible site for a process plant? Why should political stability be considered for a site location? Why is Environmental Impact Assessment (EIA) necessary for a project at an early stage of a proposed project? What are the major environmental impacts that should be analysed in this context? Why is optimization of design parameters necessary for a pre-project stage of a plant? Give three examples for the application of optimization in design in the pre-project stage without which the project could not proceed further. Saturated steam at 120 °C is passing through a steel pipe with an OD of 275 mm. The cost of steam is Rs. 0.30/kg. The pipe is insulated with an insulating material having a thermal conductivity of 5.2 × 10–2 W/m K. The cost of installed insulation per m is Rs. 12,500 × t, where t is the insulation thickness in m. Annual fixed charges including maintenance are 20% of the installed cost. The total length of the pipe is 300 m and the average temperature of the surroundings is 25 °C. Air film coefficient is constant at 11.4 W/m2 K. Other heat transfer coefficients are neglected. (a) (b)

Formulate the objective function to determine optimum economic insulation thickness. Determine the optimum economic insulation thickness using a computational trial-and-error method.

2.9 Safety and Hazard Issues

7.

8.

57

Why is scaling important for a process plant? Give an example of the application of scaling down in chemical industries. Compare scaling up by number and by size for a process plant. What is meant by ‘similarity-based scaling’? Mention a few aspects of similarity-based scaling. Indicate a few hazards of a process plant. Why are they considered hazards? Why it is necessary to be aware of the potential hazards before planning for a process plant? What is the difference between a risk and a hazard?

References 1. KLM Technology Group Project Engineering Standard. (2011, February). Recommended practice for feasibility studies. Project Standards and Specifications. 2. Pawar, D. N., & Nikam, D. K. (2017). Fundamentals of project planning and engineering. Penram International Publishing (I) Ltd. 3. Pavithra, P., & Pullaiah, G. (2013). Lecture notes on environmental impact assessment and management. College of Engineering and Technology. 4. Watermeyer, P. (2002). Handbook for process plant project engineers. Professional Engineering Publishing Limited. 5. Sinnot, R., & Towler, G. (2009). Chemical engineering design. Coulson and Richardson’s Chemical Engineering Series (Vol. 6). Elsevier (Butterworth-Heinemann). 6. Peter, M., Timmerhaus, K. D., & West, R. E. (2004). Plant design and economics for chemical engineers, Fifth International Edition. McGraw-Hill. 7. Agam, G. (1994). Scale-up and scale-down. In Industrial chemicals: Their characteristics and development, 1st edn. Industrial Chemistry Library (Vol. 6, pp. 169–187). Elsevier. 8. McKnight, N. (2013). Scale-down model qualification and use in process characterization. CMC Strategy Forum, 28 January, Genentech. 9. Luyben, K. Ch. A. M., Heijnen, J. J., van der Lans, R. G. J. M., & Potters, J. J. M. (2006). Lecture notes on scale-up/scale-down. Delft University of Technology, Faculty of Applied Sciences, Department of Biotechnology. 10. Crowl, D. A., & Louvar, J. F. (1990). Chemical process safety: Fundamentals with applications. International Series in the Physical and Chemical Engineering Sciences. Prentice Hall.

Chapter 3

Basic Engineering

After the Pre-Project works are done, design of the plant should start. The basis of design is material and energy balance—but basic material and energy balance or process equipment design are out of scope of this chapter; instead, here it shows how to present the outcome of the materials and energy balance and process equipment design. Process flow diagrams (PFD) show conditions of the process streams whereas necessary instrumentation and sizes of pipelines are shown in the piping and instrumentation diagram (P&ID). Types of P&ID, symbols for equipment and instruments, format for numbering lines—all these are described with examples. Learning objectives • To know the first few steps of how a ‘process’ becomes a ‘plant’. • To know about the basic drawings of a plant. • To be aware of the preliminary considerations for a plant design.

3.1 Preamble In Chap. 1, we came across the life cycle of a chemical engineering project. Basic Engineering is the first stepping stone by which a ‘process’ is converted to a ‘plant’. For that, there are many aspects like selection of the most appropriate equipment, environmental and safety considerations. Moreover, some basic documents have to be generated based on which further steps of plant design would be performed. Some of the aspects of Basic Engineering are the generation of process drawings, design and selection criteria of equipment, addition/alteration of process design concerning safety and environment considerations.

© The Author(s), under exclusive license to Springer Nature Singapore Pte Ltd. 2022 S. Chakrabarti, Project Engineering Primer for Chemical Engineers, https://doi.org/10.1007/978-981-19-0660-2_3

59

60

3 Basic Engineering

3.2 Process Drawings It was clarified that at the very beginning, the process of production has to be understood by all the parties involved in the project. For this, we have to explain the process not only by narration but also by schematic drawings in standard technical terms. The drawings by which the process is explained to all concerned to facilitate construction, operation, maintenance and revamp are called process drawings. Process drawings include Block process diagram (BFD), Process flow diagram (PFD), Piping and instrumentation diagram (P&ID) and Equipment layout. The purpose of process drawings are: • Used by process personnel to understand or explain a process with a visual representation of the process and equipment. • Used by detailed engineering personnel for specification of equipment/instrument/pipe materials. • During post-installation repair and maintenance, the PFD may be utilized by the technician. • Referred by operating staff to become familiar with the process in a safe environment and also during preparation of operation and maintenance manual for the whole plant. • Referred by engineers of other disciplines during interfacial engineering. The basic characteristics of the process drawings should be their simplicity and their explanatory nature. Moreover, it should use standard symbols and expressions so that it can be understood by members of the chemical engineering fraternity all over the world irrespective of the language and country.

3.2.1 Block Flow Diagram (BFD) A block (flow) diagram is the simplest form of a process flow diagram. It uses blocks for each stage of the process, in a sequence of processes leading to output from input. It is generally used to understand the whole process dividing them into several blocks. Each block may comprise several equipment [1] . Block diagrams may be of two types—block diagram for plant (Fig. 3.1) and block diagram for the process (Fig 3.2).

Fig. 3.1 Block flow diagram for a complex plant

3.2 Process Drawings

61

Fig. 3.2 Block flow diagram for a simple process

Compared with the previous figure, it may be observed that the block flow diagram for the process of production of benzene from toluene indicated in Fig. 3.2 is more specific and gives an idea about the particular chemical process.

3.2.2 Process Flow Diagram (PFD) Preliminary PFD is developed by the basic engineering consultant and provided under the basic engineering package. Later, it is used by different departments of the detailed engineering contractor for developing P&ID and other documents. The flow sheet is drawn up from material and energy balance made over the complete process and each individual unit. There are several authentic books for material and energy balance. Knowledge of reaction engineering is also required. However, discussions on basic reaction engineering, material and energy balance are excluded from the present scope assuming that those are already covered in other courses of the programme. PFD shall comprise but not limited to the following items/information: (1) (2) (3)

(4)

All process streams (properly numbered) as well as the operating conditions that are essential for the materials and energy (heat) balance. All utility streams (numbered) with their types (such as air, DM water) that are continuously required inside the plant (or unit) battery limit. Equipment (symbolic representation) should be placed serially according to the order of the process flow, their duty/designation and equipment number. Symbols for common equipment are given (Tables 3.1 and 3.2). Instrumentation and control systems applicable to the major process parameters control should be shown.

62

3 Basic Engineering

(5) (6) (7) (8)

(9) (10)

Major process parameter analysers. Operating conditions around major equipment. Duty of heating or cooling for all heat-transfer equipment should be indicated. Process conditions like operating pressure and temperature may vary along a process flow line through heating, cooling, compression or reaction; the changing conditions should be indicated using different stream numbers. All alternative operating conditions. A table containing flow rates, composition and physical properties of the streams should be furnished at the bottom of the flow diagram.

Among the above, the following information are mandatory [2]: (a)

Stream composition, in the form of either: (i) (ii)

(b) (c) (d)

the flow rate of each individual component kg./hr. the stream composition as a weight fraction.

Total stream flow-rate kg/hr. Stream temperature. Nominal operating pressure.

Sometimes the liquid flow rate, gas flow rate, stream temperature and pressure are shown on the respective streams instead of the stream table. For this purpose, specific labels are used on the streams that indicate flow rates, pressure and temperature (Table 3.3). The following items are generally NOT shown on PFD, except in special cases: (a) (b) (c) (d) (e) (f) (g) (h) (i)

Minor process lines are not usually used in normal operation and minor equipment, such as block valves, safety/relief valves, etc. Elevation of equipment. All spare equipment. Heat transfer equipment, pumps, compressor, etc., to be operated in parallel or series shall be shown as one unit. Piping information such as size, orifice plates, strainers, and classification into hot or cold insulated jacket piping. Instrumentation not related to automatic control. Instrumentation of trip system (because it cannot be decided at the PFD preparation stage). Drivers of rotating machinery except where they are important for control line of the process conditions. Any dimensional information on equipment, such as internal diameter, height, length, and volume. If an internal part of any equipment is required to demonstrate the working principle of the equipment in the process, only then that particular internal may be shown in the PFD.

PFDs are not drawn to scale, but the relative sizes of the equipment should be maintained for aesthetics. The direction of process lines should be from left to right. Major process lines should be thicker compared to the minor lines or utility lines.

3.2 Process Drawings

63

Table 3.1 Symbols (for PFD and P&ID) of a few equipment [3] Centrifugal pump

Packed column

Reciprocating pump with motor

Heat exchanger

Jacketed vessel

Plate type heat exchanger

Plate and frame filter

Plate column

Rotary vacuum filter

Atmospheric storage tank

Agitator with motor

Rotary dryer/kiln

Electric furnace

64

3 Basic Engineering

Table 3.2 Symbols of a few valves and piping materials [3] Gate valve

Control valve

Globe valve

Needle valve

Check valve

Motor operated valve

Butterfly valve

Steam trap

Diaphragm valve

Plugged drain

Ball valve

Vent

Safety valve (spring-loaded)

Open drain funnel

Solenoid valve (three-way)

Expander/reducer

Venturi meter

Orifice

Rotameter

Strainer

Hose connector

Expansion joint

Cross-over of lines should be avoided; however, where they must cross, the horizontal lines should be continuous and the vertical line should be broken. Basic information extracted from a PFD can be divided into three categories— process topology, equipment information and stream information. All these together generate a PFD.

3.2 Process Drawings

65

Table 3.3 Stream specification in PFD [3] Liquid flow (kg/h) Gas flow (kg/h) Pressure and temperature

This indicates that stream no. 1 carries 1300 kg/h liquid at 8 atm (abs) pressure and 25 °C temperature. No gas is there

In process topology, PFD indicates the relationship between various equipment and process streams. As specified before, equipment and process stream conditions are indicated using universally accepted symbols. From a standard protocol for equipment numbering, the type of equipment and the section of the plant where it operates can easily be identified. From the standard flags appearing on a particular stream, as well as from the stream table at the bottom of the PFD, information of the respective stream can be obtained. Approximately 10,000 TPA nitric acid is to be produced from the oxidation of ammonia with air in a catalytic reactor. Heat is recovered from the outlet stream of the reactor; after that, the product gas is absorbed in an absorber column by process water. Condenser acid is mixed with the bottom product and product acid is released thereafter. 60–65% HNO3 is produced (Fig. 3.3) [2].

3.2.3 Piping and Instrumentation Diagram (P&ID) Process flow diagram (PFD) is a description of the process. Piping and Instrumentation Diagram (P & ID) shows the engineering details of the equipment, instrument, piping, valve and fittings and their arrangements. It is often called the Engineering Flow Sheet or Engineering Line Diagram. It is considered the most significant engineering document for a process plant.

3.2.3.1 • • • • •

Purpose and Use of P&ID

Defines the scope of work. Used to prepare equipment list. Used to estimate manhour. Transmits essential design information to the engineering design groups. Used to prepare preliminary piping and special materials BOM.

66

3 Basic Engineering

Fig. 3.3 Sample PFD

• Used to prepare preliminary plot plans. • Used to provide the design basis. • Used to prepare basic job documents like line designation table, instrument index, insulation and steam-tracing, piping material classes, motor and driver schedule. • Used for controlling Engineering Change Request. • Used for constructability review. • Used by construction, start-up and operations group. • Used to establish methods of testing and commissioning. • Used for preparing operation and maintenance manual. 3.2.3.2 (1)

Types of P&ID

Process P & ID

Process P&IDs are P&IDs related to the main production process. It may have different units and accordingly, there may be different P&IDs; but it contains a description of the process in focus.

3.2 Process Drawings

67

Fig. 3.4 Sample part of process P&ID

(a)

On-plot process unit P&ID—The adjective ‘On-plot’ indicates that these are within the process plant battery limit or area. P&IDs for the units such as compressor, scrubber or reactors are for detailing out the instruments, control and pipelines in that particular unit; a collection of such unit P&IDs make the P&ID of the whole process plant. Each of these P&IDs is called On-plot process unit P&ID.

In Fig. 3.4, pipelines, instrumentation and control of a separator vessel are shown with proper symbols. It is part of a Process P&ID. The lines and valves are numbered as well as the instruments. Control logics are explained later. It may be observed that in/out of streams may be from/to some other Process P&IDs. (b)

Off–plot P&IDs—P&IDs for storage tanks, blending and shipping systems are generally considered under this category. Storage of raw materials and products, blending and packaging units are generally not within the main processing area; still, it is very much a part of the process plant. Hence, piping and instrumentation details of such units are called ‘off-plot’.

In Fig. 3.5, a storage tank is shown with its necessary instrumentation and control. It is also a part of a storage tank area P&ID. From the storage area, fluid is sent to the process plant. (2)

Utility plant P & ID

Cooling water, demineralized water, steam, instrument air—these are some of the substances that are required for all chemical plants irrespective of the product. These are called ‘utilities’. Recycled cooling waters should be treated by dosing and cooling before reuse. Demineralized water should be produced for the boilers by ion exchange. Instrument air should be prepared by purification of ambient air

68

3 Basic Engineering

Fig. 3.5 Sample part of tank area P&ID

and so on. All these processes need some unit operations, but these processes are not related to the main process plant. P&IDs indicating the engineering details of such treatment facilities of the utilities are called utility plant P&ID. Cooling towers, air compressors, boilers, water treatment plants are some of the common units in such P&IDs. In the following utility plant P&ID for instrument air (Fig. 3.6), ambient air is filtered, compressed and cooled before moisture is removed from it. In addition to standard equipment, a vendor’s package IA-D-1101 (say) is used. Since details of the package are not known, the only outline is used. (3)

Utility distribution P & ID

Above mentioned utilities are required in various sections of the whole plant. All utilities are generally distributed to different sections of the plant through distribution headers from the utility processing plants. These headers may require fittings and instrumentation/control in course of its run from the utility plant (source) to the specific section of the main process plant (destination). P&IDs of distribution of these headers, where the destination locations are specified along with the instrumentations, are called utility distribution P&ID. Proper distribution as per the requirement to the specific equipment or section is very important and needs significant engineering. This distribution should correspond to the physical layout. Figure 3.7 shows a typical utility distribution P&ID. It may be observed that a particular utility may be required in different sections of the plant. Sometimes those utility lines need specific instrumentation before entering that section. These are covered by utility distribution P&ID. Respective equipment are generally shown by

3.2 Process Drawings

69

Fig. 3.6 Sample part of utility (instrument air) plant P&ID (referred to for special control P&ID, Fig. 3.10)

Fig. 3.7 Sample part of the utility distribution P&ID

70

3 Basic Engineering

Fig. 3.8 Typical segment of interconnecting line P&ID

dotted lines. Flare lines are sometimes shown among these lines where discharges from safety valves are also included. The basic difference between utility plant P&ID (or simply utility P&ID) and utility distribution P&ID is that the latter connects the former with the process plant segments where the respective utilities are distributed; the former for individual utility describes the processing of that particular utility. (4)

Inter connecting line P & IDs

These type of P&IDs shows connecting line between individual process, utility plant and utility distribution P&ID. Physically interconnecting P&ID describes the situation in between different units of the plant (Fig. 3.8). Generally prepared for offsite pipe racks and link the various process and utility plants. (5)

Auxiliary P & ID

Auxiliary P&ID’s are prepared for rotating equipment such as compressors, turbines, pumps and centrifuges where equipment is sufficiently complicated and the vendor does not provide separate P&ID for a specific auxiliary service. For example, if a pump vendor does not provide details of the lubrication system of their pump, an auxiliary P&ID is required to show the distribution of lubricating oil into various parts of the pump. The auxiliary P&IDs shall show auxiliary equipment, utility piping etc. and the piping, instrumentation etc. to be supplied by the vendor. It should also define the scope of supply by the vendor. It should cross-refer the vendor’s terminal connections and/or document number. Bearings of the motor-driven pump require lubricating oil for their smooth movement. Hydrodynamically lubricated sleeve bearings may require a pressurized lube

3.2 Process Drawings

71

oil system. Lube oil reservoir, oil circulation pump, lube oil cooler, pipelines, valves and instruments constitute the auxiliary lube oil system for a pump. API 610 and 614 are standards for the pump lubrication system. This auxiliary lube oil system is not shown in the process P&ID. Figure 3.9 is an example of the auxiliary P&ID. Generally, this system is provided by the vendor, hence no tag number is used for its equipment and instruments. If the vendor does not provide the system, this P&ID should be prepared during the detailed engineering phase with the vendor’s recommendation. Auxiliary P&IDs are not included in process P&ID since it would make process P&ID complicated to understand. (6)

Special control diagrams

We know that the main process P&ID shows primary control elements, but control systems for a few types of equipment are too complex to be shown in the process P&ID. In such cases, separate P&IDs are required for showing their control schemes. Reference to the main process P&ID should be indicated. This type of P&ID is generally prepared during the detailed engineering stage with the consultation of the vendor of the equipment (Fig. 3.10). Figure 3.6 should be referred back here for understanding the use of special control P&ID. A cloud was drawn around the compressor and this Fig. 3.10 was referred for special controls required for the compressor. This control and instrumentation are required for the compressor; but if these were included along with the compressor

Fig. 3.9 Sample part of auxiliary P&ID

72

3 Basic Engineering

Fig. 3.10 Sample special control P&ID for a compressor (referred to in Fig. 3.6)

in Fig. 3.6, the utility P&ID would be difficult to understand. That is why a separate special control P&ID is prepared.

3.2.3.3

Information Generally Present on a P&ID

Details of the information gathered on a P&ID at different stages of its development will be described later under various stages of the P&ID. Generally, the following information should be present (1)

(2)

(a)

All the process equipment should be shown bearing the proper equipment number. The equipment should be drawn roughly in proportion, and the location of nozzles shown. The static head, if any, should be mentioned. All pipes are identified by a line number. The pipe size, rating and material of construction (MOC) should be shown. The material should be incorporated as part of the line number. Line numbering system

A typical line number should be as follows (Fig. 3.11). According to Indian standard, codes for a few selected fluids and insulations are given as below (Table 3.4). Plant section—A chemical plant is generally made up of several sections. All such sections are identified with a number and have a separate P&ID. Sometimes more than one P&ID is required for one section. Line, fittings and instruments are

3.2 Process Drawings

73

Fig. 3.11 Typical line numbering system

Table 3.4 Codes generally used for pipeline numbering Service fluid code

Insulation code

Process fluid

P*

Hot insulation

IH

Cooling water supply/return

WCS/WCR

Cold insulation

IC

Raw/treated water

WR/WT

Steam tracing

IT

Instrument air

AI

Electrically traced

IE

Fuel oil/gas

FO/FG

Drain

D

High/medium/low-pressure steam

SH/SM/SL

Note: In the present example (Fig. 3.11), the fluid is medium pressure (50–250 psig) steam, so SM legend is used

*

Different legends can be provided to different process fluids instead of generalization

numbered according to the plant section number so that they can easily be found out. In the given example, the pipeline belongs to Sect. 21 of a plant. Serial number—There are many pipelines for each service in a particular section. They should be numbered serially according to appearance. In the present example, the pipeline is the first line carrying medium pressure steam in plant section 21 (say). Piping class—Piping class is a part of pipe specification, which is usually prepared or specified during basic engineering. This is useful not only during the construction of a new plant but also during the maintenance of an operating plant. It includes the type of pipe, schedule, material, flange rating, branch types, valve types, valve trim materials, gasket and all other requirements to be used for transporting various fluids under different operating conditions in a plant. Based on the operating temperature, pressure and corrosion vulnerability, the class of a pipe is defined; a shortcode is provided for each pipe class to cover all information applicable for that particular class. This pipe class is included in the line number so that the site construction engineer can identify the material. Pipelines carrying different fluids through it and having different characteristics should be designated with different symbols in the P&ID. Some of the symbols for pipelines and instrumentation signals are given below (Table 3.5). (b)

Recommended velocities for pipeline sizing

Sometimes pipelines have to be sized for line numbering. As a rule of thumb following velocities (Table 3.6) are recommended for sizing of single-phase flow through pipes. Sizing criteria for pipes carrying two-phase flow are, however, more complicated.

74

3 Basic Engineering

Table 3.5 Symbols generally used for types of pipeline and instrumentation Process line

Instrument air signal

Traced line

Hydraulic line

Electrical signal

Instrument capillary tubing

Software signal

Buried line

Table 3.6 Recommended velocities for a few fluids in pipes

(c)

Fluid

Recommended velocity (m/s)

Water

1.5–2.5, typical 2

Air

10–30; typical 25

Steam Low pressure High pressure

20–45 30 45

Process gas Dry gas Wet gas

15–45 30 18–20

Colour codes of pipeline paintings

Through colours are not specified in the P&ID, it is pertinent for a chemical process engineer to know the colour codes of the painting of the pipelines to identify the fluid flowing through it. Pipes are identified by ground colours and colour bands. Ground colours are applied to identify the basic nature of fluids whereas colour bands are used to identify the different conditions of the same fluid or to distinguish between the two fluids belonging to the same group. Paints are applied throughout the length on the uninsulated pipe or the metal cladding of the insulation. While details of the colour codes are given in BIS-2379-1990 (Indian standard), some basic colour codes are given in Table 3.7. Hazard marking—Stripes black and golden yellow. Hazard marking of radioactive fluid—Cross stripes of light orange and black. (3)

(4) (5)

All control and isolation (block) valves with an identification number. The type and size should be shown. The type may be shown by the symbol used for the valve or included in the code used for the valve number. Ancillary fittings that are part of the piping system, such as inline sight glasses, strainers and stream traps, each with an identification number. Pumps and compressors, identified by a suitable code number.

3.2 Process Drawings Table 3.7 Recommended colour codes for pipelines [4]

(6)

75 Fluid

Ground colour First colour band

Cooling water Sea green

French blue

Drinking water

Sea green

French blue

Untreated water

Sea green

White

Wastewater

Sea green

Canary yellow

Compressed air over 15 kg/cm2

Sky blue

Signal red

Plant air

Sky blue

Silver grey

Second colour band Signal Red

Signal Red

Instrument air Sky blue

French blue

Very high-pressure steam

Aluminium as per IS-2339

Signal red

Low-pressure steam

Aluminium as per IS-2339

Canary yellow

High-speed diesel fuel

Light brown

Lube oil

Light brown

Light grey

Chlorine gas

Canary yellow

Dark violet

Coal gas

Canary yellow

Signal red

Brilliant green

Ethylene gas

Canary yellow

Dark violet

Signal red

Hydrogen

Canary yellow

Light orange

Signal red

French blue

Strong caustic Smoke grey

French blue

White

Sulphuric acid Dark violet

Brilliant green

Light orange

Nitric acid

Dark violet

French blue

Light orange

Mercury

Black

White

Brilliant green

Soap solution

Black

Light orange

White

All control loops and instruments, with respective identification numbers. However, these pieces of information are not all put together at the beginning of the P&ID; rather the pieces of information generally are inputs from different parties involved and incorporated at different stages of the P&ID development. Relevant symbols are given in Tables 3.8 and 3.9.

Example: At the outlet line of an air compressor, a local pressure indicator is mounted. From that indicator, an electrical signal is going to the DCS and therefrom the driver motor of the compressor is accordingly controlled to keep the outlet pressure as desired. It is pictorially represented in Fig. 3.12.

76

3 Basic Engineering

Table 3.8 Recommended symbols for instrument position Main panel

Rear panel

Local

Local panel

Interlock

Distributed control system (DCS)

Programmable logic control PLC

PLC on local panel

Table 3.9 Instrument identification letter/legend

Letter

First letter

Succeeding letter

A

Analyser

Alarm

C



Controller, close

E

Voltage

Element

F

Flow rate



H



High

I

Electric current

Indicator

J

Power



K

Time, schedule

Control station

L

Level, lock

Light, low

P

Pressure

Point

R

Radiation

Recorder

S

Speed, frequency, safety

Switch

T

Temperature

Transmitter

V

Vibration

Valve

Y

Event, state

Relay, compute

Z

Position

Driver, actuator

Note 1: Since the measured quantity is pressure, the first letter is P. Since it is an indicator, the second letter is I. An arbitrary number 101 is assigned—say it is the first pressure indicator in the plant Sect. 3.1. Since it is a locally mounted instrument, this symbol is used. Note 2: Symbol for the electrical signal is used. Note 3: Since the measured quantity is pressure, the first letter P is used and since it is a controller, the second letter used is C. Because it is connected with indicator 101 and it is the first controller of plant section 1 (say), the number is 101. The symbol of DCS is used as well.

3.2 Process Drawings

77

Fig. 3.12 Symbols of P&ID explained

Control loops [2] Level control—A vessel V-301 is receiving feed and gets unloaded by pump P-301. It should be monitored that the feed level in the tank should not fall below a set level to prevent the pump from cavitation. A level indicator controller LIC 301 is used for control valve LCV 301. The desired level is set at the controller below which the valve will be closed by the controller. A low-level alarm is also used where the alarm will be given at a level just above the closing set point. The control is schematically shown in Fig. 3.13. Pressure control: A vessel V-201 contains fluid at a particular pressure. If the pressure is higher than a set value, it will vent out some vapour through control valve PCV201B. On the other hand, if it is lower than a set value, the pressure will be maintained by injecting nitrogen gas through the control valve PCV-201A (Fig. 3.14).

Fig. 3.13 A typical level control loop

78

3 Basic Engineering

Fig. 3.14 A typical pressure control loop

However, if the fluid or vapour stored is toxic, hazardous or valuable, it should not be vented to the atmosphere, but a closed pipeline leading to flare or vessel. Temperature and flow control: Temperature and flow controls are shown separately in the same Fig. 3.15. Cold fluid is getting heated to a particular temperature by steam in vessel H-101. The temperature inside the vessel is monitored and transmitted by TT101; accordingly, the inflow of steam is regulated by TV-101. The outflow of heated

Fig. 3.15 Typical temperature and flow control loops

3.2 Process Drawings

79

Fig. 3.16 Typical cascaded level and flow control loop

fluid is required at a particular flow rate, which is controlled by a flow control valve FV-101, which receives flow rate signal from FT-101 via flow controller FIC-101. Controllers are all panel mounted whereas elements and transmitters are local. Cascade control—Sometimes, the output of one controller adjusts the set point of another controller. This type of control is called cascade control. Figure 3.16 shows one such control scheme. In a vessel V-201, a particular liquid level should be maintained adjusting the inflow. The set value of the level is fed to LIC-201, which feeds that to flow controller FIC-201. In FC-201, the flow rate is also fed by FIT-201. Depending on the flow value received, FIC-201 controls the inflow by manipulating FV-201. Ratio control—In a chemical plant, it is very common to mix two liquids at a particular ratio. Figure 3.17 shows such a typical ratio control schematics. The flow of fluid B is controlled by the level and its flow rate. Required ratio of the flow rates of A: B is set into the ratio controller X-202 where the flow rate of fluid B is also entered. By maintaining the particular ratio, the flow of fluid A is monitored via FC- and FV-202A. Alarms, safety trips and interlocks: Alarms are used to alert operators of serious and potentially hazardous deviations in set process conditions. Key instruments are fitted with switches and relays to operate audible and visual alarms on the control and annunciation panels. Where delay or lack of response by the operator is likely to lead to the rapid development of a hazardous situation, instruments are fitted with a trip system to take action automatically to avoid the hazard such as shutting down pumps

80

3 Basic Engineering

Fig. 3.17 Typical ratio control of two flows

closing valves etc. Trip and alarm may either be combined or may be separate (Fig. 3.18a, b). The basic components of an automatic trip system are: 1. 2. 3.

A sensor to monitor the control variable and provide an output signal when a preset value is exceeded. A link to transfer the signal to the actuator usually consisting of a system of pneumatic or electric relays. An actuator to carry out the required action close or open a valve switch off a motor.

Fig. 3.18 Alarm and trip: a together b separate

3.2 Process Drawings

81

In Fig. 3.18a, the TIC raises an alarm for a high temperature and closes the control valve. In Fig. 3.18b, there are two separate sensors—one for alarm only and the other for alarm and closing of the valve for shutting down. In this case, generally, two set points are set—the first set point is only for alarm at a lower value of temperature. If the issue is taken care of and the temperature is controlled within a reasonable time, there will be no shutdown for which the setpoint is slightly higher. If the temperature cannot be controlled after the alarm, the second alarm and trip occur together. Interlocks are used when a fixed sequence of operations should be followed. The sequence can be programmed before and the operations take place sequentially as per the programme. A typical example is the sequence of operation during the start-up or shutdown of a plant or plant section. (d)

Distillation column control—a typical example:

The primary objective in the process control of a distillation column is to maintain a specified composition of the top and bottom products and any side streams. Following process parameters should be controlled: 1. 2. 3. 4. 5. 6.

Feed flow rate and temperature. Composition of distillate product. Steam supply pressure—for the reboiler at the bottom. Cooling water pressure and flow rate—for the condenser at the top. Temperatures of the reboiler and the condensers. Reflux rate.

It is observed that the column has one top and one bottom product and no product is taken out from any intermediate position. There are many more controls in a distillation column for a perfect operation. With the advancement of technology, each tray of a distillation column nowadays has many separate control loops. However, the following control scheme (Fig. 3.19) is given for a simplified and preliminary idea of control loops in operating equipment.

3.2.3.4

Different Versions of a P&ID

As described in Chap. 1, the development of P&ID is a continuous process involving various parties. At the same time, it is also true that the development of P&ID should not continue for so long that it may alter the overall project schedule. It should be finalized for further work of the project as soon as possible incorporating inputs from all the parties concerned. But the inputs are incorporated at various stages. These different stages may also be referred to as versions of the P&ID. The development of P&ID from ‘Preliminary’ to ‘For Construction’ versions are divided into the following stages or versions Issue 1: Preliminary, for in-house review Rev A This is a draft version developed from the PFD by the Process group and is sent to different sub-groups like mechanical and instrumentation for their inputs. At this

82

3 Basic Engineering

Fig. 3.19 Typical P&ID showing control scheme of a distillation column

stage, no information is available from the vendors and this version should not be sent to the owner or client. The information that have to be present in this version are Equipment: • • • •

All major equipment including numbers, titles. Critical elevations of special equipment. Vessel internals. Outlines of supplier–furnished package units.

Piping: • All main flow lines including valves and breakout spools on lines and equipment. • Defined auxiliary systems. Instruments: • Principal Control Systems/Instrumentation functions and preliminary location of function (i.e. local, distributed system, pneumatic etc.) mainly required for the process. • Location of the safety relief valves. Issue 2: Issue for Client’s approval, results from in-house review, Rev B

3.2 Process Drawings

83

Various departments from the house itself comment on the Preliminary version of P&ID and the ‘For Client’s Approval’ version is generated. In this version, the information already presents in the previous version is further refined and some more information are added. In addition to the information present in Issue 1, the following information should be added to this version Equipment: • Complete heading information. • Material classification on columns and vessels. Piping: • • • • • • •

Line numbers, sizes and specs. Boiler code and limits. Sample points and types of sampling devices. Steam traps and types. Complete missing auxiliary systems and piping. All Valving. Tight shut off valves and operating blinds.

Instruments: • • • •

Detailed control schemes. Control valve manifolds. Relief valve. Control/measurement/alarm functions/location mounted, DCS, PLC, computer, relay logic). • Secondary and local instruments.

of

function—(local/panel

Issue 3: Issue for design, results from client’s review, Rev C After the client approves the P&ID with or without inputs from their side, the P&ID is used for detailed engineering. This version of the P&ID forms the basis of the detailed design of the plant. Detailed Engineering contractor prepares the drawings/documents for procurement and construction based on this version of the P&ID. In addition to the previous version (Issue 2), following information are given in this version of P&ID (Issue 3). Equipment: • Final number of exchangers. • All ancillary equipment (fan, damper, air preheater etc.). Piping: • • • •

Sizes of all relief lines. Complete operating vents and drains. Final line sizes and specs. Code numbers for special items.

84

• • • • • • • •

3 Basic Engineering

Utility tie—ins. Complete gas purge—lines. Complete steam—out and surfing steam lines. Complete/closed drain system. Complete minimum flow by-passes if required. Lines for special pump seal fluids. Major expansion joints. Complete type of sample connections and of steam traps.

Instruments: • • • • • •

Instrument numbers. Control valves with manifold type and fail position (with notes). Relief valves. Flow meter run sizes if different from line size. Instrument purging and winterizing. Detailed functional requirements (complex control schemes, software lines etc.).

It may be noted that some of the above information are often incorporated by the Detailed Engineering contractor as part of the detailed design. Actually, there is no hard and fast division about who would incorporate which information. Necessary information are included by any of the parties and after ensuring all necessary data are available, the P&ID is released for construction with the consent of all parties concerned. Issue 4: Issue for construction, results from design—development, Rev D After the detailed design is complete with most of the information including those from various vendors, all parties approve the P&ID and release the same for construction of the plant. In addition to the information gathered so far, following information should be included in this version of the P&ID Equipment: • • • • •

Final number, size and nozzle details of exchangers. Heater arrangements with all ancillary equipment like air preheater and dampers. Sizing and details of most of the static and rotary equipment. Drivers with details. Updated information about equipment and piping furnished by supplier/vendor. Drawings/documents from vendors should be referred to at respective places on P&ID. A ‘hold’ is marked on items that are not yet firm.

Piping: • • • •

Sizes of all relief lines. Complete operating vents and drains. Final line sizes, specs, insulation and tracing requirements. Code numbers for special items.

3.2 Process Drawings

85

• Utility tie—ins. Instruments: • Details received from vendors/suppliers should be updated. • Utilities required and its connections should be marked. • Scope of supply should be clearly marked. For example, if an instrument requires an isolation valve, but the vendor would not supply it, it should be marked and to be arranged. It may be noted that this version of the P&ID is developed during the Detailed Engineering stage rather than the Basic Engineering stage. However, for continuity, this is described in this chapter. This version of P&ID is used for construction at the project site. Any modification after issuance of this version of P&ID should be routed through all parties as Engineering Change Notice (ECN). After the construction of plant is over, another version of P&ID is released—that is called ‘As built’ version. That P&ID forms part of the Operation and Maintenance Manual of the plant. It may be mentioned that each of the above versions may have more than one subversions. The P&ID of a particular version may have to be circulated many times among different parties. Sometimes those sub-versions are numbered with numerical and sometimes they are not numbered at all.

3.2.3.5 I.

General Guidelines

Title Box and directions

The Title Box of P&IDs shall be located at the bottom right corner of the sheet and should contain the following: The first line shall always be “PIPING AND INSTRUMENT DIAGRAM”. The second line shall identify the specific subject covered. Main equipment data shall be given at below the equipment number that stated at the base of P&ID. Revision/issue/signature boxes provided at the base of the P&ID. Note boxes provided at the right side of the P&ID. Note boxes and revision boxes are sometimes placed above the title box. Drafting guidelines are the same as that for PFD. Drain lines should be located at the bottom and vent lines should be located at the top. II. 1.

2.

Equipment Arrangement P&IDs shall be divided into four horizontal sections. Equipment such as towers, drums, heaters and tanks shall be placed in the two upper sections. Heat exchangers shall be shown in the third section and pumps in the lower section. The divisions into sections may be altered to suit particular situations. For example, if a condenser is to be located over an accumulator, it shall be indicated in that manner on the P&ID. When a unit has many interconnecting lines between heat exchangers, the P&IDs may be simplified by placing some

86

3.

III.

3 Basic Engineering

exchangers on the same line with the towers or pumps. Such exceptions are justifiable in order to obtain a more realistic representation and an orderly arrangement of piping. Another example where flexibility is permitted is chemical type plants where piping or ducting may be better shown in elevation. The minimum elevation shall be specified at the lower tangent line of all vertical vessels, at the bottom of all horizontal vessels and for other equipment items where the elevation is critical to the process. Where several items are interrelated, concerning elevation, the minimum elevation will be noted for only one of the items and dimensions used to show the relationship with the other items. An example is a reboiler, which must have a definite relation to its associated tower. Equipment-details:

Following should be the details of the specific equipment: (1)

Vessels and Tanks • Outlines in proportion to size. • Do not show manholes. • Internals likes trays, demister pads, steam coils etc. are to be shown schematically; in dotted lines. Details of the internals are only required to define external connections or to clarify the vessel function.

(2)

Fired Heaters: • • • • •

(3)

Air Cooled Exchangers: • • • •

(4)

Show arrangements of convection and radiant coils. Show all parallel passes. Draw schematically all related equipment and components. Show complete detail of one burner and write the number of burners. Allow generous space to accommodate required instrumentation.

Show all cooler sections. Show all inlet and outlet manifolds. Show fans, louvres, motors and motor controls. Show stream coils where required.

Shell and tube exchangers: • Show outline based on data sheet. • Show multiple shells when required. • Do not show multiple sections for double-pipe exchangers.

(5)

Pumps, compressors, stream turbines: • Show strainers on turbine inlet streamlines. • Show temporary strainers on pumps and compressor suction lines. • Show utility lines serving pumps and compressors on process P&ID if not shown in auxiliary P&ID.

3.2 Process Drawings

(IV) (1)

Pipelines and valve details: Lines • • • • • • • • •

(2)

87

Show main process flow lines as heavy lines. Do not differentiate between connection types. Show flanges only when required for process reasons. Process lines entering or leaving the P&ID should take the shortest route to either side of the drawing. Utility lines connecting to the equipment shall start and end at two sides. Turbine steam and condensate lines shall start and end at a convenient level above the turbine. Utility lines to pumps and compressors shall start and end at the bottom of the drawing. Process drain lines shall terminate at the bottom of the drawing. Instrument auxiliary lines shall not be shown on P&IDs.

Valves: Valves to be shown on P&ID: • All process block valves. • Operating vents and drains. • Valves isolating instrument standpipes. Valves not to be shown on P&ID: • • • • •

(V)

Individual instrument isolation valves. Instrument vent and drain valves. Steam trap valves. Valves required only for hydrostatic testing. Valve codes are shown only for speciality valves or out of spec valves.

Special System Requirements: • Show necessary information to alert design groups such as— • Elevation of thermosyphon reboilers. • Symmetrical piping connection to fired heaters, parallel heat exchangers, air coolers. • Elevation and location of special control valves. • Restrictions of length, position etc. for two-phase flow. • Lines that must be sloped or cannot have pockets.

Line numbering guidelines, the symbol for valves and nomenclature for instruments are given in Table 3.2.

88

3 Basic Engineering

3.2.4 Plot Plans or Layouts Plant layout refers to an optimum arrangement of different facilities including different sections, individual equipment, utilities and control rooms. Since a layout once implemented cannot be easily changed and the costs of such a change are substantial, the plant layout is a strategic decision. It is a crucial function that has to be performed both at the time of initial design of any facility and during its growth, development and diversification. Some of the important objectives of a good plant layout are as follows: • Simplification of the production process in terms of equipment utilization, minimization of delays, reducing manufacturing time, and better provisions for maintenance. • The overall integration of man, materials, machinery, supporting activities and any other considerations in a way that results in the best compromise. • Minimization of material handling cost by suitably placing the facilities in the best flow sequence. • Saving in floor space, effective space utilization and less congestion/confusion. • Increased output and reduced in-process inventories. • Better supervision and control. • Worker convenience and worker satisfaction. • Better working environment, the safety of employees and reduced hazards minimization of waste and higher productivity. • Avoid unnecessary capital investment. • Higher flexibility and adaptability to changing conditions. 3.2.4.1

Master Plot Plan and Unit Plot Plan

During the development of Process Flow and Piping and Instrumentation Diagrams, the equipment are also to be arranged in a logical sequence within the given plot area so that the materials flow is economical and movement of the personnel are safe. Such spatial arrangement of equipment, pipeline and instruments along with their interconnections within the piece of land earmarked for the particular unit are called equipment layout or plant layout. This is an important aspect of plant design since a good layout ensures the proper and safe functioning of the plant. Provision for future expansion should also be made within the layout. As mentioned earlier, a plant is generally comprised of several sections or units. For example, a naphtha cracker plant generally has processing sections like a hot section, quench section, cold section and so on. Each of the sections has a separate PFD, P&ID and equipment layout. The equipment layout for a particular processing section is called Unit Plot Plan. Apart from the process units, there are several facilities and areas on the site. Some of them are: • Storage for raw materials and products—tank farms and warehouses.

3.2 Process Drawings

89

• • • • • •

Maintenance workshops. Stores for maintenance and operating supplies. Laboratories for process quality control. Fire stations and other emergency services. Utilities—steam, air and water processing units. Effluent disposal plant—wastewater treatment, solid and or liquid waste collection. • Offices for general administration. • Canteens and other amenity buildings—such as medical centres. • Parking lots. Accommodating all these facilities within the land economically and safely is called Site Plan or Master Plot Plan. It may be mentioned that ‘layout’ drawings are those that are produced before the stage of receiving final equipment details and structural design. The layouts, therefore, include overall or critical dimensions only, whereas the final ‘general arrangements’ are those used for controlling and verifying plant construction details. Layouts are intermediate drawings, not for construction. However, final (frozen) layouts must be able to accommodate any variations in equipment size, piping design, etc. that may be required by final detail design [2, 5, 6] . Master plot plan—considerations for development As defined earlier, a master plot plan shows the location of each process unit, roadway and buildings inside the specific piece of land regarding the highway and railroad available nearby. Generally, the plant is divided into blocks that are most conveniently separated by roadways. A sample master plot plan is given in Fig. 3.20. Following factors have to be considered while developing a Master Plot plan: • General climate and site conditions: Prevailing weather and site conditions should be considered in the development of general plant arrangements. • Terrain and topography: Site topography, soil conditions and geology should be considered and used to minimize earthmoving and foundation requirements. Multiple ground elevations may be used in a plant to avoid extensive cut and fill. The drainage system should be planned from a contour map of the site. The portion of the site requiring the least amount of fill and having the best load-bearing characteristics are selected for roadways, process units and buildings. The highest area may be advantageously used for the storage of products so that gravity flow is possible. Adequate dikes are to be built around these tanks for safety. • Reference point: Grade elevation should be referenced to a datum (e.g. Elevation = 100 feet) for convenience in design and to establish a consistent elevation relationship between design disciplines. Reference point or reference coordinates are necessary for locating the plant relative to its surroundings. • Safety considerations: Plant equipment should be located far enough from public areas and thoroughfares to minimize risk to or from the public.

90

3 Basic Engineering

Fig. 3.20 Typical Master plot plan (Indicative only—not to scale)

Fire and safety equipment should be located to maximize accessibility and minimize exposure to fires, explosions or releases. Plant and equipment layout should ensure that a safe means of egress is provided for personnel evacuation in the event of an emergency. Egress routes should be continuous (not necessarily in a straight line), unobstructed, clearly marked and lighted. Equipment containing hazardous materials should be grouped within a paved and curbed area that is drained or transferred to waste neutralizing • Offsites and process unit equipment spacing requirements should be following the local standard. • Existing highway and railroad facility: Offices and warehouses must be readily accessible from the main highway and warehouses, storage yards and product loading areas must be convenient to the nearest railroad. If the plant is to have

3.2 Process Drawings

91

harbour—facilities, product storage should be in the vicinity of docks, though far enough to minimize the risk of the dock fire. Existing or possible future railroads and highways adjacent to the plant must be known to plan rail sidings (siding is a railroad truck used by one train to pass or meet another train. A siding is laid adjacent to a through track) and access roads within the plant. Railroad spurs (spur is a railroad track on which container cars are left for loading, unloading or storage) and roadways of the correct capacity and at the right location should be provided for in a traffic study and overall master track and road plan of the plant area. • Location of legal boundaries: Shape of land and usage of adjacent lands are also important. Equipment layout inside the plant are affected by the nature of usage of the adjacent land. • Location of public facilities nearby: Location of local water supply header, river or well, from which water would be supplied to the plant, has an important role both for the master and unit plot plans. Public waste disposal facility or power supply lines should also be located because of the similar importance. • Types of process units: If the product from one process unit is fed directly to another, the units should be adjacent to reduce piping and pumping costs. The operation, maintenance and utility distribution are often simplified by locating similar units in one section of the plant. • Plant Services: The power plant, shops, warehouse, cafeteria and change house should be located not only for maximum efficiency and convenience but also for minimum interference with the process operation. • Future expansion: Expansion of the plant must always be kept in mind. Options between multiplying the number of units or increasing the size of existing units are to be judged carefully. Any future plot needs, as required by the owner for process and supporting equipment, should be considered well in advance. Unit plot plan—considerations for development Unit plot plans are generally called equipment layouts. It is needed in any of the following situations • • • •

Planning a completely new facility. Expanding or relocating an existing facility. Rearrangement of existing layout. Minor modifications in the present layout. Depending upon the focus of layout design, the basic types of the layouts are:

Product or line layout—This type of layout is generally used for one or two products of constant demand. The raw material enters at one end of the line and goes from one operation to another rapidly with minimum storage and material handling. The equipment are arranged in order of their appearance in the production process. The prerequisite of such a process is an uninterrupted supply of materials.

92

3 Basic Engineering

Some of the major advantages of this type of layout are reduction in material handling, better utilization and specialization of labour and effective supervision. The major problem is to decide the cycle time and the sub-division of work which should be properly balanced. Process or functional layout—This layout is suitable for low volumes of batch production when the product is not standardized. It is also economical when flexibility is the basic system requirement. Here, the machines are not arranged according to the sequence of operations but according to the nature or type of the operations. Cellular or group layout—It is a special type of functional layout in which the facilities are grouped into cells that can perform a similar type of function for a group of products. ‘Fixed Position’ Layout—This is suitable for producing single, large, high-cost components or products. Here, the product is static. Labour, tools and equipment come to the worksite. Flow patterns in a layout: According to the pattern of flow, the layout plan arranges the work area for each operation or process to have a smooth flow from the entry of raw materials to the shipping of the products along through the production/service facilities. The basic types of flow patterns that are employed in designing the layout are as follows (Fig. 3.21a–e) [7]. The points to consider during preparation of a Unit Plot plan are as below. A sample unit plot plan is given in Fig. 3.22. Safety first: Sufficient space has to be provided for human escape and cleaning or firefighting with all equipment. Process flow: The process flow diagram, equipment list and additional process information show how the pieces of process equipment are interconnected and provide special required elevation requirements. The flow sequence and function of each piece of equipment should be thoroughly understood so that its arrangement in the plot can be functional. Elevation of equipment: Usually the elevation of equipment is dictated by pump suction requirements or other process requirements. Elevation of equipment is always costly and should be done only if necessary for the satisfactory operation of the process. Plant terrain should be utilized for elevated equipment. Equipment maintenance: Reserved space should be provided for routine maintenance activities (e.g. filter cartridge removal, catalyst handling, tray removal, relief valve removal, etc.) Access and clearances for operation and maintenance on proprietary equipment or parts of proprietary equipment should be following the equipment manufacturer’s standards. Clearance should be provided for tube bundle removal and channel or bonnet removal.

3.2 Process Drawings

93

Fig. 3.21 a–e Basic types of layouts. a I-flow—Separate receiving and shipping areas at the two sides of a plant. When straight-line flow chart is to be accommodated. b L-flow—When a straight-line flow chart is mandatory and the receiving area is at 90º with the shipping area. c U-flow—Very common as a central loading and unloading facility is provided. d S-flow—When the production line is long and zigzagging on the production floor is required for accommodating all equipment. e O-flow—When it is desired to terminate the flow near where it is originated

Access to air-cooled exchangers should be provided for cooler removal, cooler maintenance, fan motor maintenance and header box access. Equipment spacing: Standard spacing rules are generally provided for equipment layouts. For example, access of approximately 3ft should be provided around and between pumps. Vertical pumps should have appropriate overhead clearances for the removal of drivers, shafts, impellers and other parts. Storage tank and containment: Adequate space should be provided for containment dikes to avoid unnecessary expenses for dike construction. Typically an economically and properly constructed earthen dike is 2.4 m (8 ft) wide at the top and has side slopes of three horizontal to one vertical. Therefore, the plot plan space for an earthen dike is six times the height required plus 2.4 m (8 ft).

94

3 Basic Engineering

Fig. 3.22 Typical unit plot plan (indicative only)

Equipment erection/installation: The process of assembly, erection and installation of each piece of equipment should be thoroughly studied. The provision of space for the erection of equipment should be provided. Equipment connections: Connections of particular equipment should be considered during its placement. Utility distribution: Efficient distribution of utility and connection of particular equipment to the utility distribution facility has to be taken care of.

3.2 Process Drawings

3.2.4.2

95

Methods for Developing Plot Plans

Several techniques were used for Master and unit plot plans. Though nowadays the most familiar method is the software-based method with the help of computers. A few old techniques have been discussed below [6]. Two-dimensional layout—Scale reduced cardboard cut-outs of the outlines of equipment are used on graph papers to make trial plot plans. Three-dimensional layout—Scale reduced simple block models (Fig. 3.23) of rectangular or cylindrical shapes according to the respective equipment can be used on graph papers to study alternative layouts in plan and elevation. Both cut-outs and blocks can be used for unit and master plot plans. Once the major pieces or sections are decided, then piping, structural and other arrangements can be designed. Scale model method—Three dimensional scaled models are made to detail out not only the position of the equipment but also the piping and instrumentation general arrangement (GA) drawings with elevation and isometrics. For this, the model has to be sufficiently large. After finalizing the layout and GA drawings, such models are utilized for the construction and training of the operators. Computer-aided method—Computer-aided design tools (like Auto-CAD) are now used for plant layout studies. There are advantages over the physical models due to the ease of revision and transfer of the latter by all parties involved. Interference between piping and other components is easily detectable.

Fig. 3.23 Typical 3D model of a chemical plant [8]

96

3 Basic Engineering

3.3 Design and Selection of Equipment The selection and design of equipment are very crucial for the execution of a project. There are a good number of books and materials on the design of various equipment. However, the selection criteria and a few rules of thumb are available based on the experiences of professionals. Though the thumb rules or heuristics may vary a little in different countries and industries, a few generally accepted guidelines for the selection and design of some common equipment are summarized below [9]. However, books and software should be referred to for detailed design. Pumps The selection of a pump for a specific service requires knowledge of the physical properties of the liquid handled as well as the suction and discharge heads. Different types of pumps commonly used in process plants are centrifugal, positive displacement reciprocating or rotary, diaphragm and jet pumps. Different pumps work best in different operating ranges of capacities and heads. For this, curves are available in standard books of design. Generally, a normal pump suction head should be maintained at 1–6 m of the head of the liquid it is handling. Table 3.10 summarizes the applicability of various pumps for various purposes. Mixer and agitator A mixer is a subset of agitation devices. Mixers blend separate fluids to form a homogeneous mixture whereas agitators as a class suspend solid particles in the Table 3.10 Operating range and applicability of pumps Type

Max Pr (kPa)

Max P/stage (kPa)

Approx capacity (m3 /h)

Efficiency (%)

Remarks

Centrifugal (radial)

48,000

2000

10

40–80

Simple and suitable for low viscosity

Screw

20,000

2000

0.1

40–70

Wide viscosity range

100,000

15,000

0.03

50–90

Suitable for low capacity high head service

35,000

7000

0.006

20–50

Can handle viscous, corrosive, abrasive, toxic, and flammable liquids

Reciprocating Piston

Diaphragm

3.3 Design and Selection of Equipment

97

fluid, disperse gases, emulsify liquids and enhances heat and mass transfer. The type of mixing device depends much on the characteristics of the feed. Propeller or turbine mixers are the most widely used mechanical agitators for low to medium viscosity fluids. Turbine impellers are larger and slower than propeller type agitators. For highly viscous materials, kneaders, extruders, muller mixers, rotor mixers and mixing rolls are used. Reactors The selection of the best reactors for a given process is subject to several considerations. Some of the considerations are as below: • • • • •

Temperature and pressure of the reaction. Whether the process is batch or continuous. Physical nature of catalysts, if any. Limitation of reactor. Cost consideration. Moreover, some general rules of thumb for the selection of the reactor are:

• Continuous stirred tank reactors (CSTRs) are usually used for slow, liquid phase or slurry reactions. • For small scale, slow reactions, which may foul the reactor and which require intensive monitoring or control, batch reactors are suitable. • For conversion up to 95%, five or more CSTRs in series approaches that of a plug flow reactor (PFR) in performance. • In a catalytic reactor, if the catalyst particles are ~0.003 m in size, a fixed bed reactor is suitable. For a catalyst of ~0.001 m in size, a slurry reactor is effective whereas, with ~0.0001 m catalysts, a fluidized bed reactor works the best. Optimum proportions of stirred tank reactors at lower pressure range are with a liquid level equal to the tank diameter. • Ideal CSTR behaviour is approached when the mean residence time is 5–10 times the time required for homogenization. Heat exchangers The selection process of a heat exchanger normally includes, but not limited to, the following factors: • • • • •

Thermal and hydraulic requirements Material suitability for service Cost Safety aspects Operational conditions like fouling. The working ranges and applicability are tabulated below (Table 3.11). A few rules of thumb for design are:

98

3 Basic Engineering

Table 3.11 Operating range and applicability of heat exchangers Type

Max pr range (MPa)

Temperature range (°C)

Normal area range (m2 )

Fluid velocity (m/s)

Remarks

Double pipe

30

−100 to 600

0.25–20

Liq 2–3 Gas 10–20

Small-scale; limited by material

Shell and tube

30

−200 to 600

3–1000

Liq Shell 1–2 Tube 2–3 Gas Shell 5–10 Tube10-20

Many types are available. Adaptable and flexible

Scraped wall

0.11

Up to 200

2–20

Liq Hot 1–2 Cold 1–2

For crystallization. Liquids solidify on the hot surface

Gasketed plate

0.1–2.5

−25 to 175

1–2500

Liq 1–2 Gas 5–10

Limited by gasket material Used in food or beverage service; modular form

Spiral plate 2

Up to 300

10–200

Liq 1–2 Gas 5–10

For corrosive and viscous liquid

Spiral tube

350

1–50

Liq 2–3 Gas 5–10

Low maintenance

50

• The minimum temperature approach is 10–25 °C with ambient coolants and 5 °C or less with refrigerants. • Pressure drops are generally around 10 kPa for boiling conditions and 20–60 kPa for other services • Double pipe heat exchangers are suitable at duties requiring 10–20 m2 heat transfer area • Stainless steel plate and frame exchangers are 25–50% cheaper than the corresponding shell and tube exchanger for the same duty. • Compact plate and fin exchangers provide about 3–4 times more heat transfer area per unit volume compared to shell and tube exchangers Separation process and equipment In case of separation of the desired substance from the others, first, the process should be selected. Once the appropriate separation process is identified, a piece of suitable equipment should be selected and designed. Selection of the best separation process from some possible alternatives depend mainly on the following factors The property based on which the separation would be accomplished.

3.3 Design and Selection of Equipment

99

Characteristics of the particular separation process. Flow rate and concentration of the key component to be separated, in the feed. A few examples of the mass transfer-based separation processes and the property difference based on which they are established are tabulated below (Table 3.12). In addition, physical or mechanical separation based on the difference of particle size or density, like centrifugation, decantation or scrubbing is also available. Plate versus packed columns Packed columns have smaller pressure drop per theoretical stage and so suitable for vacuum application. It has a smaller liquid residence time and so useful for handling heat-sensitive liquids. It is a natural choice for corrosive service also since ceramic packing can be used here. Foaming is less in a packed tower. A packed column is less costly than a tray column since the latter contains much more fabricated internals. The packed column can be operated at a low gas flow rate whereas, at low gas flow rate, tray columns can weep. A tray column is necessary when different fractions have to be withdrawn from different trays. This stage-wise arrangement is also suitable for removing heat from the liquid. Tray column is suitable for dirty liquid or slurry whereas packing of the packed column may be choked or clogged and cleaning of packing is difficult compared to cleaning trays. For the design of plate columns, the following points may be considered: Distillation is the cheapest method of separating liquid mixtures. For multicomponent mixtures, the easiest separation should be performed first and the most difficult one should be performed at last. The optimum reflux ratio is generally between 1.2 and 1.25. Tray spacings are generally assumed as 0.6–0.8 m. Pressure drop per tray is generally assumed as 0.75 kPa. For separating light hydrocarbons and aqueous solutions, tray efficiencies are assumed to be 60–90% whereas, for gas absorption and stripping, it is about 10–20%. For separation of homogeneous mixtures, Table 3.13 summarizes the operating range and applicability of various separation processes. The drying of wet solids is a complex process involving various forms of heat and mass transfer. Types of drying equipment are also different in configuration and operation. The key factor of selecting a dryer is therefore the nature of the wet Table 3.12 Separation processes and corresponding governing properties

Separation process

Property

Distillation Stripping Drying

The difference in vapour pressure

Crystallization Leaching Absorption

Difference insolubility

Adsorption Solvent extraction

The difference in the distribution coefficient

100

3 Basic Engineering

Table 3.13 Operating range and applicability of separation processes Separation process

Feed condition and Separating mass fraction of key principle/agent component

Normal feed capacity (kg/s)

Remarks

Distillation

Liquid–vapour 0.1–0.95

Combined heat and mass transfer

0.01–100

Requires thermal stability and difference in volatility

Absorption

Vapour 0.1–0.95

Liquid absorbent/solubility

10–4 –50

For recovery of soluble gas

Stripping

Liquid 10–3 –0.75

Stripping vapour/ Vapour pressure

10–4 –50

For removal of a volatile component from the liquid

Extraction

Liquid

Liquid solvent distribution co-efficient

10–4 –50

Flexible and scalable

Adsorption

Vapour or liquid Solid 2 × 10–3 to 2 × 10–1 adsorbent/Relative affinity

10–4 –30

Good for gas purification. High sensitivity for low concentration solutes

solid that should be dried, product quality required and recovery of solvent and dust. The cost factor is however common to all equipment whatsoever. Following are the general guidelines (Table 3.14) [9] for drying equipment selection. Table 3.14 Different types of dryers—a brief introduction Type of dryer

Nature of solid dried

Advantages

Limitations

Continuous tunnel dryer

Sludges, pastes, large solids, thermally sensitive materials

Simple and flexible

Labour intensive, not suitable for dusty materials

Rotary dryer

Fine, granular or fibrous materials

High capacity, flexible Unsuitable for slurry, and efficient pastes or heat-sensitive materials

Vacuum dryer

Fine flowing granular, fibrous or powdery solids

Handles non-sticking heat-sensitive materials

Unsuitable for gummy pastes or slurries, costly operation

Spray dryer

Slurry or solution

Excellent for heat-sensitive materials, light product—very small time required

The liquid must be sprayable

High capacity, low processing time

Limited by hot gas used

Pneumatic conveyor Very fine free-flowing solids

3.3 Design and Selection of Equipment

101

Rotary dryers operate with superficial air velocity of 1.5–3 m/s; if the material is coarse, it may increase up to 10 m/s. Residence time can vary between 5 and 90 min. 85% free cross-section is assumed with 7–8% solid hold up. In fluidized bed dryers, gas velocities of double the fluidization velocity are recommended. In spray dryers, surface moisture is removed in 5 s and most drying is complete within a minute.

3.4 Safety, Hazard and Environmental Considerations in Design Various potential hazards in a plant were identified in Chap. 2. Good engineering practice can prevent the hazards to cause an accident. Regular maintenance, personal protection equipment, regular health check-up of the workers are some of the good engineering practices. However, good engineering practice is part of plant operation [10]. Before that, some design features should be added to the plant design itself. Some of such design aspects are summarized in Table 3.15.

3.5 HAZOP Analysis A hazard and operability (HAZOP) analysis is a systematic procedure for critical examination of the operability of a process. During the study of the operability of a process, the HAZOP study indicates potential hazards that can arise if there is deviation or departure from the intended design conditions. It also suggests the action be taken if those deviations are not within acceptable limits. It can be used to make a preliminary examination of the design at the flow sheet stage. It may again be performed at a later stage of detailed engineering when a full process description, final flow sheets, P&I diagram and equipment details are available. An ‘as built’ HAZOP is often carried out after installation of the plant before commissioning. HAZOP study is generally done by a group or committee of experienced people. A HAZOP study requires a big investment in terms of time, money and effort. An important aspect of the HAZOP study is to break down the whole process into several small parts or units. Each unit is intended for a particular process duty. Now, one should consider all possible ways by which this intention is not fulfilled. This departure from intention is deviation. HAZOP approach is applied to each process unit and every process pipeline into and out of a unit [10]. There are a few guidewords based on which HAZOP analysis is generally carried out. The guide words and their explanations are given in Table 3.16. In addition, a few other words may be used in special ways. INTENTION:

It defines how the particular part or unit of the process was intended to function by the designer.

102

3 Basic Engineering

Table 3.15 Influence of safety and hazard issues on design Hazard

Design strategy/aspects for preventing accidents

Fire and explosion

• Inerting and purging This is a process of adding an inert gas to a combustible mixture to take the mixture out of the flammability range • Control of static electricity Static electricity can be controlled by suitable earthing, relaxation or dip-pipe devices • Ventilation Local and dilution ventilation systems are introduced. An open-air plant is preferred if possible • Explosion-proof equipment and instrument Depending upon the nature and degree of the process hazard, there are several classes, groups and divisions of explosion-proof equipment to be chosen for a particular application • Sprinkler and deluge systems These water spraying techniques are used to contain the fire in a particular area • On-line analysers Analysers with alarms for flammable gases and vapours with high accuracy should be installed in the potentially hazardous area as an alert • Isolation valves Different parts of a plant should be isolated so that flammable matter cannot immigrate from one region to another and fire hazard is contained • Proper equipment lay-out Provision of shower and wash, escape routes should be kept in layout. The storage area for the organic liquid should be isolated from railroad and flare. Many other safety aspects are there • Adequate water supply Sufficient water supply should be ensured in the plant area so that in case of emergency, there is no water shortage for fire-fighting • Fail-safe design for control valves Wherever possible, fail-safe designs are to be provided for valves so that no danger is posed if it fails

Toxicity

• Substitution If possible, use of less toxic raw materials for process • Containment To use robust equipment design to avoid leaks • Ventilation Sufficient ventilation for dilution of gas • Disposal Piped vent, flare and relief discharge line for safe disposal of toxic hazardous gas • Emergency equipment Respirator, eyewash and shower station in plant (continued)

3.5 HAZOP Analysis

103

Table 3.15 (continued) Hazard

Design strategy/aspects for preventing accidents

Overpressure and vacuum • Relief devices in equipment and pipelines The relief device comprises a relief valve, discharge pipeline, knockout drum and flare. Relief valves may be spring-loaded, bellowed and rupture disk All vessels where pressure increase can occur should be provided with relief valves. Blocked in sections and discharge line of positive displacement pumps and compressors need relief valves. Steam jackets are also provided with relief valves. Rupture disks are used alone or in combination with spring-loaded valves • Vent piping and flare Containment of the pressure relief discharges is necessary for venting flammable or toxic gas. Vent piping for toxic gases should end up in a scrubber whereas the same for flammable gases should be sent to a high flare. Ideally, the venting system must be capable of venting at the same rate as the vapour being generated • Vacuum breakers Analogous to the safety relief valves in high-pressure vessels, there are valves that open to the atmosphere when the internal pressure of a vessel drops below atmospheric pressure. This valve acts as vacuum breaker Temperature deviation

• • • • •

Noise

• Control at source The source of noise can be modified by acoustic treatment of machine surfaces or by design changes • Control at the path of propagation Path of noise transmission can be modified by containing the source inside a sound-proof enclosure, by constructing a noise barrier or by providing sound-absorbing surfaces along the path

DEVIATION: CAUSES: CONSEQUENCES: HAZARDS: ACTION:

High-temperature alarm Emergency cooling Intrinsically safe heating system Fire-safe structural material Heating system for abnormally low ambient temperature

Lists out the possible departures from the intention of the designer. Reasons why and how the deviation from intention can occur. The results that occur from the occurrence of a meaningful deviation. Consequences that can cause loss, damage or injury are termed hazards. The remedial measures of the consequences or hazards.

For referring to time, SOONER THAN and LATER THAN are also used. Only the deviations leading to action and/or consequences to be alert of are only recorded. The investigation proceeds through all the process units until the entire process is complete. In a process, each piece of equipment and line are analyzed. After completion of a particular line and vessel, it is marked in the P&ID.

104

3 Basic Engineering

Table 3.16 Guide words and explanations for HAZOP analysis Guide words

Explanations

NO or NOT

Complete negation of the intention; can be used for pressure, temperature, flow, level or any other process parameter

MORE

Quantitative increase of the value of a process parameter

LESS

Quantitative decrease of the value of a process parameter

AS WELL AS

Qualitative increase; stands for something in addition to the design intention. For example, impurities, side reactions etc

PART OF

The qualitative decrease is used when part of the design intention is fulfilled or realized instead of the full aim. For example, a missing component

REVERSE

A logical opposite to the intention of design; it stands for indicating the effect opposite to the intention. For example, a stream is getting cold instead of being heated in a heater

OTHER THAN Complete substitution; it covers all possible situations other than that intended

Since HAZOP is tedious and time-consuming, sometimes formal and informal Safety Reviews are used for immediate application. HAZOP does not indicate the risk and its severity. Let us take a simple example of a reactor where an exothermic reaction is being carried out. A cooling coil is inserted within the reaction mixture to take out the generated heat. So the INTENTION of the design was to cool the reaction mixture up to a desired temperature. Now if we have to make a HAZOP study of this process, then, first of all, we have to consider two DEVIATIONs from the INTENDED design. 1. 2.

NO or LESS coolant flow through the coil. MORE coolant flow through the coil.

Both the cases are considered as DEVIATIONS from the INTENDED DESIGN and both are undesirable. Hence, one should look into the POSSIBLE REASONS and CONSEQUENCES of such DEVIATIONS and propose suitable ACTION to manage the consequences from such deviations. For the first deviation, the POSSIBLE CAUSES may be: • • • •

Failure of control valve closure mechanism. Plugging of coolant line. Failure of cooling water service header. Failure of air-pressure to the pneumatic control valve. The apprehended CONSEQUENCES are as below:

• Increase in temperature of the reaction mixture inside reactor. • Possible thermal run away from the reactor. The remediation ACTIONS to be taken are: • Provision of back-up control valve or manual bypass line.

3.5 HAZOP Analysis

• • • • • •

105

Installation of filter or strainer to prevent valve clogging. Provision of back-up source for cooling water. Provision of fail-open control valve. Installation of cooling water flow meter with low-flow alarm. Installing high-temperature alarm in the reactor. Provision of emergency shutdown after high-temperature alarm. For the second deviation, POSSIBLE CAUSES may be

• • • • •

Failure of control valve to properly control the coolant flow rate. Failure of the controller. The probable consequences are: Cooling of reaction mixture below required temperature. No/less reaction and build-up of reactants. ACTIONS may be

• • • •

Provision of back-up control valve or manual bypass line. Installation of cooling water flow meter with high-flow alarm. Installing low temperature alarm in the reactor. To make the operators aware to control manually if required.

In this way, the HAZOP analysis is done considering all the worst scenarios of a particular design or operation. Summary A few important topics of the basic engineering of a process plant are enunciated. Development of process drawings such as flow diagrams, piping and instrumentation diagrams and equipment layouts are described stepwise with standard symbols indicated. In course of developing P&ID, approximate methods of sizing pipelines are described. Worked out examples are there to explain and demonstrate the procedure. Standard engineering practices and rules of thumb on the design and selection of common process equipment are suggested. The design of a process plant is often influenced by environmental and safety considerations. Design features generated out of the above issues are included and a summary of the HAZOP analysis is given for a preliminary idea of the process. Exercise 1. 2. 3.

4. 5.

What is Basic Engineering for a process plant? Why it is so called? Distinguish between (i) a block diagram and a PFD (ii) a PFD and a P&ID. What is meant by the plot plan of a process plant? Why plot plans are required before designing a process plant? How many types of plot plans are there? Define all of them. What are the factors that should be considered for preparing a Master Plot plan? Name a few items that must be shown in a Master Plot Plan. What are the different flow patterns that are followed for placing equipment in a unit plot plan? For each pattern, indicate the criteria for its selection.

106

6. 7. 8.

3 Basic Engineering

What are the considerations for the construction of a unit plot plan? Justify each consideration. How many types of P&IDs are there? Mention the use and application for each type. Compare the following types of P&IDs: a. b. c.

9.

Draw the symbols for the following: a. b. c. d. e.

10. 11.

Utility P&ID and Utility distribution P&ID. Auxiliary P&ID and special control P&ID. Utility distribution P&ID and Interconnecting line P&ID.

Steam ejector. Compressor. Control valve assembly. Instrument air line. Jacketed reactor with mixer and motor.

Why safety hazards and environmental issues should be considered under Basic Engineering for the construction of a process plant? Explain with examples. A liquid raw material for a process is being pumped into the tube side of a shell-and-tube heat exchanger at a specific flow rate for heating. The heating medium is steam passing through the shell side. According to the process requirement, the heat exchanger inlet flow rate and the outlet temperature of the heated liquid should be controlled. The flow rate of steam is controlled for controlling the heated liquid temperature.

Draw a control and instrumentation schematic around the heat exchanger based on the above process description. Use usual symbols for the equipment, instruments and control loops. Justify the use of each instrument and explain the control logic.

References 1. Technologies, I. D. C. (2003). Practical fundamentals of chemical engineering; Pty 2003. Ver, 1, 2. 2. Sinnot, R., & Towler, G. (2009). Chemical engineering design (Vol., 65th edn.). Coulson and Richardson’s Chemical Engineering Series. Elsevier. 3. IS 3232: 1999: Indian Standard: Recommendations on graphical symbols for process flow diagrams, piping and instrumentation diagrams (2nd Revision). 4. IS 2379:1990: Indian Standard: Pipeline Identification Colour Code (First Revision). 5. Process Industry Practices, (June 2013); Process Unit and Offsites Layout Guide. 6. Food Processing Plant Design and Layout; Lesson 7. Plant Layout. Retrieved November 2020, from ecoursesonline.iasri.res.in/mod/page/view.php?id=1133. 7. www.yourarticlelibrary.com/industries/plant-layout/four-main-types-of-plant-layout/34604. Accessed in June 2020. 8. https://3dexport.com/3dmodel-chemical-plant-156395.htm. Accessed in June 2020

References

107

9. Peter, M., Timmerhaus, K. D., & West, R.E. (2004). Plant design and economics for chemical engineers (Fifth International Edition). McGraw-Hill. 10. Crowl, D. A., & Louvar J. F. (1990). Chemical process safety: Fundamentals with applications. Prentice-Hall International Series in the Physical and Chemical Engineering Sciences.

Chapter 4

Detailed Engineering

Based on the process drawings and documents generated via Basic Engineering activities, detailing is done by Detailed Engineering activities. Fundamentally detailed engineering involves co-ordination between all branches of engineering. Outcomes of interdisciplinary engineering are represented in the form of several documents like specification sheets, equipment and line schedule. Engineering change notice for small revisions in design or operation and maintenance manual after construction is also under the purview of detailed engineering; all these topics are covered here. Learning Objectives • To understand how to proceed with the actual construction of a plant using basic engineering data. • To prepare specification sheets, operation manuals and other documents for the construction of a process plant.

4.1 Preamble Detailed engineering of a chemical process plant includes the exaction of the essential information from the basic engineering package and providing drawings and documents required for the construction and operation of a plant, based on such information. The drawing and documents include the same for all branches of engineering, though in this chapter we will discuss only some of the drawings and documents generated by the Process sub-division.

© The Author(s), under exclusive license to Springer Nature Singapore Pte Ltd. 2022 S. Chakrabarti, Project Engineering Primer for Chemical Engineers, https://doi.org/10.1007/978-981-19-0660-2_4

109

110

4 Detailed Engineering

4.2 Drawing and Documents for a Project The Project Documentation will include but is not limited to the following [1]: 1.

General Correspondence within the design group and with • • • • •

2.

Government Departments. Equipment/Inst. Vendors. Clients. Site Personnel. Other Departments.

Calculation Sheets • Design Calculations. • Costing.

3.

Drawings • • • • • • • • •

4.

Specification (Data) Sheets • • • •

5.

Equipment. Instrument. Piping. Valves.

Lists and schedules • • • • • •

6. 7. 8. 9. 10.

Flow Sheets or PFDS. P & IDS—Process and Utilities. Layouts—Equipment and Piping. Site Plans. Design Sketches. Equipment general arrangement (GA) Drawings. Vendor Drawings. Civil and structural drawings. Electrical and Instrumentation Layout.

Equipment list. Instrument list. Line list. Valve list. Piping Bill of Quantities (BOQ). Drawing and document list.

Progress Reports and Schedules. Engineering Change Reports. Purchase Orders. Approvals. Manuals.

4.2 Drawing and Documents for a Project

111

• Operating manual from the vendors. • Operation, maintenance and start-up manual. Among the above drawings and documents, updating of the PFDs and P&IDs received in the basic engineering package should be done during the detailed engineering from time to time based on the engineering changes and or vendor’s inputs. Though civil, electrical, instrumentation and piping drawings and documents are not under the purview of a process or chemical engineer, a project engineer should ensure that the process requirements are fulfilled on such documents. It may once again be mentioned that many of the design documents are revised several times till the construction is complete by several parties involved. Hence it is not possible to specify any particular document to be confined to any particular stage of the project.

4.3 Specification or Data Sheets for Equipment Specification summarizes the important essential features of equipment, instrument, material or service. Specification may be of the buyers or the sellers. A buyer should specify what is their requirement so that the seller can match any of their product with it. Similarly, a seller should describe the features of their product so that the buyer can get their requirement. Here, we would discuss generally the specifications of the equipment or instruments required for the installation of a process plant from the buyers’ point of view. Among the group of equipment, some are to be fabricated and some are purchased as such. Storage tanks, pressure vessels, reactors and heat exchangers are fabricated whereas pumps, blowers and agitators are directly bought out. Valves, pipes, fittings and instruments are generally bought out. As mentioned before, an approximate sizing of the fabricated items and a preliminary selection of the bought-out items are generally done during basic engineering. Hence it is assumed that the information regarding the particular equipment is available before a specification or data sheet is being prepared. For fabricated items, the duty of a process (Chemical) engineer is to specify the process design data and the code by which the design should be done. Mechanical details will be provided by the fabricator.

4.3.1 Specification for an Atmospheric Storage Tank It may be clear from the specification sheet (Fig. 4.1) that the project engineer should fill up the identification, process data, part of the design data, materials and construction as well as the size and service of the nozzles. During filling of materials data,

112

Fig. 4.1 Sample specification sheet for storage tank

4 Detailed Engineering

4.3 Specification or Data Sheets for Equipment

113

s/he should refer to the material specification for the particular project. S/he should also specify the code according to the basic engineering or pre-engineering data. The rest of the spaces would be filled up by the fabricator or a mechanical engineer. This specification sheet will then go to the client for approval, based on which general arrangement (GA) and fabrication drawings will be prepared.

4.3.2 Specification for a Shell and Tube Heat Exchanger A shell and tube heat exchanger may either be fabricated or can be a bought-out item. In either case, the process group (Chemical Engineers) should undertake the process design and fill up the process data and part of the constructional features (Fig. 4.2). The remaining data should be filled up by the vendor (manufacturer) or the mechanical engineer who is doing the mechanical design and fabrication. In all cases, the material and other details should be decided by both the process and the mechanical engineers. TEMA is the most conventional design standard for shell and tube exchangers [2].

4.3.3 Specification for a Centrifugal Pump From the data sheet (Fig. 4.3), it may be understood that the process data or operating data is to be filled up by the Process (Chemical) Engineer. Technical data are generally filled up by the pump manufacturer and to be checked by mechanical and electrical engineers. A Project Engineer should co-ordinate a review of this part. Materials are also specified by the manufacturer but should be checked for suitability by the chemical/ project engineer concerning the materials specified in the basic engineering package of the particular project. Centrifugal pumps are selected from the characteristic and performance curve of a particular vendor by them, and accordingly, they specify some of the data in the specification sheet. Available NPSH as well as the suction and discharge pressures are to be specified by the chemical engineer. For this, s/he should refer to the basic engineering package—the PFD, P&ID and the plot plan. From PFD, the flow rate and the properties of fluid will be obtained. From P&ID, the valves, fittings, equipment and instruments through which the fluid should be drawn or discharged are determined. All those contribute to the resistance to flow and the pump head should overcome it. From equipment layout (or plot plan), one should know about the route—number of bends required and difference in height (so that hydrostatic head should be provided) between various points. Considering all the factors, pressures and heads of a pump have to be specified. A few problems are worked out below to explain the calculations.

114

4 Detailed Engineering

Fig. 4.2 Sample specification sheet for shell and tube heat exchanger

4.4 Worked Out Examples on Pump Heads Example 4.1: Calculation of NPSH A liquified gas was being unloaded from a tanker to a storage vessel. Given the following information, calculate the NPSH (available) at the inlet of the pump:

4.4 Worked Out Examples on Pump Heads

115

Fig. 4.3 Sample specification sheet for centrifugal pump

Flow rate: 15,000 kg/h; Density: 1280 kg/m3 ; Vapour pressure of the liquified gas at the maximum prevailing temperature: 684 kPa; viscosity of the liquified gas: 0.35 centipoises. The pressure in the tanker is 700 kPa. The total length of the pipeline from the tanker to the storage tank is 50 m. The pump is situated 10 m below the outlet of the tanker. 50 mm ID commercial steel pipe is used. Miscellaneous frictional loss in the pump suction line is considered as 1000 equivalent diameter. The friction factor is 0.00225. Solution: A schematic sketch is as follows (Fig. 4.4): NPSH(a) = Liquid source pressure + Static head − Frictional loss − Vapour pressure. 700 = 55.80m. Liquid source pressure = Pressure in the tanker is 700 kPa 1.280×9.8 Static head = 10m.

116

4 Detailed Engineering

Fig. 4.4 Figure for Problem 4.1

Frictional loss is calculated as follows: Total length of pipe = straight length + equivalent length for frictional loss (1000 times diameter) = 50 m + 1000 × 0.05 m = 100 m. Cross - sectional area = A = Velocity :

2 π  50 × 10−3 = 1.96 × 10−3 m2 . 4

15000 = 1.66 m/s. 1280 × 3600 × 1.96 × 10−3

Now frictional loss 100 L ρu 2 1280 × (1.66)2 = 8 × 0.002255 × × Di 2 50 × 10−3 2 2 = 63630 N/m = 63.63 kPa = 5.072 m.

P f = 8 f

Vapour pressure contribution: 684 = 54.52m. 1.280 × 9.8 So NPSH = 55.80 + 10 − 5.072 − 54.52 = 6.208m.

4.4 Worked Out Examples on Pump Heads

117

Fig. 4.5 Figure for Problem 4.2

Example 4.2: Calculation of NPSH and Total Discharge Head Specify the suction and discharge heads of the pump shown in Fig. 4.5. Pressure in the source vessel is 1 bar and the destination vessel is at 2 bar. Flow rate of the service fluid is 10,000 kg/h; density at the pumping temperature: 1305 kg/m3 , viscosity: 1.0 cP; vapour pressure at pumping temperature 0.1 kPa; Friction factor = 0.0027. Maximum pressure drops and equivalent lengths for valves and fittings are as follows: Control valve = 200 kPa; Heat exchanger: 100 kPa; Orifice: 22 kPa; Le /D for gate valve is 18 and that of 90° bend is 30. Solution: Pipe Diameter Calculation: Typical velocity for liquid 2 m/s (1.5–2.5 m/s). Mass flow rate : Volumetric flow :

10000 = 2.78 kg/s. 3600

2.78 = 2.13 × 10−3 m3 /s. 1305

Cross - sectional area of pipe =

2.13 × 10−3 = 1.06 × 10−3 m2 . 2

Diameter = 37 mm → Since it is a non-standard size, the pipe diameter taken is 40 mm (1 ½ inch pipe).   The corrected cross-sectional area is now π4 40 × 10−3 = 1.26 × 10−3 m2 .

118

4 Detailed Engineering

Corrected fluid velocity is 1.7 m/s. Pressure Drop Calculation: Fluid velocity = 1.70 m/s. Frictional loss = 8 × f ×

L ρu 2 . Di 2

Per unit length (L = 1)

1 = 8 × 0.0027 × 40×10 −3 × 1305 × 2 = 1019 N/m = 1.02 kPa.

1.72 2

Maximum flow rate =20% above the average flow =1.2 times average flow rate. Since frictional loss is directly proportional to u2 , maximum frictional loss = (1.2)2 × 1.02 kPa/m = 1.5 kPa/m. Pump Suction Line Calculation: Length of straight pipe: 2.5 m–1.0 m = 1.5 m Equivalent length for fittings For 1 no. 90◦ − bend = 1 × 30 × D = 30 × 40 × 10−3 = 1.2 m. For 1 no. Gate valve = 1 × 18 × D = 1 × 18 × 40 × 10−3 = 0.7 m. Total Length = 1.5 + 1.2 + 0.7 = 3.4 m. Frictional loss at maximum flow rate = 1.5 × 3.4 = 5.1 kPa. 1305 × (1.7 × 1.2)2 ρu 2 = 2 2 × 10−3 = 2.7 kPa.

Entry loss at maximum flow rate =

Static Head = 1.5 m. ρgh = 1.5 × 9.8 × 1305 = 19198.2 = 19.2 kPa.

4.4 Worked Out Examples on Pump Heads

119

Source vessel pressure : 1 bar = 101.3 kPa. Total Suction Line Pressure in kPa = (101.3 + 19.2) − (5.1 + 2.7) = 112.7 kPa. Vapour Pressure = 0.1 kPa. NPSH = 112.7 kPa − 0.1 kPa = 112.6 kPa = 112.6 × 103 N/m2 112.6 × 1000 = 8.8 m. 1305 × 9.8 Pump Discharge Line Calculation: Straight pipe length = 4 + 5.5 + 20 + 5 + 0.5 + 1 + 6.5 + 2 = 44.5 m. 6 nos. 90◦ Bends = 6 × 30 × 40 × 10−3 = 7.2 m. 4 nos. Gate Valves = 4 × 18 × 40 × 10−3 = 2.92 m. Total equivalent pipeline length = 54.62 m. Frictional loss in the pipeline = 1.5 × 54.62 = 82 kPa. Pressure drops in Heat Exchanger

Max

200 k Pa

Control Valve

Max

100 k Pa

Orifice

Max

22 k Pa

Total pressure drop for equipment:

322 kPa

Static Head = (7.5 – 1) = 6.5 m. Static pressure = ρgh = 6.5 × 1.306 × 9.8 = 83.2 k Pa. Destination equipment Pressure = 2 bar = 202.6 k Pa. Total discharge pressure = 82 + 322 + 83.2 + 202.6 = 689.8 kPa. So, total discharge head: 53.9 m. Suction Pressure = 112.6 k Pa; Suction head: 8.8 m. Differential Pressure = 577.2 k Pa; Differential Head = 45.1 m. Example 4.3: Alternative Discharge Heads Pump discharge via two routes—specification of pump discharge head In Fig. 4.6, a centrifugal pump P-01 is pumping liquid to two tanks T-101 and T-201 via two discharge routes—Route-I and Route-II. Lengths of straight pipes in Route-I

120

4 Detailed Engineering

Fig. 4.6 Figure for Problem 4.3, [Legend: GT-gate valve, HE-heat exchanger, OR-orifice and CV-control valve]

and Route-II are 13 and 16 m, respectively. The diameter of the pipe in both the routes is 50 mm. The density of the liquid is 1000 kg/m3 . Frictional loss is 1.5 kPa/m. Pressure drop data: Control valve: 100 kPa; Heat exchanger 200 kPa; Orifice: 20 kPa. Le /D values: gate valve: 18; 90° bend: 30; T-joint: 40. (i) (ii) (iii)

What will be the total discharge pressures required to deliver fluid via each route? If NPSH available of the pump is 1.5 m, what will be the differential heads required for the pump for each route? Which differential pressure/ head and total discharge pressure will you specify for procurement of the pump?

Solution: Route-I: 1 no. Control valve: P = 100 K Pa (given). 1 no. Heat Exchanger: P = 200 K Pa (given). Static head:1.5 m (given). Static pressure: 1.5 × 1000 × 9.8 = 14,700 N/m2 = 14.7 kPa. Straight pipe length: 13 m (given). 6 nos. Gate valve:6 × 18 × D; Le = 18 × 6 × 0.05 = 5.4 m. 3 nos. 90° bend: 3 × 30 × D; Le = 3 × 30 × 0.05 = 4.5 m.

4.4 Worked Out Examples on Pump Heads

121

1 no. T -joint: 1 × 40 × D; Le = 1 × 40 × 0.05 = 2.0 m. Total equivalent length of pipes and fittings = 24.9 m. Frictional loss is given as 1.5 kPa/m. Frictional loss for the pipes and fittings: 37.35 kPa. Destination vessel is opened to atmosphere: 101.3 kPa. Therefore, the total discharge pressure required in Route-I: 100 + 200 + 14.7 + 37.35 + 101.3 = 453.35 kPa. Discharge head for Route-I: 46.26 m. Differential head for Route-I: 46.26 – 1.5 = 44.76 m. Route-II: 1 no. Orifice P = 20 kPa. 1 no. Heat exchangerP = 200 kPa. Straight pipe length:16 m(given). 4 nos. Gate valve:4 × 18 × D; Le = 18 × 4 × 0.05 = 3.6 m. 1 no. T -joint: 1 × 40 × D; Le = 1 × 40 × 0.05 = 2.0 m. Total equivalent length of pipes and fittings = 21.6 m. Frictional loss is given as 1.5 kPa/m. Frictional loss for the pipes and fittings: 32.4 kPa. Destination vessel is opened to atmosphere: 101.3 kPa. Therefore, the total discharge pressure required in Route-II: 20 + 200 + 32.4 + 101.3 = 353.7 kPa. Discharge head for Route-II: 36.09 m. Differential head for Route-II: 36.09 – 1.5 = 34.6 m. For procurement, the maximum head required should be specified. So the pump with a 46.26 m discharge head (approximately 50 m) should be purchased.

4.4.1 Pressure Drop Calculation Sheet for Pumps Sometimes, a pressure drop calculation sheet of the following format may be used (Table 4.1). A schematic sketch of the route for which pressure drop is being calculated should be provided with the calculation (Fig. 4.1 and Table 4.2). Recommended velocities are provided in Table 4.2 [3].

4.5 Specification of the Instruments and Control Valves The chemical engineer has to provide the flow rate and physical properties of the fluid flowing through the line on which a particular instrument should operate. For this, s/he should refer to the PFD. The range of operation and the accuracy required along with the requirement of any high or low alarm should also be specified by the chemical engineer (Table 4.3).

122

4 Detailed Engineering

Table 4.1 Pressure drop calculation sheet for centrifugal pump Name of the company

Pressure drop calculation sheet No

Project

Job no

Plant section

Pump no

P&ID NO

Line no

Fluid: Flow rate: Viscosity: Density: Temperature: Pressure: Vapour pressure: Line segment diameter Le /D values for

Schematic sketch Qty

Le

Qty

Le

Qty

Le

Valves: Gate valve Globe valve Check valve Diaphragm valve Fittings: 90° bend 45° elbow Expander/ Reducer Tee joint (hard) Tee joint (soft) Other equipment Control valve Orifice Heat exchanger Any other Total eqv. length for fittings Straight length Total eqv. length Pr. Drop/ 100 m Total line loss Hydrostatic head Total pressure drop Revision

Date

Prepared

Checked

Approved

4.5 Specification of the Instruments and Control Valves

123

Table 4.2 Pressure drops in pipelines Pipe size (mm)

Lateral Flow m3 /s

Header Velocity m/s

Head loss m/100 m

Flow m3 /s

Velocity m/s

Head loss m/100 m

75

0.006

1.323

4.47

0.004

0.93

2.31

100

0.013

1.539

4.29

0.009

1.08

2.22

150

0.032

1.695

3.19

0.024

1.29

1.92

200

0.057

1.759

2.48

0.041

1.27

1.36

250

0.095

1.859

2.11

0.069

1.37

1.19

300

0.151

2.076

2.1

0.114

1.56

1.23

350

0.196

2.195

2.1

0.139

1.56

1.14

400

0.284

2.411

2.09

0.208

1.80

1.16

450

0.379

2.533

1.99

0.284

1.90

1.17

500

0.379

2.03

1.17

600

0.694

2.38

1.19

700

1.199

2.64

1.11

Table 4.3 Equivalent lengths of various fittings [4]

Description

Equivalent length in pipe diameter Le/D

Fully opened gate valve

13

Fully opened globe valve

340

Non-return check valve

135

Foot valve with strainer

420

Fully opened butterfly valve

20

Three-way valve Flow through straight run

44

Flow through branch

140

90° Standard elbow

30

90° long radius elbow

20

45° standard elbow

16

Standard Tee joint Flow through straight run

20

Flow through branch

60

Close pattern return bend

50

For example, for specifying a pressure gauge, the chemical engineer has to specify the following data (Table 4.4). For the control valve, a chemical engineer should specify the following data (Table 4.5).

124 Table 4.4 Process data to specify for a pressure gauge

4 Detailed Engineering Name/tag number P&ID number/line number Related equipment number/line size Fluid flow rate Density viscosity Normal/minimum/maximum temperature Normal/minimum/maximum pressure Instrument/element type Range of pressure required Material of element

Table 4.5 Process data to specify for a pressure gauge

Name/tag number P&ID number/line number Related equipment number/line size Fluid flow rate Density viscosity Normal/minimum/maximum temperature Normal/minimum/maximum pressure Vapour pressure Controller type Range of control required Materials Instrument air details (in case of pneumatic controller) Power details (in case of electrical/electronic controller) By-pass line details

Other details are generally specified by the vendor and are checked by the instrumentation engineer. A project engineer should co-ordinate the review.

4.6 Line and Valve List or Schedule All the lines and valves should be enlisted in the line list (Table 4.6) or valve list (Table 4.7). Sometimes, they are called line and valve schedules [5]. Formats are different for different contractors, but the information is generally the same. Similarly, valves are also enlisted as per a similar format.

Line no

.

.

Sl. no

.

.

.

.

P&ID no

Table 4.6 Sample line list

.

.

Plant section

.

.

From

.

.

To

.

.

Fluid service .

.

Temp

.

.

Pr

.

.

Line material grade

.

.

Insulation

.

.

Remarks

4.6 Line and Valve List or Schedule 125

126

4 Detailed Engineering

Table 4.7 Sample valve list Sl. no

Valve no

Valve type

Valve size

Service

Connection

Line no

P&ID no

Plant section

Remarks

.

.

.

.

.

.

.

.

.

.

.

.

.

.

.

.

.

.

.

.

4.7 Equipment List Equipment are generally enlisted as per the category. Different formats are used for different items like heat exchangers, tanks, etc. For example, a sample format for a column should contain the following information: 1. 2. 3. 4. 5. 6. 7. 8. 9. 10. 11. 12.

Sl. No. Column No. Service/Name. Plant segment/P&ID no. Product—Top–Bottom. Pressure—Top–Bottom. Temperature—Top–Bottom. No. of trays/packed bed. Type of tray/packing. Material of Construction—Shell-Head-tray/packing. Vendor. Remarks.

4.8 Engineering Change Notice Sometimes, engineering change becomes necessary after the design is frozen and finalized for construction. An Engineering Change Notice (ECN) is a document generated for authorizing, recording and communicating design changes to all the stakeholders throughout the life cycle of a product. A format of ECN should contain the following information: • Identification of the portion of the drawing or document that is going to be changed. This should include the part number and name of the component and reference to drawings. • Valid reason(s) for the change. • Description of the change. This may include a drawing of the component before and after the change. • List of documents (and in industry, the departments) going to be affected by the change. All pertinent groups should be notified and all documents should be updated. • Approval of the change by the relevant authority.

4.8 Engineering Change Notice

127

Table 4.8 Typical format for engineering change notice (ECN) Name of company

Name of project

Project number

Original part/drawing/no

Revision

Part/section description

Revised part/drawing/no

Revision

Department

Description of change Reason for change Affected drawings/documents whether mark-up attached 1 2 Originated by (name)

Signature and date

Signature of HOD

Action to be taken Approval of other departments Department

Name

Signature and date

Approval of Project Managers Engineering contractor

Client/ Owner

Name Signature and date

Name

Signature and date

• Instruction about when to introduce the change—whether should be with immediate effect or at a particular time in the future. A typical format for the engineering change notice is given below (Table 4.8).

4.9 Evaluation of Quotations from Vendors—Procurement Assistance Earlier in this chapter, the preparation of the specification sheets for equipment was described. Preparation of bid documents has been introduced in Chap. 2. Enquiry for purchase of equipment should now be sent to some reputed vendors known for supplying of that particular equipment. Document for inviting quotations or the enquiry documents generally include the following: • Instruction to the bidders and scope of supply. • A very brief description of the plant with a mention where this particular equipment will be required. • Number of units required and specification sheet. • Delivery schedule and place of delivery. • Commercial terms and conditions. Vendors are generally asked to submit their credentials and clientele for similar supply and to ensure the possibility of a visit to a running unit. Commercial and legal

128

4 Detailed Engineering

documents are also asked from the vendor as their eligibility criteria. Though these are not within the look-outs of the project engineer, quotations from non-qualified vendors are not technically considered. Generally, there is a particular date for submitting the quotation. In most cases, the quotations are two-part quotations—one part is technical and the other part is commercial with the price. Sometimes, commercial terms and conditions are submitted together without price. For small value items, however, combined quotations are submitted [6]. After receiving the quotations, first technical bids are opened to see whether the offered equipment complies with the specification or requirement. If the material is found suitable, then it is qualified for opening the price bid. Sometimes, minor modifications are required. A technical negotiation or discussion is sometimes necessary for clarification of certain issues. After all the bids are technically evaluated, commercial bids and prices are opened. A commercial and price negotiation is also done, but that is generally done by the people of the purchase or commercial department, hence out of the purview of process or project engineer. A comparative statement should be made by the project engineer about the technical quality of the item to be procured. A simplified format is as below (Table 4.9), though it may vary from company to company. At least three quotations from qualified vendors are required for purchase. Sometimes, for very specialized items, single quotations are considered with the prior permission of all parties. Generally, a technically qualified quotation having the lowest price is recommended for purchase, but for special cases, a technically excellent quotation may be approved for purchase at a higher price.

4.10 Plant Commissioning and Start-up The transition from construction to operation is commissioning and start-up. Commissioning is a process by which an equipment, facility or installed plant is tested to verify whether it functions according to its design objectives and specifications. Pre-commissioning is a construction activity that verifies the functional operability of elements within the system to become ready for commissioning and startup. Generally, pre-commissioning and commissioning are difficult to separate, and these two activities lead to plant start-up. Roughly, the cost of commissioning as a percentage of total plant investment is given as follows: • 5–10% for established processes. • 10–15% for relatively new processes. • 15–20% for novel processes. There are several aspects of preparations for commissioning and start-up:

4.10 Plant Commissioning and Start-up

129

Table 4.9 Format for comparative statement for procurement Name of company

Name of project

Project number

Comparative statement for procurement of equipment Name of equipment

Equipment No

Plant/Area/P&ID no

Enquiry no. and date Description

Date of submission Specified

Vendor-I

Vendor-II

Vendor-III

Quotation no. and date Scope of supply Deviation in scope, if any Key characteristics of the item* 1 2 3 4 Deviation in technical specification, if any 1 2 3 Justification of deviation 1 2 3 Whether suitable for the purpose Approval with date Prepared by

Checked by

Approved by

*

This is item-specific. For example, for a pump, capacity, total head, NPSH, etc. should be given and for a storage tank, diameter, height and head should be there

Proper planning is necessary • Pre-commissioning and commissioning teams should be formed. • The plant should be divided into several segments according to the order of priority. • A pre-commissioning schedule has to be prepared. Personnel deployment • The right person should be placed for all parts. A separate organizational structure should be prepared for the commissioning team.

130

4 Detailed Engineering

• A dedicated and skilled labour force with necessary tools, equipment and tackles are required to be in the commissioning team. Preparation Before pre-commissioning, each piece of equipment should have its name, flow sheet number and identification number painted and/or stamped on it. All pipelines should have line numbers stamped on them. The various steps of preparation are as follows: • • • • • •

Operational checklist. Hydrostatic testing. Final inspection of vessels and instruments. Flushing of lines. Acid cleaning. Breaking in pumps and compressors (checking whether piston rings are properly seated and bearings are fully lubricated). • Drying out and boiling out of lines (generally for boilers). • Catalyst loading. • Tightness check. The operational checklist includes the following: • All lines should be checked against the flow sheet and all items should be located physically. • All valves should be checked for proper installation. Check valves should be checked against the direction of flow. • Steam tracing and insulation are to be checked. • Pumps and compressors should be checked. • Sewer and blowdown systems should be checked for operability. • Heating furnaces should be checked for burner installation, refractories, stack damper controls, etc. Facilities like the following should be installed: • • • • •

Start-up by-pass line. Purge connections. Steam-out connections. Drains and vents. Blinds.

Commissioning of utilities • Utilities such as steam, cooling water and air should be put in service as early as possible. • Once the steam is available, the system should be blown of all debris using steam. During blow off, all steam traps, control valves, ejectors and strainers should be

4.10 Plant Commissioning and Start-up

• • • •

131

removed or blinded off from the system. When the blowdown of the steam system is complete, all such valves, fittings and equipment should be restored. Cooling water lines should be flushed with cooling water. Here also, all equipment and instrument should be disconnected during flushing. In a similar manner, plant air and instrument air pipelines should be blown off thoroughly by air. Potable water lines to drinking water fountains as well as to safety shower and eyewash showers should be flushed. Each fire hydrant and turret should be flushed after removing all nozzles.

Cleaning of lines and equipment • Pipelines must be cleaned to remove foreign materials that may have left or formed during construction or piping work. Various types of cleaning methods exist— mechanical cleaning, water flushing, air blowing and chemical cleaning. • Mechanical cleaning by wire brush and cloth is required initially when the equipment is very dirty. Water flushing and air blowing are done after the initial mechanical cleaning. • For process requirements, some units need further cleaning beyond just flushing or blowing. In such cases, cleaning is done by circulating appropriate solutions through the piping and equipment. Such cleaning is usually done by specialists. • The cleaning is done by cold water, caustic soda, detergent solution, demineralized water, hot citric acid and corrosion inhibitor in sequence. After the concentration of iron is stabilized in the wash liquid, the system is drained under a nitrogen blanket. DM water is then circulated again. • During chemical cleaning, all control valves, orifice plates and other instruments are made offline. Calibration of tanks and silos • Level indicators should be calibrated against the containers. • For levels in silos, weighing, capacitive or ultrasonic measurement is used. Rotating equipment • Electrical specialists should check and verify the machines for smooth operations. • Rotational direction, bearing temperature and vibration should be checked. Drying of the system including furnace • Moisture should be removed, generally using instrument air or nitrogen. • Furnaces should be dried and the firebox should be purged by steam. Catalyst loading • Generally, two types of catalyst loading are there—low density and high density. Low-density loading is done with the help of a stationary hopper fitted with a canvas sleeve up to the bottom. As the catalyst height builds up inside the reactor, the length of the canvas sleeve is decreased.

132

4 Detailed Engineering

• A transportable kit is used for high-density loading where a rotating distributor loads the catalyst. Inspection • Licensor’s personnel should inspect all equipment for compliance. • Technical audit and plant handover • Various sections of the plant are offered to the owner for audit, for comparing with the P&ID and specifications. • After the technical audit, start-up and operator training, the plant is handed over to the owner [7, 8].

4.11 Operation and Maintenance Manual The purpose of an operating manual is not only to help the operation engineers, technicians and staff to operate the plant safely but also to present all detailed procedures for the plant start-up and shutdown in the various operation cases. This manual also helps with routine maintenance and troubleshooting as well as in safety audits from time to time. The task of writing an operation manual may be of two types—for the revamped or extended existing plant (brownfield projects) and the entirely new plant (greenfield projects). For the existing plants’ established systems, procedures and protocols are already existing—only the additional information for the modified part should only be added. This task is relatively simple compared to the same for an entirely new plant. Though the operating manual varies depending upon the plant, there are a few general guidelines for its preparation. The mandatory requirements of an O&M manual are given below. Before starting the actual write-up, terminologies like ‘Owner’, ‘Licensor’ and ‘Contractor’ should be defined. There should also be a list of abbreviations. The basis of design should also be described at the beginning of the O&M manual. Section 1 of the O&M manual should contain an overview of the plant including its basic purpose, description of the process and utilities and description of the major equipment with their respective functions, operating variables and major instrumentation and control system. Necessary drawings and documents from the suppliers of the equipment should be given here. Section 2 of the O&M manual should describe the start-up protocols including the permit to work (PTW) closure. There may be more than one sub-sections for describing (i) a commissioning checklist to indicate the readiness of the plant for starting, (ii) conditions of the utilities for the start-up and (iii) start-up conditions of the main process lines. This may include conditions of the valve and other instruments in addition to the conditions of the equipment and pipelines. Vendor drawings/documents/ instructions should be appended to this section wherever necessary.

4.11 Operation and Maintenance Manual

133

Section 3 of the O&M manual generally covers the normal operating conditions. Here also, drawings/documents/ instructions from the vendors of the boughtout items should be appended wherever necessary. Troubleshooting guidelines for various issues are sometimes included here. Section 4 of the O&M manual is for describing planned and regular shutdown protocol. Routine maintenance programmes may also be included here. Section 5 of the O&M manual includes emergency shutdown situations along with the specific type of emergency like power failure, fire and so on. Recommendations from the HAZOP study may be appended in this section. Section 6 of the O&M manual should contain alarms, trips and other precautionary measures. Section 7 of the O&M manual is on the interlocks with their logic—cause and effect. Abnormal process conditions and troubleshooting are generally compiled in section 8 of the O&M manual. Section 9 of the manual should be on the safety and environmental issues including hazards, material safety data sheet or MSDS and so on. This section should include a general description of the installed safety system in the plant. This section should describe the zone classification, availability and usage of protecting equipment, emergency evacuation process and other safety-related information. There may be a subsection for environmental issues, which should include discharge limits of various effluents specific to the particular industry, treatment of various effluent streams and local environmental laws. All flow diagrams like PFD and P&IDs should be compiled into section 10 of the O&M manual. Flow diagrams received from the vendors are also included here. In Appendices, details of auxiliary items like internals, catalysts or chemicals may be compiled. Sometimes, separate sections are provided for utilities and effluents instead of including them in the section 1. As indicated, the O&M manual should include the manuals for the licensed processes or bought-out items as its integral part. Linkage of the general operating manual with the licensed or vendor item should be made with the approval of the licensor or vendor [9, 10]. Summary This chapter defines and describes ‘Detailed Engineering’ as the subsequent step of Basic Engineering towards setting up a chemical process plant. Based on the process drawings and documents, detailed engineering generates drawings and documents necessary to transform a process into a plant. Such drawings and documents include specification sheets of various equipment/instruments, comparative statement for procurement of various items, equipment list and line list, engineering change report, operation and maintenance manual and so on. Exercise 1.

What is Detailed Engineering for the installation of a chemical process plant? Why Detailed Engineering is necessary in addition to the Basic Engineering?

134

2.

4 Detailed Engineering

What are the process data in the specification sheet of a a. b.

Storage tank? Heat exchanger?

Explain why they are considered process data. 3.

4. 5.

6. 7. 8.

9.

Name a few drawings and documents required for building up a chemical process plant. Explain their uses and mention the respective stages of execution of the project where they are generated. Why calculation of NPSH(a) and the total discharge head of a pump is necessary for a detailed engineering stage of a chemical project? What are the equivalent lengths of different valves and fittings in a pipeline? How these equivalent lengths are related to the total discharge head or NPSH(a) of a pump? Why line lists, valve lists and equipment lists are required for a chemical plant project? What is the importance of the ECN (Engineering Change Notice)? Why it should be circulated among all the design and operating groups of the project and plant? What is meant by plant commissioning? How it is different from precommissioning? What is an operational checklist, related to commissioning? Name a few parameters that should be included in the operational checklist. Explain the importance of the inclusion of each of the parameters. What is the operation and maintenance (O&M) manual for a chemical process plant? Name the sections recommended to be included in an O&M manual. Justify the inclusion of vendor drawings and documents in an O&M manual.

References 1. Sinnot, R., & Towler, G. (2009). Chemical engineering design; 5th Edn. Coulson and Richardson’s Chemical Engineering Series, Vol. 6, Elsevier (Butterworth-Heinemann). 2. Thakore, S. B., & Bhatt, B. I. (2015). Introduction to process engineering and design; 2nd Edition; McGraw Hill Education (India) Private Limited. 3. Branan, C. R. (Ed.). (2002). Rules of thumb for chemical engineers; Third Edition; Gulf Professional Publishing (an imprint of Elsevier Science). 4. Foust, A. S., Wenzel, L. A., Clump, C. W., Maus, L., & Anderson, L. B. (1980). Principles of unit operations (2nd ed.). John Wiley & Sons. 5. Ludwig E. E. (1999). Applied process design for chemical and petrochemical plants; Vol. 1–3, 3rd edition; Gulf Professional Publishing Company (Butterworth-Heinemann). 6. Pawar, D. N., & Nikam, D. K. (2017). Fundamentals of project planning and engineering; 2017; Penram International Publishing (I) Ltd. Mumbai. 7. KLM Technology Group; Project Engineering Standard. (2011, February). Start-up sequence and general commissioning procedures- Project Standards and Specifications. 8. Mukherjee, S. (2005, January). Preparations for initial startup of a process unit; Lurgi India Company Ltd.; Chemical Engineering (www.che.com). 9. iFluids Engineering (www.ifluids.com); How to Write a plant operating Manual. (accessed in September 2020). 10. KLM Technology Group Project Engineering Standard. (2011, February). Plant Operating Manual- Project Standards and Specifications.

Chapter 5

Financial Aspects of Project Engineering

Learning Objectives • To understand the cash flow of a project and to know the significance of all quantities in cash flow. • To calculate capital investment, interests and depreciation. • To estimate profitability. • To understand the basics of the balance sheet of a company. • To decide on alternative investments as well as the replacement of an asset.

5.1 Preamble A design is acceptable only when it represents a plant that can produce a product that will generate a profit. An estimate of the initial investment is required and the cost of production should be calculated to assess the profitability of a plant or project. Though cost estimation is a specialized subject, a chemical engineer/design engineer should be able to make a rough cost estimate to decide between different alternative design parameters and to optimize the design. In this chapter, the components of cost estimates are introduced along with overviews of some techniques. Common terminologies are explained. For cost estimation, a few cost data or indices are required which are available in the reports regularly published by a few companies or professional bodies.

5.2 Cash Flow Project cash flow is a part of the financial planning of a project. Within a particular period, the net amount of cash or its equivalent that enters into and exits out of a project is called Cash Flow for that period (Fig. 5.1). Positive cash flow indicates © The Author(s), under exclusive license to Springer Nature Singapore Pte Ltd. 2022 S. Chakrabarti, Project Engineering Primer for Chemical Engineers, https://doi.org/10.1007/978-981-19-0660-2_5

135

136

5 Financial Aspects of Project Engineering

Fig. 5.1 Schematic of cash flow

that the company is adding to its cash reserve allowing it to re-invest, paying out dividends to the shareholders or settling future expenditures; whereas negative cash flow indicates the company is taking out cash from its reserve [1]. Cash flow can be of different forms—investing, operating and financing. Free cash flow is an indicator of the financial health of the company to the economic analysts. It represents the net cash in hand after considering the cash outflow; it can be used for the operation and maintenance of its asset. Let us assume the corporate treasury as a reservoir of money where there are inflows and outflows. The fund inflows include loan, stock issues, bond sales and other capital inputs. Outflows from the reservoir are generally the following: • Total capital investments. • Dividends to shareholders.

5.2 Cash Flow

137

• Other investments. • Repayment of debts. For the time being, this Total Capital Investment (TCI) will be our focus of interest. Total Capital Investment considered here excludes the cost of land. There are three components of TCI as follows: • Manufacturing Fixed Capital Investment. • Non-manufacturing Fixed Capital Investment. • Working capital investment. Now let us introduce various terms appearing in the cash flow schematic.

5.2.1 Capital Investment A traditional economic definition of ‘Capital’ is ‘a stock of accumulated wealth’. Practically, capital is the savings that the owner can use in whatever way s/he wants [1, 2]. Before an industrial plant is put into operation, a large sum of money must be available to purchase and install the required machinery and equipment. Money is required for purchasing land, to make service facilities available and for erection with piping and control. In addition, funds are required for paying the expenses involved for operation before sales revenue becomes available. The capital needed to supply the required manufacturing and plant facilities is called the fixed capital investment (FCI) while that necessary for the operation of the plant is termed as working capital (WC). The sum of fixed and working capital is known as a total capital investment (TCI). The fixed capital portion may be sub-divided into manufacturing fixed capital investment or Direct cost and non-manufacturing fixed capital investment or Indirect cost. Manufacturing Fixed Capital Investment includes all major process equipment, piping, civil works, etc. Non-manufacturing Fixed Capital Investment includes indirect field costs like field expenses and services, construction cost, labour benefits, etc. Fixed Capital Investment may also be segregated into the following components: • • • •

Inside Battery Limit (ISBL) investment. Offsite or Outside Battery Limit (OSBL) investment. Engineering and construction cost. Contingency charges.

138

5 Financial Aspects of Project Engineering

5.2.2 ISBL Plant Cost During the early stage of inception or at the pre-project stage, ISBL scope and cost should be estimated meticulously so that the overall project economics is properly analysed. ISBL plant costs include direct and indirect field costs [3]. Direct field costs include the following: • • • •

All the major process equipment and instruments. Bulk items like pipe, fittings, catalysts, solvents, paints, packings, lube oils, etc. Civil construction works. Labour and supervision costs for installation.

Indirect field costs include the following: • Construction equipment rental, temporary water and power connection for construction, etc. • Field expenses like temporary canteens, specialist’s visit cost and adverse weather cost. • Construction insurance. • Labour social security and compensation. • Miscellaneous overhead items like legal costs, patent or loyalty fees, import duties and special freight costs.

5.2.3 Offsite Cost or OSBL Investment Offsite cost or OSBL investment means the investment necessary in addition to the basic plant or project. These investments are required for the site infrastructure and developing the facilities which are outside the main plant but absolutely necessary for the same. These include the following: • The captive power plant, substation, dedicated transformer, turbine engine, generator and switchgear. • Boiler, steam mains, condensate and treatment plant for boiler feedwater. • Cooling tower, circulation pump and cooling water treatment plant. • Water and wastewater treatment plants. • Air and instrument air treatment plant. • Storage tanks, loading–unloading facilities, warehouse and lift-trucks. • Laboratories and control rooms. • Emergency medical and fire-fighting facilities. • Site security, fencing and landscaping. Offsite investments generally involve utility companies. Since they would have more impact on the local community due to water, air treatment and power generation, they should be subjected to rigorous scrutiny before an investment is made.

5.2 Cash Flow

139

In the early stage of design, offsite (OSBL) costs are estimated in comparison with the ISBL costs. Depending on the scope of work and nature of the project, offsite costs vary from 10–100%. For greenfield projects (completely new projects on vacant sites or land), offsite costs are higher than that of a brownfield (projects which are modified or upgraded) project, because some offsite facilities are expected to be already existing in the latter case. For typical petrochemical projects, offsite costs are assumed about 40% of the ISBL costs. Engineering cost means the costs for the basic and detailed design and engineering, cost for the process of procurement, supervision for construction and services and project management costs like inspection and travel for engineering purposes. Fees for the engineering contractor are also included. Though many factors influence engineering costs, it is generally 30% of the sum of ISBL and OSBL costs for small projects and about 10% of the same for large projects. Contingency charges take care of unexpected expenses like costs of change in engineering, price escalation, currency value fluctuation or labour disputes. It is generally assumed as 10% of the sum of ISBL and OSBL costs. Its fraction may, however, change depending on the maturity of the technology used and the confidence level of the cost estimates. Cost of production and working capital—After the plant is constructed, some amount of money is needed for running the plant till the plant itself starts earning money by selling its product. For a single-product industry with small product storage, the working capital may be approximately 5% of the total fixed capital (sum of ISBL and OSBL costs) whereas for an industry with a diverse product range and large product storage, the fraction may be 30% of the total fixed capital. In the case of a petrochemical plant, the fraction of working capital is about 15% of the total fixed investment. However, Working capital has many components which are estimated and expressed as the fractions of the cost of production rather than relative to the fixed capital. So it is pertinent to understand the various components of the cost of production in course of estimation of the working capital. The ratio of working capital to total capital investment varies with different companies, but most chemical plants use an initial working capital equal to 10–20% of total capital investment. The percentage may increase to 50% or more for companies producing products of seasonal demand. This is due to the large inventory to be kept. Cost of production refers to the total money needed for the production of a particular quantity of the output or product. It has two major divisions—the fixed cost of production and the variable cost of production. Fixed costs of production are the costs that should be incurred regardless of the quantity of the product or the rate of the running plant. For small capacity plants, the fixed cost of production is a burden. For higher production rates, fixed cost per unit or product decreases, and the cost of the product becomes flexible. So in a critical situation, small industries cannot reduce the price of a particular product below a certain limit, but large companies can. Fixed costs are not directly influenced by the

140

5 Financial Aspects of Project Engineering

better design or operation of a plant. It is rather influenced by the local business environment and corporate decisions. Fixed cost of production generally includes the following: • Operating labour and its supervision charges. Supervision charges are generally about 25% of the labour charges. • General plant overhead costs like costs for research and development, information technology and management and financial and legal management. • Direct salary overhead like payroll taxes, fringe benefits or health insurance. It is generally assumed as 40–60% of operating labour and its supervision cost. • Maintenance costs include both labour and material. Generally, it is assumed as 3–5% of the ISBL investment. • Property tax and insurance—typically it is about 1–2% of the ISBL fixed capital. • Rent of land or building, if applicable. • Environmental charges—this is the cost to be paid as compensation for the actual or potential deterioration of the natural assets due to the project. Its amount differs as per the rule of the land. • Running licence and patent fees. • Loan repayment or capital charges. • Sales and marketing costs. Variable costs of production are costs that depend on the rate of production or output. This generally includes the following: • • • • •

Cost of raw materials consumed. Cost of consumables spent. Costs of utilities used. Effluent/waste disposal cost. Packaging and shipping cost.

Now based on the cost of production, we can now estimate the working capital as follows: • Value of raw material inventory—generally assumed as 2 weeks delivered cost of raw materials. • Value of product and by-product inventory—generally assumed as 2 weeks cost of production. • Cash on hand—assumed 1 week cost of production used for paying wages or contingency. • Receivable accounts—Cost of products already shipped but payments for which have not been received yet. • Payable accounts—Cost of raw materials, solvents and catalysts received but not paid for; basically, this is a credit. Assumed to be 1 month delivered cost. • Spare parts inventory—estimated as 1–2% of the total sum of ISBL and OSBL costs.

5.2 Cash Flow

141

5.2.4 Depreciation Charges and Other Investment Incentives Governments of nearly all countries offer incentives to encourage capital investments for the development of the local economy. Tax waivers, first-year allowances, low-interest loans and depreciation are among such incentives. Most of these are subject to local laws but depreciation is generally applicable all over the world. Depreciation charges can be thought of as an allowance for the wear and tear, deterioration and obsolescence of the fixed asset as a result of its use. It is assumed that its value is decreasing over time from its original purchase value. The amount decreased is considered as spent. The reduced value of the asset is called ‘book value’. However, it does not have any formal relationship with its resale value or current market value. Basically, depreciation is a non-cash charge reported as an expense that reduces taxable income. There is no cash outlay for depreciation and no money is transferred to any account. Hence, the depreciation charges are added back to the net income after the deduction of tax to give the total cash outflow from the operation. Mathematical expressions for the above statements are as follows: Taxable income = Gross profit −Tax incentives. CF = P − (P − D1 )tr = P(1 − tr ) + D1 tr

(5.1)

where C F = cash flow after tax, P = Gross profit, D1 = sum of tax incentives and t r = rate of taxation. Again with respect to taxable income and depreciation, CF = I − (I · tr ) + D · tr Since

(5.2)

I = (P − D) then, CF = (P − D) − {(P − D)tr + D = P(1 − tr ) + D · tr

where I = taxable income and D = depreciation tax incentive/allowance. If depreciation is the only tax allowance, the two equations are the same. It may be noted that the land does not depreciate, and generally, only the fixed capital investment is allowed to have the facility of depreciation, not the working or total capital investment.

142

5 Financial Aspects of Project Engineering

The schedule of deduction of depreciation charges is governed by the law of the land. There are several methods for calculating depreciation charges—a few of them will be described in Sect. 5.4.

5.3 Estimation of Capital Investment An estimate for the capital investment for a process may vary from a very rough predesign to the detailed estimate prepared from complete drawing and specification. There are many estimates in between these two extremes. Some of them are given as follows: Order of magnitude estimate or ratio estimate: It is based on similar previous cost data. Probable accuracy of estimate over ±30%. Study estimate or factored estimate: Based on knowledge of major items of equipment. Probable accuracy ±30%. A preliminary budget authorization or scope estimate: Based on sufficient data to permit the estimate. Probable accuracy ±20%. Definitive or project control estimate: Based on almost complete data but before completion of drawing and specification; Probable accuracy ±10%. Detailed or contractor’s estimate: Based on complete engineering drawings and specifications and site surveys; Probable accuracy ±5%. The first three of the above are considered as pre-design cost estimates whereas the last two are considered as the firm estimates. Pre-design cost estimates require much less detail than firm estimates. However, the pre-design estimates are extremely important for determining whether a proposed project should be further considered. Pre-design estimates are used to provide a basis for requesting and obtaining a capital appropriation from company management or funding agency. Later estimates may indicate that the project will cost more or less than the amount appropriated [1].

5.3.1 Cost Indexes Because prices may have changed considerably with time due to changes in economic conditions, some methods must be used for updating cost data applicable at a past date. This is generally done by cost indexes [1, 4]. A cost index is an index value for a given time showing the cost at that time relative to a certain base time. If the cost at some time in the past is known, the equivalent cost at present can be estimated by multiplying the original cost by the ratio at the present index value applicable when the original cost was obtained.  Present cost = Original cost

Index value at the present index value at the time when the original cost was obtained



(5.3)

5.3 Estimation of Capital Investment

143

Cost indexes are used for a general estimate. No cost index can consider all the factors. The common cost indexes predict fairly well within less than 10 years. Many cost indexes are published regularly. Some of them are suitable for equipment cost, some are for labour cost and so on. A few of these indexes are the Chemical Engineering Plant cost index, Nelson-Farrar Refinery Construction index, Process Industry equipment Index and so on. All cost indexes are based on a limited sampling of the goods and services; therefore, two indexes covering the same type of projects may give results that differ considerably.

5.3.2 Cost Components of Capital Investment Fractional costs of each major component of fixed capital investment (FCI) are considered as follows. The following table (Table 5.1) summarizes typical variation in component cost as a percentage of fixed capital invested for multi-process grass root plants or large battery limit additions. Table 5.1 Cost of various components as fractions of FCI [1]

Components

Range of FCI %

Direct costs Purchased equipment(PE)

15–40

Installation of PE

6–14

Instrument and control (installed)

2–12

Piping (installed)

4–17

Electrical (Installed)

2–10

Building and service

2–18

Yard improvements

2–5

Service facilities installed

8–30

Land

1–2

Indirect costs Engineering and supervision

4–20

Construction

4–17

Legal expenses

1–3

Contractor’s feed

2–6

Contingency

5–15

144

5 Financial Aspects of Project Engineering

5.3.3 Estimation of Purchased Equipment Cost For estimating capital investment, the basis used by many pre-design methods is often the cost of the purchased equipment. The various types of equipment can be divided into three sub-groups—processing equipment, raw material handling equipment and finished product handling and storage equipment. The most important thing to get an accurate estimate of equipment is its detailed specification and material of construction. The most dependable source of the cost estimate is the bid obtained from the fabricator or supplier against a detailed specification. If this is not possible, the next reliable estimate can be made from the previous purchase order for a piece of similar equipment. For estimating the cost of the new equipment, the purchase order price of the old equipment should be updated using an appropriate cost index ratio. The quickest way to make an order of magnitude estimate of a plant or equipment is to scale it from the known cost of a previous plant or equipment that used the same technology. This requires no other information than the capacity or throughput. The capital costs of two plants/equipment with different capacities are related as follows:  C2 = C1 ×

S2 S1

n (5.4)

where C 2 = cost of plant/equipment with capacity S 2 and C 1 = cost of plant/equipment with capacity S 1 . Value of the exponent n = 0.8–0.9 for a process using a lot of mechanical work or gas compression = 0.7 for a typical petrochemical plant or process; = 0.4–0.5 for small-scale highly instrumented process. The average value of n is about 0.6; hence, the above equation reduces to  C2 = C1 ×

S2 S1

0.6 (5.5)

It is generally referred to as the ‘six-tenths rule’ which is best represented by a log– log plot of capacity vs cost ratio. Many such plots are available for different equipment where the key parameters are plotted against the cost. For example, to estimate the cost of a reactor, purchased costs are plotted against volumetric capacities, whereas for heat exchangers, heating surface areas are plotted against costs and for an electric motor, delivered power is the cost determining factor. Instead of the average value of 0.6, more accurate values of the exponent n can be used for various equipment/systems to obtain a better estimate of the cost. The following table (Table 5.2) tabulates some of the process equipment with the corresponding value of n.

5.3 Estimation of Capital Investment

145

Table 5.2 Value of the exponent for different process equipment Name of the process equipment

Capacity/size range

Value of n

Vacuum batch crystallizer

15–200 m3

0.37

Reciprocating air-cooled two-stage compressor, discharge pr. 10 atm

0.005–0.19m3 /min

0.69

Fixed-head shell and tube heat exchanger—carbon steel

10–40m2 heating surface area

0.44

Explosion-proof induction motor with squirrel cage

15–150 kW, 440 V

0.99

Horizontal centrifugal pump (cast steel) 4–40m3 /s with motor

0.33

Stainless steel reactor

0.4–4m3 , up to 20 atm pressure

0.56

Flat-headed carbon steel tank

0.4–40m3

0.57

Bubble cap tray—carbon steel

1–3 m diameter

1.2

Prices of the purchased equipment are quoted as FOB (free on board) or CIF (Cost, insurance and freight). FOB price means the purchaser should pay for the insurance and transport cost whereas, in the case of CIF price, the supplier pays the insurance and freight cost. Freight or transportation cost depends on many factors like distance to carry, equipment size, material and weight as well as the mode of transport. Generally, it is assumed as 10% of the equipment cost for estimation purposes. Installation of process equipment involves labour cost, platform, support and other factors related to the erection of the equipment at the site. It is generally assumed as 25–55% of the delivered equipment cost. Sometimes, insulation and piping costs are included under this head. For a normal solid–fluid chemical processing plant, the cost for instrumentation and control is generally assumed as 26% of the delivered purchased equipment cost. This is approximately 5% of the total capital investment. The cost of piping includes costs for pipes, valves and fittings along with their installation cost. For a solid–fluid type chemical plant, it is assumed to be 30% of the delivered equipment cost or 7% of the total fixed cost investment. Some of the most reliable information can be found in current professional cost engineering literature published by various professional bodies all over the world. Academicians usually do not have access to such high-quality and dependable cost data to generate suitable correlations.

5.3.4 Methods of Estimating Capital Investment Various methods can be applied for estimating capital investment the types of which are described before [1]:

146

5 Financial Aspects of Project Engineering

Detailed item estimate—A detailed item-wise estimate is made. Equipment and material needs are determined from completed drawings and specifications and are priced either from current cost data or from firm quotations. Estimates of installation costs are determined from accurate labour rates, efficiencies and employee-hour calculation. Accurate estimates for engineering charges, field supervision and field expenses are made. An accuracy of ± 5% is expected. Unit cost estimate—The unit cost method gives good estimates provided accurate records have been kept of previous cost experiences. A unit cost is also applied for engineering employee hours, the number of drawings, specifications, etc. A factor for construction expenses, contractor’s fees and contingency are estimated from previously completed projects. Cn =

       f e He + f d dn × f F (5.6) f x Mx + f L M L + (E + E L ) +

where C n = new estimated capital investment, E = delivered purchased equipment cost, E L = Labour cost of delivered equipment, f x = specific material unit cost, M x = specific material quantity, f L = Specific material labour unit cost per employee hour, M L = Specific material labour employee hour, f e = unit cost of engineering employee hours, H e = engineering employee hours, f d = unit cost per drawing/document/specification, d n = number of drawing/document/specification and f F = construction or field expense factor. Percentage of delivered equipment cost: This method estimates fixed capital and total capital investment. It requires determination of the delivered equipment cost. The other items included in the total direct plant cost are estimated as a percentage of delivered equipment cost. Cn =

 (E + f 1 E + f 2 E + . . . f n E)

Cn =



E(1 + f 1 + f 2 + . . . f n )

where C n = new estimated capital investment,

(5.7) (5.8)

5.3 Estimation of Capital Investment

147

E = delivered equipment cost and f 1 ….f n = factors for piping, electrical, civil, etc. available in the literature. Estimation by this method is commonly used for preliminary and study estimates. The expected accuracy is ±20–30%. Lang factors for approximation of capital investment: This is basically an order of magnitude cost estimate. It recognizes that cost of a process plant may be obtained by multiplying equipment cost by some factor. The factor varies depending on the type of process plant. This method may consider different factors for different types of equipment. Another approach is to use different factors for utilities, piping foundation, etc. Sometimes, each item of cost is divided into material and labour costs. So each factor may have a range of value, and the chemical engineer must rely upon his/her personal experience.    Cn = fl E (1 + f F + f P + f m ) + E i + A

(5.9)

where the three installation cost factors are    e   fv  , log f F = 0.635 − 0.154log 0.001E − 0.992  + 0.506 E E  e   p   log f p = −0.266 − 0.014log 0.001E − 0.156  + 0.556  , E E     t  log f m = 0.344 + 0.033log 0.001E + 1.194 , E

(5.10) (5.11) (5.12)

E  = purchased equipment FOB basis, f L = indirect cost factor (>1) Lang factor, f F = cost factor for field labour, f p = cost factor for piping materials, f m = cost factor for miscellaneous items including the material cost for instruments, insulation, foundation, etc., A = incremental cost for corrosion resistant alloy materials, e = heat exchanger cost (less cost of alloy), f v = cost of the pre-fabricated vessel (less cost of alloy), f p = cost of pump and driver (less cost of alloy) and t = cost of tower shells (less cost of alloy).

148

5 Financial Aspects of Project Engineering

Power factor applied to plant /capacity ratio: This is an order of magnitude estimate used for the study. Here, fixed capital estimates of a new process plant are determined by an exponential power ratio of the fixed capital investment of similar previously constructed plants. So, Cn = C f e R x

(5.13)

where C n = fixed capital investment of a new facility, C = fixed capital investment of an already constructed similar facility, R = ratio of the capacity of a new facility to the capacity of the old facility, x = power having an average value between 0.6–0.7 and f e = cost index ratio at the time of cost of C n to that at the time of C. Another relationship that involves both direct and indirect plant costs is as follows: Cn = f (D R x + I )

(5.14)

where f = a lumped cost index factor relative to the original facility cost; it is the product of geographic labour cost index corresponding to area labour productivity index and a material and equipment cost index, D = direct cost and I = total indirect cost for the previously installed facility of a similar unit on an equivalent site. The value of the power factor x approaches 1 if the capacity is increased by increasing the number of units rather than increasing size. Investment cost per unit of capacity: The product of investment cost per unit of capacity and the annual production capacity of the proposed plant may give a rough estimate of fixed capital investment for a given process. Necessary correction of cost with time should be made with the cost index. Turnover ratio: It is also an order of magnitude estimate using turnover ratio, which is defined as follows: Turnover ratio =

Gross annual sales Fixed capital investment

(5.15)

The reciprocal of ‘turnover ratio’ is ‘capital ratio’ or ‘investment ratio’. For chemical industries, the turnover ratio is approximately 0.5.

5.4 Methods for Calculation of Depreciation Charges

149

5.4 Methods for Calculation of Depreciation Charges Further to the previous section where the term ‘depreciation’ was introduced, a few methods for calculating depreciation charges are described here. Straight-line depreciation is the simplest method. The depreciable value Cd depreciates over a period of n years with an annual charge of Di in the ith year. Then, Di =

Cd n

(5.16)

The depreciable value is the initial cost of the fixed asset minus the salvage value at the end of the depreciable period. The salvage value or scrap value of an asset is the estimated resale value of a fixed asset at the end of its useful life. It is subtracted from the original cost of the fixed asset to determine the depreciation cost. Hence, salvage, scrap or residual value is used as a component for the calculation of depreciation. For chemical plants, salvage value is often taken as 0, since the plant generally operates for many years beyond the depreciation period. The book value of the asset, after m years of depreciation, is Bm , which is Bm = C −

m 

Di

(5.17)

i=1

Bm = C −

mCd n

(5.18)

where C is the original cost of the fixed asset. If the book value becomes equal to the salvage value, the fixed asset will depreciate no more, and hence, no depreciation charges should be considered. The straight-line method is generally used to calculate depreciation in the US software industries with 36 months of depreciable life. Declining balance method of calculating depreciation considers the annual depreciation charge as a fixed fraction of the book value of the same. If the fraction is F d , then, In the 1st year, depreciation D1 = CF d and book value B1 = C − CF d = C(1 − F d ); in the 2nd year, depreciation D2 = B1 F d = C(1 − F d )F d . and book value B2 = B1 − D2 = C(1 − F d ) − C(1 − F d )F d = C(1 − F d )2 . Hence, by induction, in mth year, depreciation, Dm = C(1 − Fd )m−1 Fd

(5.19)

150

5 Financial Aspects of Project Engineering

and book value, Bm = C(1 − Fd )m .

(5.20)

The fraction F d must be ≤ n2 . When Fd = n2 , it is called the double-declining balance method. The formula for calculating depreciation by double-declining balance method is Depreciation = 2 × Straight line deprecistion × initial book value

(5.21)

The advantage of the declining-balance method is that it allows higher depreciation charges in the early years of a project. Hence, higher cash flow is there in the early years and thereby the project economy improves. Another method is the Modified Accelerated Cost Recovery System (MACRS). It is basically a combination of the double-declining method and the straight-line method. It calculates depreciation using the double-declining balance method until the depreciation charges become less than the amount calculated by the straight-line method. After that point, the straight-line method is used. In this method, different recovery periods are assigned to different kinds of assets based on their respective usable lives. Another important aspect of this method is it assumes that all assets were acquired in the middle of the year; hence, for the first and the last years, depreciation charges are calculated for half the year rather than a full year. Under the Sinking Fund method, a company maintains a fund to replace the fixed asset after its useful life is over. Accordingly, they calculate depreciation depending on the value of the asset and its life n years. The firm can invest the sinking fund to earn interest until the asset needs replacement. Hence, the rate of depreciation by this method is D=

Ci (1 + i)n − 1

(5.22)

where C = Cost of the fixed asset, i = fractional rate of annual interest and n = estimated life of the asset in years. The Sum of the year’s digit method is a form of accelerated depreciation. The depreciation expenses are computed as the percentage of the remaining life of the asset to the total of all the years of its estimated life concerning the present book value of the asset. Depreciation =

Remaining life (Starting book value − Residual value) (5.23) of the years

The annuity method is based on the opportunity cost.

5.4 Methods for Calculation of Depreciation Charges

151

An annuity is a type of financial investment that pays out a fixed and regular dividend. It is a series of payments made in equal intervals that is in each year against an investment. There are different types of annuities according to the frequency of premium, type and duration of pay-outs or number of the purchaser. There are several options also for the investor like life, inflation-linked or repurchase. Perpetuity is a security that pays for an indefinite time. It is an endless constant stream of cash flow of the same amount. PV =

C C C C + (5.24) + + . . . . . . . . . . . . . . . . . . . . . . . . .. = (1 + i) (1 + i)2 i (1 + i)3

where PV = present value, i = discount rate and C = cash flow. Opportunity cost is sometimes referred to as alternative cost. Here, the depreciation is computed based on the interest it could earn if the amount used for the purchase of the fixed asset was invested somewhere else. So, annuity depreciation =

Ci(1 + i)n (1 + i)n − 1

(5.25)

5.5 Economic Analysis and Profitability Study The purpose of investigating money in a chemical plant is to earn a profit. So a study of the profitability of a particular project is done before investing in a project. There are several methods of analysing the profitability of a project. Before describing the methods, a few terms should be explained or introduced. Interest: Interest is the additional earning when money is lent; it is the cost of the borrowed money. Rate of interest is the amount of money paid on a unit of principal in a unit of time; it is generally expressed as a fraction or percentage per year. Interest to be paid on a loan taken for a project is an important parameter for its economic analysis. Modes of interest payment maybe three—simple interest, compound interest and continuous compound interest. Simple interest is the simplest form of interest payment. It means payment at a constant interest rate based on the original principal. If P is the principal, n is the interest period and i is the rate of interest per unit period, then the total amount of

152

5 Financial Aspects of Project Engineering

principal interest, F, is F = P(1 + in)

(5.26)

Compound interest is generally used in business transactions. In this case, the amount of interest earned in the first year adds to the principal for the second year and so on. Hence, in the first year, the principal is P, interest is Pi and the total amount is P(1 + i). In the second year, principal is P(1 + i), interest is P(1 + i)i and the total amount is P(1 + i) + P(1 + i)i = P(1 + i)2 . So by induction for an n interest period or year, total amount F = P(1 + i)n . If the interest is compounded m times in an interest period n, then the total amount F becomes i mn ) n

F = P(1 +

(5.27)

Now to determine an effective compound interest rate that is compounded annually, P(1 + i eff )n = P(1 + Simplifying, we get i eff

i mn ) n

  i n = 1+ −1 n

(5.28) (5.29)

Continuous compound interest is that where interest is compounded continuously. Physically this ‘continuously’ means the number of times it is compounded in an interest period is infinity. Hence, we can say F = P lim (1 + m→∞

i mn ) m

i i (in) F = P lim (1 + ) m→∞ m m

Now from the basic definition of ‘e’, the base of the natural logarithm and the exponential series, we know m

e = lim (1 + m→∞

i i ) = 2.71828 m

So, F = Pein

(5.30)

5.5 Economic Analysis and Profitability Study

153

Effective rate of interest, i e f f = ei − 1 ei = 1 + i e f f

(5.31)

and ein = (1 + i eff )n as well as i = ln(1 + i eff ) So, F = Pein = P(1 + i e f f )n

(5.32)

Time value of money: We know that an amount of money that is invested increases in value or magnitude with time. In other words, an amount of money available at some time in the future is equivalent to a smaller amount at present. This effect of changing the value of money with time is known as the time value of money. The time value of money is a very important parameter for economic comparisons where all cash flows should be made equivalent. It enables to determine the value of the investment at any point of time. The time value of money is an important consideration when investments are compared that require or generate amounts of funds at different times. It has already been observed that the future worth of a present amount of money can be calculated using compound interest formulae. Now the inverse of such formulae may be used to calculate the present worth of a future amount of money. Hence, for discrete compounding, we get P=

F (1 + i)n

For continuous compounding, P = Fe−in

(5.33) (5.34)

Cost of Capital: Most of the projects are funded by both debt and equity funding. The weighted average of the cost of debt and cost of equity is the overall cost of capital. Debt is an amount of money borrowed by one party from another. Debt is used as a method of making large expenditures that one could not afford under normal circumstances. A debt arrangement gives the borrowing party provision to borrow money with the condition to pay it back at a later date, usually with interest.

154

5 Financial Aspects of Project Engineering

Equity is a mandatory parameter found on a company’s balance sheet and is one financial metric to assess the financial health of the company. It is typically referred to as shareholder’s equity which means the amount of money that would be returned to the shareholders if all the assets were liquidated and all the debts were paid off. Shareholder’s equity = Total assets – Total liabilities Now, the cost of capital can be expressed in terms of debt and equity as follows: i C = dr i d + {(1 − dr )i e }

(5.35)

where i C = cost of capital, dr = interest on debt, i d = debt ratio and i e = cost of equity. Debt ratio: The ratio of total debt to total assets, expressed as a fraction or percentage, is defined as the debt ratio. It is often interpreted as the fraction of the company’s total assets financed by debt. A ratio greater than 1 means that the company has more liabilities than assets. A high ratio also indicates that a company may be putting itself at risk of default on its loans if interest rates were to rise suddenly. A ratio below 1 translates to the fact that a greater portion of a company’s assets is funded by equity. The overall cost of capital sets the rate of interest that is used for the economic analysis of the project. Cost of capital is considered as a basic standard for profitability since any project must earn at least at the rate of the cost of capital just to repay the external sources of capital—loan repayment and shareholders’ equity. Minimum acceptable rate of return (MARR): It is the rate of earning that must be achieved by an investment for it to become attractive to an investor. The symbol is mar (minimum acceptable annual rate of return) and is expressed as a percentage per year. It is based on the highest rate of earnings on safe alternative investments available in the market for the investor. Otherwise, the investor will not invest in a project. Moreover, the uncertainties and risks associated with the project should also be considered while fixing the mar .

5.5.1 Methods for Calculating Profitability There are several methods for calculating profitability—some of them do not consider the time value of money and some do (Fig. 5.2). Payback period and Return on Investment (ROI) are examples of the first category whereas Discounted Cash Flow Rate of Return (DCFRR) is included in the second category [5].

5.5 Economic Analysis and Profitability Study

155

Fig. 5.2 Classification of the methods for profitability analysis

Cash flow diagram and cash flow pattern Before going into the details of the methods mentioned above, let us look into the cash flow once again. The following figure (Fig. 5.3) elaborates the cash flow mentioned in Fig. 5.1 from a different approach. If we look at the cash flow against time, it will look like the following figure. The cash flow diagram and the pattern of cash flow are indicative of the profitability of a project. Now let us analyse different segments of the cash flow diagram. The project starts at A. A–B segment is for pre-project and designing the plant. Segment B–B indicates high investment for the purchase of equipment and building of the plant. At point B , working capital is required (B –C) and the plant starts operating. C–C shows the maximum point of investment summing up the fixed and working capitals. Till then there is no earning from the plant, and hence, the cumulative cash flow is negative. But from point C, the cash flow starts growing in a positive direction. However, the earning is still less than the investments made so far, and the cumulative cash flow remains negative. At point C on the zero-cash line, the plant starts operation, and at point D, the cumulative cash flow reaches zero. That means at this point of time, all the investments are paid off. This is called Break-even Point, and the time required to reach this point is called Payback Period. The slope of line C–D is also important for reaching a break-even point. If the slope is small, that if the rate of earning is less, it would take a longer time to reach a break-even point to make the project less attractive for the investors. The portion D–E has a positive cash flow and indicates that the revenue is being earned and the project gives profit. After point E, the rate of profit is slow at the end of project life. The cash flow pattern is also a parameter for the economic analysis. Since cash flow occurs throughout the life of the project, it is often necessary to convert them to the equivalent values at a particular point in time. It may be done either by discounting future cash flow or by compounding previous cash flow up to that point. Generally, during the design phase, the projected plant start-up time is considered as the ‘present’ point and the values of cash flow are calculated accordingly.

156

5 Financial Aspects of Project Engineering

Fig. 5.3 Cash flow diagram against time [3]

Graphical cash flow patterns may be discrete or continuous depending on the occurrence of cash flow. Balance sheet The balance sheet summarizes the financial position of a company by indicating what the company owns and owes along with the investments made by the shareholders at a particular point of time [6]. Balance sheet, cash flow and income statement are the three basic documents by which the financial health of a company is analysed. A few ratios indicating the financial health of the company to the investors, including the debt-equity ratio or acid-test ratio, can be derived from the balance sheet. The governing formula of the balance sheet is Asset = Liabilities + Shareholder’s equity

5.5 Economic Analysis and Profitability Study

157

The formula is obvious—a company has to pay for all the things it owns (assets) by either borrowing money (taking on liabilities) or taking it from investors (issuing shareholders’ equity). Assets An asset is a resource with economic value that an individual, corporation, or country owns or controls with the expectation that it will provide a future benefit. In the assets segment of the balance sheet, assets are listed from top to bottom in order of their liquidity, that is, the ease with which they can be converted into cash. Current assets are converted to cash in one year or less; non-current or long-term assets are not [2]. Current assets include the following: • • • • • • • •

Cash and cash equivalents. Notes and accounts receivable. Raw materials, products, spare parts and other supplies in store. Prepaid expenses and taxes. Joint ventures or investments in other companies. Non-current or fixed assets include the following: Property, plant and machinery listed at book value and land. Patents, trademarks, goodwill, etc.

Liabilities Liabilities are the money that a company owes to outside parties, from bills it has to pay to suppliers to interest on bonds it has issued to creditors to rent, utilities and salaries. Current liabilities are those that are due within one year and are listed in order of their due date. At any point of time after one year, long-term liabilities are due; they are listed in the order they are due. Current liabilities may include the following: • • • • • • •

Current portion of long-term debt. Bank loan. Interests payable. Wages and salary payable. Dividends payable and others. Earned and unearned premiums. Accounts payable. Long-term liability includes the following:

• Long-term debt, that is, interest and principal on bonds issued. • Pension fund liability, that is, the money a company should pay into its employees’ retirement accounts. • Deferred tax liability, the taxes that have been accrued but will not be paid for another year.

158

5 Financial Aspects of Project Engineering

Shareholder’s Equity It is the difference between the total asset and total liability. It is the debt to the non-shareholders and is also known as ‘net assets’. Methods not considering the time value of money Payback time or Payback period is the time necessary for the total return to be equal to the capital investment. If total fixed capital investment is FC and average annual cash flow is Av , then the payback period, PB = FC /Av years. This payback period ignores taxes or depreciation. It ignores the time value of money. For simplicity, it assumes that all the capital investments are made in time zero, and revenues begin to come immediately. For most chemical plants, this is not the actual scenario. Practically, investments are typically spread over 1–3 years and revenues do not generally reach their full capacity before the second year. Hence, this payback period is not the same as that indicated in the cash flow diagram. Return on investment (ROI) is defined as follows: ROI =

Net annual profit × 100% Total investment

(5.36)

If ROI is calculated as an average over the whole project, then ROI − P =

Cumulative net profit × 100% Plant life × Initial investment

(5.37)

A pre-tax ROI may also be calculated as follows: ROI − pt =

Pre − tax annual cash flow × 100 Total investment

(5.38)

Methods considering the time value of money These methods consider earning powers of the invested money [3]. Net Present Worth or Net Present Value (NPW or NPV) is the total of the present worth of all cash flows minus the present worth of all capital investments as defined by NPV =

t  n=1

where CF n = Cash flow in year n, t = project life in years and

C Fn (1 + i)n

(5.39)

5.5 Economic Analysis and Profitability Study

159

i = cost of capital (rate of interest, fraction). Net present value is less than the total future worth of the project because of discounting of future cash flows. NPV is a strong function of the interest rate used and the time period studied. Time periods analysed are sometimes denoted by a number, e.g. NPV10 means the NPV over 10 years. Discounted cash flow rate of return (DCFROR or DCFR)—By calculating the NPV at various interest rates, it is possible to find an interest rate at which the cumulative net present value at the end of the project is zero (since at break-even point, net cash flow is zero). This rate is called discounted cash flow rate of return and is a measure of the maximum interest rate that the project can pay for reaching the break-even point at the end of project life. t  n=1

C Fn =0 (1 + i  )n

(5.40)

Here, i is the discounted cash flow rate of return. A more profitable project pays a higher DCFROR. DCFROR is a useful way of comparing the performances of capital investments for different projects, irrespective of the amount used, project size, project life or the actual interest rate prevailing at any given point of time. For the investment to be profitable, DCFROR should be greater than the cost of capital. Annualized Cost Methods—This method converts the capital cost into a future capital charge. That is as if the investment is done annually in an instalment rather than at one time at the beginning of the project. If an amount P is invested at a compound interest rate i, then after n years, it will become P(1 + i)n . Instead, if an amount A is invested annually, the matured amount (S) after n years will be S = A + A(1 + i) + A(1 + i)2 + . . . . . . . . . . . . . . . + A(1 + i)n−1 Or, S(1 + i) = A(1 + i) + A(1 + i)2 + A(1 + i)3 . . . . . . . . . . . . . . . . . . + A(1 + i)n Subtracting the first equation from the second, we get

Si = A (1 + i)n − 1 S=

A (1 + i)n − 1 i

(5.41)

If annually invested capital should be equal to the one-time invested capital after n years, then S should be = F

160

5 Financial Aspects of Project Engineering

So, F = P(1 + i)n =

A (1 + i)n − 1 i

(5.42)

Then, A i(1 + i)n

= P (1 + i)n − 1

(5.43)

This A/P ratio is called the annual capital charge ratio (ACCR). It is the fraction of principal that must be paid each year so that the total sum of principal and interest can be paid off within the investment life. This is generally used for the calculation of EMI of house building loans and other similar loans. If the interest rate is considered as the only cost of capital, then ACCR can be used to calculate the annual capital charge (ACC) or annualized capital cost from initial capital expenses. ACC = ACC R × Total capital cost

(5.44)

Annual capital charge (ACC) plus the annual operational costs gives the total annual cost (TAC) which is sometimes referred to as total cost of production (TCOP). This method does not include taxes or depreciation and assumes that all the revenue is available for the return on the initial investment. This method is useful for small projects involving less time so that errors are minimal. It is also used for comparing the cost of the equipment with different life expectancies. Clarifications of a few income tax situations for probability analysis are as follows: • • • •

Revenue = Total income. Net profit = gross profit-income tax = revenue − all expenses − income tax. All expenses = cash expenses + depreciation. Amount of income tax = gross profit × rate of income tax = (revenue − all expenses) × rate of income tax.

• Cash flow = net profit + depreciation = revenue (1 − rate of tax) − cash expenses (1 − rate of tax) + depreciation × rate of tax = revenue (1 − rate of tax) − all expenses (1 − rate of tax) + depreciation. Example for a rate of income tax of 25% per annum with Re.1. Re. 1 of revenue means a cash flow of Re. 0.75. Re. 1 of cash expenses means cash (out) flow of Re. 0.75. Re. 1 of depreciation means cash (in) flow of Re. 0.25.

5.5 Economic Analysis and Profitability Study

161

Effect of inflation on profitability analysis: Inflation is an increase in the price of goods or services over time. Inflation influences the amount of investment required for acquiring goods and services. It is measured as a percentage or fractional change in the price of a quantity with time and almost universally expressed as per cent per year compounded annually. So it is an effective annual rate. j for annual compounding. Price at jth year = Price at 0th year × 1 + i  = Price at 0th year × er



j

for continuous compounding.

where r = nominal value of continuous compounding corresponding to i = ln 1 + i  ,

(5.45)

i = annual rate of inflation and j = no. of years. Inflation should not be confused with the ‘time value of money’. Time value of money is a consequence of the earning power of money with time; it is the money available but inflation is a measure of the money required. Inflation before time zero, that is inflation that occurs between the time of an estimate and the time of the actual expenditure, influences the amount of capital investment required in a project. Hence, it affects the total product cost at time zero and eventually the selling price of the product. Since these quantities are important parameters for economic analysis, inflation should be critically taken care of. A capital cost index described before can provide a reasonable value to calculate inflation. However, this rate of inflation should be extrapolated to the exact point of time when the capital expenditures are expected (zero time). After time zero, capital investments are already made and therefore are not to be subjected to inflation. Depreciation charges are fractions of fixed capital investment; hence, it also will not change over time. Only the operational costs and price of the product may change after time zero. One approach for assessing the effect of inflation after time zero is to increase the sales and the product cost (without depreciation) by the same factor if the same inflation rate applies to both. Another approach is to analyse and compare the purchasing power of the cash flow at time zero with its purchase power on year j after zero time. However, this analysis shows that the influence of inflation after time zero is rather small. Moreover, the prediction of the inflation rate is uncertain. Hence, the effect of inflation on profitability analysis after time zero is sometimes ignored. Sometimes, a sensitivity analysis is done using different inflation rates for various components of revenue and total production cost. It is generally applicable when different inflation rates are used for products and raw materials.

162

5 Financial Aspects of Project Engineering

5.6 Alternative Investments In industry, it is often observed that a particular product can be produced using different processes or technologies. The capital investment and other expenses may vary considerably depending upon the process chosen. At the same time, there may be several choices of alternative business ventures. So it is necessary to decide whether a given business venture is profitable and also to decide which of the several possible methods is the most desirable. A design engineer often encounters situations where an investment is absolutely necessary but there are different alternative designs or configurations available to choose among. There are several methods for alternative investment analysis like incremental investment return and minimum return as a cost. An example given in the worked out examples will explain the methods [1].

5.7 Replacement The term replacement refers to a special type of alternative investment where currently existing facilities are intended to replace by other different ones. Theoretically, the reasons for replacements can be classified into two broad classes. An existing property must be replaced in order to continue operation. Some of the necessitating conditions may be as follows: Property is damaged or worn out and cannot render useful services anymore. The property does not have enough capacity to meet the current demand. The operation of the property is not economically feasible and has become obsolete. The existing property is still functioning and delivering products or services, but more efficient equipment or property is available which can operate more economically. Practically, there may be some more reasons in addition to the above theoretical ones. Personal whims, Prejudice, competition, availability of excess capital or publicity are a few reasons among them. The same methods that were used for profitability analysis can also be used for profitability evaluation of a replacement. Though net present worth and discounted cash flow methods are used, the simple rate of return on investment (ROI) is just as the same effective. The difference between the total cost of the replacement property and the net realizable value of the existing property should be taken as the market value [1].

5.8 Worked Out Examples

163

5.8 Worked Out Examples Example 5.1 (On the calculation of depreciation by straight-line method) Mr. A bought a transportation truck costing Rs. 800,000, which has an estimated life of 10 years, and the scrap value is Rs. 50,000. Find out the annual depreciation expense and the rate of depreciation using the straight-line method. Solution: Annual Depreciation = (Acquisition Cost − Estimated Scrap Value)/Estimated Life in Years. Annual Depreciation = (800,000 − 50,000)/10. Annual Depreciation = Rs. 75,000. Rate of Depreciation = (Annual Depreciation Amount/Acquisition Cost) × 100. Rate of Depreciation = (75,000/800000) × 100. Rate of Depreciation = 9.38%. Example 5.2 (On the calculation of depreciation by Sinking fund method) SC Ltd. bought a crusher worth Rs. 780,000 on March 1, 2000. It has an estimated useful life of 12 years. The company adopted the sinking fund method and invested the sinking fund in a fixed deposit scheme for 12 years to earn 8% p.a. Interest. Compute the sinking fund depreciation. Solution: Sinking Fund Depreciation =

C×i . [(1+i)n −1]

Sinking Fund Depreciation =

780000×0.08 . [(1.08)12 −1]

Sinking Fund Depreciation = 62,400/1.5182. Sinking Fund Depreciation = Rs. 41,101.30. Ans. Example 5.3 ( On the calculation of depreciation by Double-declining balance method) ABC Ltd. bought a machine worth Rs. 600,000 which is depreciated @12% p.a. Find out the depreciation expenses using the double-declining balance method. Solution: Depreciation for a Period = 2 × Straight − Line Depreciation Rate × Book Value at the beginning Year 1: Depreciation = 2 × (12/100) × 600,000 = Rs. 144,000. Year 2: Depreciation = 2 × (12/100) × (600,000 − 144,000) = 0.24 × 456,000 = Rs. 109,440.

164

5 Financial Aspects of Project Engineering

Year 3: Depreciation = 2 × (12/100) × (456,000 − 109,440) = 0.24 × 346,560 = Rs. 83,174.4. Year 4: Depreciation = 2 × (12/100) × (346,560 − 83,174.4) = 0.24 × 263,385.6 = Rs. 63,212.54. Year 5: Depreciation = 2 × (12/100) × (346,560 − 83,174.4) = 0.24 × 263,385.6 = Rs. 63,212.54. And so on. Example 5.4 On double-declining balance method of calculating depreciation A distillation column having a cost of Rs. 5,00,000 has an useful life of 10 years. Assuming a double-declining balance method of calculating depreciation, what will be its book value at the end of 5 years? According to the double-declining balance method, the fraction of depreciation Fd = n2 . Here, Fd =

2 = 0.2. 10

Original cost: P0 = Rs. 5,00,000.Assuming salvage value = 0. After the 1st year, depreciation D1 = 0.2 × 5,00,000 = 1,00,000; P1 = 5,00,000– 1,00,000 = Rs. 4,00,000. After the 2nd year, D2 = 0.2 × 4,00,000 = 80,000; P2 = 4,00,000–80,000 = Rs. 3,20,000. Calculating in the same way, we get After 5 years, P5 = Rs. 1,63, 840. Ans. Example 5.5 (On the calculation of depreciation using Sum of the year digit method) XYZ Ltd. bought a filtration machine worth Rs. 370,000 and its scrap value was estimated as Rs. 40,000. If the useful life of the machine was determined for five years, figure out the depreciation expense. Solution: Depreciation Expense = (Remaining Life/Sum of Years) × (Beginning Book Value − Residual Value). Year 1: Depreciation Expense = [5/(5 + 4 + 3 + 2 + 1)] × (370,000 − 40,000). Depreciation Expense = Rs. 110,000.

5.8 Worked Out Examples

165

Year 2: Depreciation Expense = [4/(4 + 3 + 2 + 1)] × (370,000 − 110,000 − 40,000). Depreciation Expense = (2/5) × (260,000 − 40,000). Depreciation Expense = Rs. 88,000. Year 3: Depreciation Expense = [3/(3 + 2 + 1)] × (260,000 − 88,000 − 40,000). Depreciation Expense = (1/2) × (172,000 − 40,000). Depreciation Expense = Rs. 66,000. Year 4: Depreciation Expense = [2/(2 + 1)] × (172,000 − 66,000 − 40,000). Depreciation Expense = (2/3) × (106,000 − 40,000). Depreciation Expense = Rs. 44,000. Year 5: Depreciation Expense = [1/1)] × (106,000 − 44,000 − 40,000). Depreciation Expense = Rs. 22,000. The organization can opt for one or more methods of depreciation, according to its accounting policy, business type, strategic approach and asset type. Example 5.6 (On alternative investment in terms of profitability) A company has to decide among three nearly similar alternative investment proposals. The details are given as follows: Parameter

Investment-I

Investment-II Investment-III

Total initial fixed capital investment (Rs.)

1,00,000

1,70,000

2,10,000

Working capital investment (Rs.)

10,000

10,000

15,000

Salvage value at the end of service life (Rs.) 10,000

15,000

20,000

Service life (Years)

5

7

8

Annual earnings after tax (Rs.)

1st year: 30,000 42,000 2nd year: 31,000 3rd year: 36,000 4th year: 40,000 5th year: 43,000

48,000

A minimum annual return of 15% on the original investment after taxes is acceptable. The income tax rate is 25% on net annual income. Straight-line depreciation is used over the service life. Continuous cash flow and continuous compounding are assumed. Costs of land and pre-start-up are ignored. Recommend the most suitable investment by analysing all the three investments using the following: (a) (b) (c)

Rate of Return on Investment (ROI). Payback period. Net return.

166

5 Financial Aspects of Project Engineering

Solution (a)

Rate of return on investment

Investment-I Annual depreciation: (Fixed capital − salvage value)/service life. = (1,00,000 − 10,000)/5 = Rs. 18,000. Considering 25% tax, net depreciation is Rs. 13,500. Hence, net income for 1st year = total earning after tax- depreciation after tax. = Rs.30,000 − Rs. 13,500 = Rs. 16,500. 2nd year = Rs. 31,000 − Rs. 13,500 = Rs. 17,500. 3rd year = Rs. 36,000 − Rs. 13,500 = Rs. 22,500. 4th year = Rs. 40,000 − Rs. 13,500 = Rs.26,500. 5th year = Rs. 43,000 − Rs. 13,500 = Rs. 29,500. Total: Rs. 1,12,500. Average return = Rs. 1,12,500/5 = Rs. 22,400 per year. ROI per tear after tax: (Average return)/(fixed capital investment + working capital investment). = Rs. 22,400/(Rs. 1,00,000 − Rs. 10,000). = 0.2036 that is 20.36%. Since the minimum return acceptable is 15% and the return from this investment after tax is more than that, it is acceptable. Investment -II Depreciation = (Rs. 1,70,000 − Rs. 15,000)/7 = Rs. 22,143. Net depreciation = Rs. 22,143 × 0.75 = Rs. 16,607. Net income = Rs. 42,000 − Rs.16,607 = Rs. 25,393 per year. ROI after tax = Rs. 25,393/(Rs.1,70,000 + Rs. 10,000). = 0.1410 that is 14.10%. Since it is below 15%, it is not acceptable. Investment-III Depreciation = (Rs. 2,10,000−Rs. 20,000)/8 = Rs. 23,750. Net depreciation = Rs. 23,750 × 0.75 = Rs. 17,813. Net income = Rs. 48,000 − Rs.17,813 = Rs. 30,187 per year.

5.8 Worked Out Examples

167

ROI after tax = Rs. 30,187/(Rs.2,10,000 + Rs. 15,000). = 0.1341 that is 13.41%. Since it is below 15%, it is not acceptable. The only investment-I is acceptable as analysed by the ROI method. (b)

Payback period method

Investment-I Fixed capital investment = Rs. 1,00,000. Average return after tax = Rs. 22,400 per year. Depreciation (before tax) = Rs. 18,000 per year. Payback period = Rs. 1,00,000/(Rs. 22,400 + Rs.18,000) = 2.47 years. Investment-II Fixed capital investment = Rs. 1,70,000. Average return after tax = Rs. 25,393 per year. Depreciation (before tax) = Rs. 23,750 per year. Payback period = Rs. 1,70,000/(Rs. 25,393 + Rs.23,750) = 3.45 years. Investment-III Fixed capital investment = Rs. 2,10,000. Average return after tax = Rs. 30,187 per year. Depreciation (before tax) = Rs. 17,813per year. Payback period = Rs. 2,10,000/(Rs. 30,187 + Rs.17,813) = 4.37 years. Since the payback period is the lowest for investment-I, it should be recommended. (c)

Net return method

Investment-I Total return in service period of 5 years = Rs. 1,12,500. Return at the desired rate of 15% = Rs.1,10,000 × 0.15 × 5 = Rs.82,500. Net return over the required value = Rs. 1,12,500 − Rs. 82, 500 = Rs. 29,500. The value is positive; hence acceptable. Investment-II Total return in service period of 7 years = Rs. 25,393 × 7 = Rs. 1,77,751. Return at the desired rate of 15% = Rs.1,80,000 × 0.15 × 7 = Rs. 1,89,000.

168

5 Financial Aspects of Project Engineering

Net return over the required value = Rs. 1,77,751 − Rs.1,89,000 = −Rs.11,249. The value is negative; hence not acceptable. Investment-III Total return in service period of 8 years = Rs. 30,187 × 8 = Rs. 2,41,496. Return at the desired rate of 15% = Rs.2,25,000 × 0.15 × 8 = Rs. 2,70,000. Net return over the required value = Rs. 2,41,496 − Rs.2,70,000 = −Rs.28,504. The value is negative; hence not acceptable. Hence, only Investment-I is acceptable by net return method. Example 5.7 (On profitability analysis) A proposed facility requires an initial investment of Rs. 9,00,000 and working capital of Rs. 1,00,000. Annual revenue is Rs. 8,00,000. Annual expense including depreciation is Rs. 5,20,000 before income tax. At least 15% profit on initial investment before tax is acceptable for investment. Prevailing rate of income tax is 25%. Determine the following: Annual percent return before tax on total investment. Annual percent return after tax on total investment. Annual percent return on total initial investment before income tax based on capital recovery with minimum profit. Solution Annual profit before income tax: Rs. 8,00,000 − Rs. 5,20,000 = Rs. 2,80,000. % return = Rs. 2,80,000/(Rs. 9,00,000 + Rs.1,00,000) = 28%. Annual profit after income tax: Tax is 25% on revenue. So net income after tax is Rs. 8,00,000 × 0.75. % return after tax = Rs. 8,00,000 × 0.75/(Rs. 9,00,000 + Rs.1,00,000) = 21%. Minimum profit required: 15% on total investment = 0.15 × Rs. 10,00,000 = Rs. 1,50,000. This should be considered as a mandatory amount like the expenses that should be arranged for any situation. So the minimum amount for which provision should always be kept is Rs. 5,20,000 + Rs.1,50,000 = Rs. 6,70,000. Profit in addition to the minimum amount is = Rs. 8,00,000 − Rs. 6,70,000. = Rs. 1,30,000. % return in this situation is = Rs.1,30,000/Rs. 10,00,000 = 13%. Ans.

5.8 Worked Out Examples

169

Example 5.8 (On interest calculation) Rs. 1,00,000 is borrowed at a monthly interest of 2%. Determine the following: (a) (b) (c)

Total matured amount with simple interest after 2 years if no intermediate payment is made. Total matured amount with compound interest compounded monthly after 2 years if no intermediate payment is made. The effective rate of interest, if compounded monthly.

Solution: (All in Rs.) Monthly interest is 2%. For 2 years, that is 24 months, the rate of interest is 0.02 × 24. Again we know F = P(1 + i × n) = 1,00,000 (1 + 0.02 × 24) = Rs1,48,000. Ans. (a) mn When the interest is compounded monthly, F = P(1 + ni ) . Here, m = number of times of compounding interest = 24. n = period of compounding interest = 1 month. F = 1,00,000 [1 + 0.02]24 = Rs. 1,60,800. Ans. (b) Since yearly interest rate is 2 × 12 = 24%, (12 months in a year and monthly rate is 2%). n Effective yearly simple interest rate = i e f f = (1 + ni ) − 1 (1 +

0.24 12 ) 12

− 1 = 26.8%. Ans. (c)

Example 5.9 (On interest calculation) Assuming nominal interest rate to be 20%, determine the worth of 1 rupee (Re) in the following situations: (a) (b) (c)

After 1 year with continuous compounding. After 1 year with daily compounding. The effective annual interest rate for (a).

Solution (all in rupees) (a) (b) (c)

F = Pein = 1 × e0.2×1 = 1.2214. Ans (a) 0.2 365 ) = 1.2213. Ans (b) F = 1 × (1 + 365 r i e f f = e − 1 = 1.2214–1 = 0.2214 = 22.14%. Ans (c)

Example 5.10 (On present worth and discount) A bond has a maturity value of Rs. 1,00,000. The effective annual rate of discrete compound interest is 3%. Determine the following at four years before maturity: (a) (b) (c)

Present worth. Discount. Present worth with continuous compounding at 3%.

Solution: (all in Rs.) (a)

P=

F (1+i)n

=

1,00,000 (1+0.03)4

= 88, 800. Ans (a)

170

(b) (c)

5 Financial Aspects of Project Engineering

Discount = 1,00,000 – 88,800 = 11,200. Ans (b) = 86, 900. Ans (c) P = eFin = 1,00,000 e0.03×4

Example 5.11 (On alternative investment) Management of an existing plant proposes to install heat exchangers to recover heat energy lost through waste gases. Four different designs were available, details of which are given in the Table. Management has decided that at least a 15% annual return before taxes based on the initial investment will be necessary for investment. Which design among the four should be recommended? The table containing data to solve the problem Parameter

Design No. 1

No. 2

No. 3

No. 4

Initial installed cost (Rs)

700,000

1,120,000

1,400,000

1,820,000

Operating cost (Rs/year)

7000

7000

7000

7000

Fixed charges (% of initial cost/year)

20

20

20

20

Value of heat saved (Rs/year)

287,000

441,000

511,000

619,500

Solution Annual savings = Heat saved − Fixed charges Design No. 1: Annual savings = 287,000 − 0.2 × 700,000 = Rs. 147,000. = 0.21 21%. Percent return = 147000 700000 Design No. 2: Annual savings = Rs. 217,000. Percent return = 19.37% Design No. 3: Annual savings = Rs. 231,000. Percent return = 16.5%. Design No. 4: Annual savings = Rs. 255,500. Percent return = 14.03%. The last design is not at all acceptable since the management will consider only those designs that will give at least a 15% return. Among the other three options, all give more return than the per cent required by the management.

5.8 Worked Out Examples

171

But it is to be calculated that how much more investment with reference to a particular option brings how much more return, that is return on incremental investment. Now comparing Design 1 and 2, the difference in annual savings = Rs. 70,000 while the difference in initial investment is Rs. 420,000. They will get Rs. 70,000 more savings by investing Rs. 420,000 more. So per cent return of incremental investment = 16.66%. Hence, Design 2 is preferred compared to Design 1. Then let us compare Design 2 and Design 3. The difference in annual savings = Rs. 14,000 while the difference in initial investment is Rs. 280,000. They will get Rs. 14,000 more savings by investing Rs. 280,000 more. So percent return of incremental investment = 5%. Hence, Design 2 is the obvious choice. Now the same problem is solved using minimum return as a cost method. Here, we have to determine which one of the four designs will give the highest annual savings above the required annual return of 15%. Design 1: The net savings above the required return is 147,000–0.15 × 700,000 = Rs. 42,000. Design 2: 217,000–0.15 × 1,120,000 = Rs. 49,000. Design 3: 231,000–0.15 × 1,400,000 = Rs. 21,000. Design 4: 255,500–0.15 × 1,820,000 = −Rs.17500. Here also, Design 2 is recommended. Ans Example 5.12 (On Alternative investment) A plant produces 10,000 metric tons of a product. The overall yield of the manufacturing process is 70% on a mass basis. The cost of key raw material is Rs. 500/metric ton and the sale price of the product is Rs. 900/metric ton. In an alternative process, the overall yield is 75%, but adaption to that process requires an additional investment of Rs. 12, 50,000. Operating cost remains more or less the same. Decide which process should be followed in the plant. Solution If the yield increases, the requirement (also cost) of raw material will decrease, decreasing the production cost. With 70% yield, the key raw material required was 10,000 = 14,286 metric ton. 0.70 Cost of raw material:Rs.71,43,000. With 75% yield, the key raw material required will be 10,000 = 13,333 metric ton. 0.75 Cost of raw material:Rs.66,66,500. Savings in the cost of production = Rs. 4,76,500 per year. ROI- pre-tax will be = savings/investment = 38% and = 2.62 years. Payback period = Rs.12,50,000 Rs.4,76,500

172

5 Financial Aspects of Project Engineering

Hence, the second process with a higher yield should be adapted. Ans Example 5.13 (On replacement) A company is using a piece of equipment that had an original cost of Rs. 21,00,000. The equipment has been in use for a few years and presently has a net realizable value of Rs. 4,20,000. At the time of installation, the service life was estimated to be 10 years with 0 salvage value after the end of service life. The operating cost is Rs. 15,40,000 per year. A proposal of replacing the existing equipment with a new one has been made. The new equipment costs Rs. 28,00,000 and its operating cost is Rs. 10,50,000 per year. It also has the same estimated service life of 10 years with zero salvage value at the end of service. Depreciation costs should be calculated by the straight-line method. The company will not invest unless a return of at least 10% on the investment is received. Now the question is, should the company go for the replacement or continue with the current equipment till its service life? Solution Now let us compare the annual variable expenses of the proposed equipment with the existing one. Only the operating cost and the depreciation cost are variable. For the proposed equipment, annual operating cost + annual depreciation cost = Rs. 10,50,000 + Rs. 28,00,000/10 = Rs. 13,30,000. For the existing equipment, the same is Rs. 15,40,000 + Rs.21,00,000/10 = Rs. 17,50,000. So annual savings by replacement is = Rs. 4,20,000. These savings can be made by investing Rs. 28,00,000 (cost of the new equipment) − Rs. 4,20,000 (net realizable value of the old equipment) = Rs.23,80,000. This gives a return of 17.64% on the investment which is more than the desired value of 10%. Hence, the replacement is recommended. Example 5.14 (On replacement) A carbon steel heat exchanger that costs Rs. 14,00,000 is expected to have a service life of 5 years before it requires replacement. If SS-304 is used as MOC of the exchanger, the service life is increased to 10 years. The material cost factor of SS304 with respect to carbon steel is 1.3. The cost of capital is 12%. Determine whether the replacement is recommended. Solution: With carbon steel exchanger,

5.8 Worked Out Examples

173

A i(1 + i)n

= P (1 + i)n − 1 =

0.12 × (1 + 0.12)5 (1 + 0.12)5 − 1

= 0.277.

Annualized capital cost = Rs. 14,00,000 × 0.277 = Rs. 3,87,800 per year. With SS-304 exchanger at 10 years’ service life, A 0.12 × (1 + 0.12)10 = = 0.177. P (1 + 0.12)10 − 1 In this case, with material cost factor of 1.3, capital investment is Rs. 14,00,000 × 1.3 = Rs. 18,20,000. With A/P = 0.177, annualized capital cost is Rs. 18,20,000 × 0.177. = Rs. 3,22,100 per year which is less than the same with carbon steel exchanger. Hence, SS-304 exchanger will be more economical and the replacement is recommended. Example 5.15 (On the effect of inflation on profitability—before time zero) Estimated financial data for a plant are given as follows: Fixed capital investment—Rs. 6.45 cr. Working capital investment—Rs. 1.14 cr. Annual sales revenue—Rs. 3.88 cr. The annual cost of operation without depreciation—Rs. 2.05 cr. The tentative schedule is as follows: Actual construction will start after 6 months from now. The first instalment of fixed capital expenditure will occur after 1 year from now. It is 30% of the total FCI and would inflate at a rate of 5% per annum. The second instalment, which is 70% of the FCI, will be spent after 2 years from now; this would also inflate at the rate of 5% per annum. The zero time is expected after 2 years from now when the working capital will be required. Its rate of inflation would be 4% per annum; the rate is the same for inflation of annual operating cost. Sales revenue will not inflate. All rates of inflation will compound annually. Based on the above information, (a) (b)

Estimate the inflated fixed and working capital investments at time zero. Determine the annual cost of operation at time zero.

Solution At present, fixed capital investment is Rs. 6.45 cr. 30% of it, that is, Rs. 6.45 cr × 0.3 = Rs. 1.935 cr. will be required after 1 year from now. At that point of time, its inflated value will be

174

5 Financial Aspects of Project Engineering

= 1.93 × (1 + 0.05)1 = Rs. 2.03 cr. 70% of Rs. 6.45 cr, that is, Rs. 6.45 cr × 0.7 = Rs. 4.515 cr. will be required after 2 year from now. At that point of time, its inflated value will be = 4.515 ×(1 + 0.05)2 = Rs. 4.98 cr. After inflation, the total fixed capital investment will have the value at time zero = Rs. 2.03 cr. + Rs. 4.98 cr. = Rs. 7.01 cr. At zero time, working capital will inflate to = 1.14 × (1 + 0.04)2 = Rs. 1.23 cr. Annual operating cost will inflate to = 2 .05 × (1 + 0.04)2 = Rs. 2.217 cr. Since the FCI and operating cost will increase by inflation, and there will be no inflation in sales revenue, profit will decrease. Example 5.16 (On a net present value) A Company is considering purchasing a piece of equipment to be attached to the main manufacturing machine. The equipment will cost Rs. 6,00,000 and will increase the annual cash inflow by Rs.2,20,000. The useful life of the equipment is 6 years. After 6 years, it will have no salvage value. The management wants a 20% return on all investments. (a) (b)

Compute the net present value (NPV) of this investment project. Should the equipment be purchased according to NPV analysis?

Solution Annual cash flow is Rs. 2,20,000. A i(1 + i)n

= P (1 + i)n − 1 0.2(1 + 0.2)6

= 0.3007. = (1 + 0.2)6 − 1 So, the present value of the annual cash flow will be P = A/0.3007 = 2,20,000/0.3007 = Rs. 7,31,612.20. Initial investment at present is Rs. 6,00,000.00. Net present value = Rs. 1,31,612.20. Ans (a) Since the net present value is positive, the equipment can be purchased. Ans (b) Example 5.17 (Decision based on the net present value) A Manufacturing Company is planning to reduce its labour costs by automating a critical task currently performed manually. The automation requires installation of a new machine having purchase and installation cost of Rs. 15,00,000. The installation

5.8 Worked Out Examples

175

of a machine can reduce annual labour costs by Rs. 4,20,000. The life of the machine is 15 years. The salvage value of the machine after fifteen years will be zero. The required rate of return by the management of the company is at least 25%. Should the company go for automation? Solution The annual saving of the labour cost is Rs. 4,20,000 which is the annual cash flow. Like previous example, A/P = 0.2591. Present worth of the annual savings = Rs. 4,20,000/0.2591 = Rs.16,20,890. Present cost of the installed equipment is Rs. 15,00,000. The net present worth is = Rs. 1,20,890. Since the net present value is positive, the equipment can be purchased and the company can go for this automation. Example 5.18 (Net present value with uneven cash flow) A project requires an initial investment of Rs. 22,50,000 and is expected to generate the following net cash inflows: Year

Net cash flow (Rs.)

Year 1:

9,50,000

Year 2:

8,00,000

Year 3:

6,00,000

Year 4:

5,50,000

Compute the net present value of the project if the minimum desired rate of return is 12%. Solution: Present value of single payment = Future value/(1 + i)n . With this formula, the present value of all the future cash flows are calculated as follows: Year

Net cash flow (Rs.)

Year 1

9,50,000

8,48,214

Year 2

8,00,000

6,37,755

Year 3

6,00,000

4,27,068

Year 4

5,50,000

Total present value of the cash flow:

Initial investment at present: 22, 50,000. Net present value: Rs. 12, 572. Ans

Present value of cash flow (Rs.)

3,49,535 22,62,572

176

5 Financial Aspects of Project Engineering

Example 5.19 ( On cost estimation of an equipment) The cost of a shell and tube heat exchanger (carbon steel) with a heating surface of 12m2 was Rs. 10,00,000 in 2001. The cost inflation index in 2001 was 426 whereas the same in 2016 was 1125. (a) (b)

Estimate the cost of a similar heat exchanger of heating area 28m2 in 2016. What would be the difference in estimation if the six-tenths rule was followed?

Solution From Table, value of n = 0.44. Now the cost of HE in 2016 can be estimated by Cost of HE in 2001 ×

Cost index in 2016 × Cost index in 2001



Area of new HE Area of old HE

0.44

= 10, 00, 000 × (1125/426) × (28/12)0.44 Rs.38, 34, 005. Ans (a) If 0.6 was used as the value of the exponent n, then the cost would be Rs. 43, 90, 652. Ans (b) Example 5.20 (On Capitalized cost) A reactor having installed cost of Rs. 50,000 has expected to have a useful life of 10 years. If the estimated scrap value is Rs. 10,000 and the annual compound interest rate of 5%, what will be the capitalized cost of the reactor? Solution: We know, Capitalized cost = Installed cost of reactor + amount to invest at the time of installation to replace the reactor after service life at a compound interest rate i. Capitalized cost = P0 + = 50000 +

(P0 −Ps ) (1+i)n −1

(50000−10000) (1+5)10 −1

= Rs. 1,13,603. Ans Summary Different financial aspects of a project like balance sheet, cash flow, depreciation, capital investment and profitability analysis are explained with illustrative examples. Decision criteria to select among various options of alternative investments are also explained. Guidelines for the economic decision of replacing a fixed asset are discussed.

5.8 Worked Out Examples

177

Exercise 1.

2.

3.

4.

5.

6.

7. 8.

9.

10.

11.

12. 13.

14.

What is meant by project cash flow? What are its main inflows and outflows? What is the significance of cash flow for analysing the financial health of a company? What is a capital investment? Is it an inflow or outflow in the project cash flow? What are the components of total capital investment? What is meant by direct and indirect costs? What are the major components of fixed capital investment? What is the difference between direct cost and direct field cost? Which costs are covered under offsite costs or OSBL investments? What is working capital? How cost of production is related to working capital? What are the major divisions of the cost of production and what are the costs that are covered under the divisions? Define depreciation. How it is related to the project cash flow? Is depreciation an inflow or an outflow to a project cash flow? Which one of the capital investments is included for the calculation of depreciation? Name a few methods for calculating depreciation. What is a sinking fund? How straight-line method of calculating depreciation is different from the sum of the years method? Distinguish between annuity and perpetuity. How can you define annuity depreciation? What are the various methods for estimating capital investments? Of them, which methods are comparatively more accurate and why? What are the uses of the less accurate methods? What is meant by cost index? What is its use? What are the cost components of the fixed capital investments? Name four such cost components with their respective percentage contributions to the fixed capital investment. What is the six-tenth rule in the context of estimating the cost of purchased equipment? Why this rule does not uniformly hold good for estimating the cost of all types of equipment? What is the profitability of a company or project? What are the major parameters by which profitability can be analysed? Name a few methods for profitability analysis. Indicate whether there is any difference among them with respect to the basis of analysis. Is there any influence of inflation on profitability analysis? If yes, then explain with an example how one can account for it. What is the difference between alternative investment and replacement? What are the criteria for selecting one of the alternatives from a few options for investment? The cost of a carbon steel bubble cap tray of 1.5 m diameter in a plate column was Rs. 1,00,000 in the year 2010. In 2020, the company decides to replace

178

5 Financial Aspects of Project Engineering

them with SS-304 trays. The cost inflation index in 2010 was 167 whereas it was 301 in 2020. The material cost factor is 1.3. a. b. 15.

16.

17.

18.

Estimate the cost of the SS-304 tray of 2.9 m diameter in 2020. What would be the estimated cost if the six-tenth rule was followed?

A company is considering an alternative process for production that would cost a capital investment of Rs. 20,00,000 for a new setup, but if installed, it could save Rs. 3,00,000 per annum. The life of the new setup is expected to be 20 years and salvage value would be zero after that period. The required rate of return acceptable is 20% on the capital investment. Should the company switch over to the new process? A heat exchanger costs Rs. 40,000 and has a negligible scrap value after a useful life of 6 years. Assuming an effective compound interest of 10% per year, what will be the capitalized cost of the heat exchanger? A plant with a production capacity of 10,000 tpa has an overall yield of 70% in terms of kg product/kg raw material. The cost of raw material is Rs. 50,000 per ton. An investment of Rs. 12.5 crore is required to enhance the yield to 75%. How many years are required to recover the invested amount by the additional profit? Total capital investment for a small plant is Rs. 10,00,000 whereas the working capital is Rs.1,00,000. If the turnover ratio is 1.0, then what will be the gross annual sales?

References 1. Peter, M. K., Timmerhaus, D., West, R. E. (2004). Plant design and economics for chemical engineers (5th ed.). McGraw-Hill. 2. https://www.investopedia.com. Accessed Nov 2020. 3. Sinnot, R., Towler, G. (2009). Chemical engineering design: Coulson and Richardson’s chemical engineering series. Vol 6, Elsevier (Butterworth-Heinemann). 4. https://taxguru.in/income-tax/cost-inflation-index-meaning-and-index-for-all-the-years.html. Accessed Sep 2021. 5. https://www.accountingformanagement.org. Accessed Dec 2020. 6. https://corporatefinanceinstitute.com/resources/knowledge/accounting/balance-sheet/. Accessed Sep 2021.

Chapter 6

Management Aspects of Project Engineering

Project planning, scheduling and monitoring are the three major management aspects for a chemical engineering project. Various management tools like bar chart, Gantt chart and network analysis are used for these purposes. Network analysis may also be of different types like PERT (Programme Evaluation and Review Technique) and CPM (Critical Path Method). All these are explained in this chapter with simple language and examples. Another important point of management is risk management and that is also explained. Learning objectives • To be familiar with the management skills required for the implementation of a chemical process plant. • To know how to plan, schedule or monitor a process plant. • To manage risks associated with the construction and operation of a process plant.

6.1 Preamble As mentioned in the Preface, the third pillar of Project Engineering is its management aspect. Though management skills are always required formally or informally to execute a project, application of some management techniques is required especially in project planning, scheduling and monitoring. Financial, human resources and construction site management are also part of it, though chemical engineers working as process design or project engineers should emphasize on the planning, scheduling and monitoring as management aspects of project engineering. Other managements are separate courses in their own capacities.

© The Author(s), under exclusive license to Springer Nature Singapore Pte Ltd. 2022 S. Chakrabarti, Project Engineering Primer for Chemical Engineers, https://doi.org/10.1007/978-981-19-0660-2_6

179

180

6 Management Aspects of Project Engineering

6.2 Project Planning For a project, it is rightly said that ‘Plan the work, then work the plan’. A good project plan is not only the development of activity sequences and durations but also the full evaluation of objectives and circumstances, potential problems, options, and methods available before undertaking any task. Before carrying out the planned activities, it should be ascertained that the plan would meet the project objectives. So a project should start with the preparation of a verified master plan to meet the project requirements and should preferably be formally approved by the client. A Project Plan is therefore defined as a management document that describes the essentials of a project in terms of its objectives, justification and how the objectives are to be achieved. It describes how all of the major activities under each project management function are to be accomplished along with the overall project control. The project plan undergoes continuous evolution throughout the various stages of the project life cycle. It is assumed that the pre-project work and part of basic engineering are already done before a project plan is prepared [1]. Project planning is a complex and iterative task in which the following steps are involved: 1st step: Project goals The objectives of a project are achieved only when the needs of all the stakeholders of the project are fulfilled. Hence it is essential that the stakeholders and the tasks to fulfil the needs are identified at the planning stage of the project. So the most important step is. To identify all the tasks to undertake within the scope of the project subject to the technical, business and commercial constraints. 2nd step: Project deliverables The next step is to identify or decide what are the documents and the outputs that should be generated in the project with their tentative delivery time required. Responsibilities for such deliverables should also be fixed. However more accurate delivery schedule may be obtained during the third step that is project scheduling. 3rd step: Project schedule This is the most critical step of a project plan which will be described in detail in the next section. Project scheduling is organizing the project tasks in terms of sequence and time to complete the whole project. 4th step: Support system planning Human resource, communication, cost and funds as well as risk management are among the essential support systems of a project. Estimating the effort and cost of completing each task is therefore critical for the successful completion of a project. Foreseeing the risks and getting ready for them in advance is also another important

6.2 Project Planning

181

factor for a project. These may be separately planned or can be integral parts of the project planning as a whole. Under the umbrella of a project plan, there are two phases—the preparation phase and the execution phase. Plans are required for both the phases. Resource plan, financial plan, quality assurance plan, risk management plan, communication plan and procurement plans come under the preparation phase, whereas time, cost, quality and progress management and monitoring plans are under the execution planning.

6.3 Project Scheduling Project scheduling is defined as the process of determining when project activities will take place depending upon defined durations and precedent activities. It is a critical management task by which the time of completion of various activities as well as that of the project as a whole can be fixed. The project schedule also determines the sequence of activities and, in turn, determines the requirement of various resources to complete the tasks. Schedule constraints specify when an activity should start or end, based on the given duration, predecessor, resource availability and target dates. Project scheduling is also a complex and iterative task that typically involves the following: • Assigning resources to project tasks. • Assigning date of completion considering the availability of the appropriate resources within the stipulated time. • Identifying dependencies between tasks for the correct sequence of tasks. • Identifying realistic start and endpoints to accommodate the number of man-days required for each given task. • Analysis and determination of critical path to identify which are critical tasks for successful completion of the project. Although often used interchangeably, the terms ‘project planning’ and ‘project scheduling’ are two entirely different elements of the structure of the task. The ‘project plan’ serves as the master blueprint, whereas the ‘project schedule’ enlists detail of specific tasks within the ‘project plan’. As described before, the key elements of planning are the following: • • • • • • • •

Decide objectives and scope of work. Decide life cycle phases. Estimate risk. Draw work breakdown structure (WBS). List out project activities based on WBS. Estimate time and resource requirement. Develop logical sequences between activities. Prepare the schedule of the project.

182

6 Management Aspects of Project Engineering

• Finalize budget and specification. • Approve plan, schedule and budget. Work Breakdown Structure (WBS) The logical breakdown of the project into its components and sub-components in hierarchical order is called work breakdown structure [1]. WBS is generally constructed by breaking down a project into major tasks or activities; each of such activities is further divided continuously into sub-activities or sub-tasks till a set of quantifiable and controllable activities are obtained where responsibilities can be fixed for each of them. The smaller components should fulfil the following criteria. Each of them should be. • • • •

Tangible and controllable enough to assign responsibility. Independent so that should not depend on other ongoing activities. Integrable so that the total picture can be seen. Quantifiable so that progress can be measured.

Advantages of WBS • • • • •

It is required for planning. It is required for an accurate cost estimate. Responsibility can be assigned. Risk can be estimated. Project schedule and network analysis should be based on the activities generated as per WBS. • When summed up, the overall programme can be described. Various levels of work breakdown structure are as follows (Fig. 6.1): Fig. 6.1 Levels of work breakdown structure (WBS)

6.3 Project Scheduling

183

Fig. 6.2 Example of WBS—construction of a building

An example of the project, task and sub-task is given in Fig. 6.2.

6.4 Project Scheduling and Progress Control As indicated before, project scheduling is enlisting the actual jobs or activities of a project in proper sequence along with the durations envisaged to perform the activities. Manpower and material requirements at each stage of construction are calculated along with the expected completion time of each of the jobs. A schedule shows the starting and completion dates of each activity and the sequential relationship among the various activities. Basically, the project schedule is the tool that indicates, what work needs to be performed, which resources of the organization will perform the work and the timeframes in which that work needs to be performed [1]. Once the actual production (activities) has started, it becomes essential to monitor the progress of work so that corrective measures can be taken in time. Progress control means the timely achievement of a certain level of efficiency or a certain volume of production. The system of progress control should accomplish timely, adequate and accurate information about the progress, delay and under/overloading. Steps involved in progress control: (a) (b) (c) (d)

(e)

Setting up a system to watch/record the progress of the operating facility. Making a report of work progress or work accomplishment against the original schedule or plan. Transmission of the report to various control groups and accounting groups. Interpretation of the information contained in the progress report by the control group. Interpreting the information given in the status or progress report by the controlling authority. Taking corrective action, if required.

184

6 Management Aspects of Project Engineering

Several techniques are available for project scheduling. Scheduling is the projected view of progress. It is for the forecast. Criteria of a technique to be suitable for scheduling of a particular project are generally as below: Reliability, universality, accuracy, simplicity, predictability, flexibility, analysability and cost-effectivity. Progress charts are normally employed for monitoring which compares work progress against a prescribed target and points out the failure; so progress charts demand action. Nearly the same types of charts are used for project scheduling and monitoring— the most fundamentals of them are Bar Chart and/ or Gantt Chart.

6.4.1 Bar Chart The bar chart was developed by Henry L Gantt in 1920. It is simple to understand, easy to prepare and consumes less time and effort. No special training is required for its development, implementation and interpretation. It is appropriate for small projects. There are a few disadvantages of bar chart as well. It is difficult to construct for large and complicated projects. Inter-relation between activities are difficult to show. Critical activities, critical paths and floats cannot be found out straightway; therefore control of the project is difficult. The bar chart consists of several bars. Each bar has its length proportional to its duration. Example of a bar chart: A R&D project proposal broke down its major aim into the following year-wise activities. The duration of the project was envisaged as 3 years. 1st year: Project acceptance, implementation order from the University and start-up (2 months) Appointment of JRF (3 months) Literature review (to be continued throughout) Procurement of equipment and chemicals (3 months) Preliminary experiments regarding synthesis and characterization of semiconductor photocatalyst nanoparticles (6 months) Preliminary blowing of waste thermocol nanocomposite films (6 months) 2nd year: Literature review-continued (to be continued throughout) Procurement—continued (3 months) Experiment design based on preliminary experiments (9 months) Photocatalytic degradation experiments under UV and sunlight (9 months) Optimization of process parameters for photo-degradation (9 months) Data analysis (Later)

6.4 Project Scheduling and Progress Control

185

Preparation for biodegradation test (Included in biodegradation) Communication of manuscript (1.5 years) 3rd year: Literature review- continued Procurement—continued Biodegradation experiments at different environments (9 months) Optimization of process parameters for biodegradation (9 months) Data analysis and kinetic modelling (2 years) Starting patent application. (6 months) It is pictorially represented as Fig. 6.3. Though the absolute durations have been specified for each activity, there are implicit predecessor–successor relationships to be followed for the construction of such a bar chart. For example, if chemicals and equipment are not procured, experiments cannot be done and so on. Hence Bar-charts can indirectly be used for an interconnection check of activities. A linked bar chart is a variation of a bar chart schedule. It uses arrows and lines to tie the activities and subsequent items specifying the successor and predecessor of each activity. Therefore the prerequisite of starting one particular activity is easy to understand. The linked bar chart shows the effect of the delay of a particular activity on the succeeding activities. It also indicates the extra time available with an activity. The available extra time is called float, and the activities having no available extra time are called critical activities. It may be considered as an ancestor of the network analysis.

Fig. 6.3 Example of Bar chart of an R&D project

186

6 Management Aspects of Project Engineering

6.4.2 Gantt Chart Gantt milestone chart is an improved version of Bar chart. A milestone implies some specific stage or point where a major activity either starts or ends or cost data becomes critical. A milestone chart shows the relationship between milestones within the same activity; thus it has better control. The Gantt chart is a modified bar chart. It was used to keep track of multiple machine schedules. Here the load is marked against a time scale with a horizontal bar or line allocated to each machine. A Gantt chart indicates the following: • • • • •

Plan for future. Progress on the present operation. Past achievements till date. Relationship among several variables. Marking situations of threatening delays.

A cursor attached to the Gantt chart can be moved across the chart to know work progress till any particular day. For production control, two basic types of Gantt chart are used—Order control chart and Machine load chart [2]. Example of a Gantt Chart : Let us consider three orders A-1, B-1 and C-1. Three machines (M-1, M-2 and M-3) are required in different orders to complete the jobs. • Order A-1 needs M-1 for the whole of January, M-3 for February and M-2 for half of the month of March. • Order B-1 needs only M-2 from the third week of January till the second week of February. • Order C-1 needs M-3 for the first two weeks in January, M-1 from the last week of January to the second week of February and M-3 again from the last week of February to the second week of March (Fig. 6.4).

Fig. 6.4 Example of order control by Gantt chart

6.4 Project Scheduling and Progress Control

187

Fig. 6.5 Example of machine control by Gantt chart

If someone wants to monitor the status of the order at the end of January, s/he would put the cursor (a vertical line) there and can realize that order A-1 has completed its job with M-1 and started work with M-3. Order B-1 is half complete with M-2. Order C-1 has completed its first spell with M-3 and continuing with M-1. However, in terms of machine-load, the situation is as below (Fig. 6.5):

6.4.3 Network Analysis Projects are broken down into individual tasks or activities which are arranged in a logical sequence. In network analysis, such activities are analysed mainly concerning time to plan the project as a whole. It is also decided as to which tasks will be performed simultaneously and which others sequentially. A network diagram is constructed which presents visually the relationship between all the activities involved. The activities require various resources like time, manpower, money or other resources which are allocated to them. The most commonly used network techniques are PERT (Programme Evaluation and Review Technique) and CPM (Critical path method). Terms related to network planning methods Event: An event is the specific instant of time that marks the start and the end of an activity. Even consumes no time or resource. It is nothing but a node in the network. It is represented by a circle, and the event number is written in the circle. Activity: Projects consist of a set of tasks which are called activities. It consumes time and resources. An activity is shown by an arrow; it begins with and ends with an event.

188

6 Management Aspects of Project Engineering

Here, the circles with numbers denote events whereas the arrow with alphabet denotes activity. Denotes the tail event of activity A whereas denotes head event of activity A. Activities are classified as below: Critical activity—Critical are those activities which, if consumed more than the estimated times, the project will be delayed. If the earliest start time + duration = the latest finishing time, then the activity can be called critical. Non-critical activity—Such activities have provisions (float or slack) so that, even if they consume up to a specified time in addition to their estimated times, the project will not be delayed. Dummy activities—When two activities start at the same instant of time, the head events are joined by a dotted arrow and this is known as a dummy activity. A dummy activity does not consume time; it can be critical or non-critical. It becomes critical if its earliest start time is the same as its latest finishing time.

Critical Path: Project duration is dictated by the sequence of activities and the sum of their durations. The critical path is formed by critical activities. A critical path consumes maximum resources. Among the various paths from the start to the end of the project, the critical path is the longest one that consumes the maximum time. A critical path has zero floats. A dummy activity joining two critical activities is also critical. Before the determination of the critical path, the earliest start time and earliest finish times of all the activities should be determined. For this, forward pass computations should be made starting from the first event. The earliest start time (EST) for the first activity is 0. Adding its duration with the start time, the earliest finish time (EFT) is obtained. If more than one activities are incoming to the event, the maximum of the start times should be considered to compute the earliest finish time. Duration: It is the estimated or actual time required to complete a particular activity. Total project time: It is the time that will be required to complete a project. It is the sum of the duration of the activities on a critical path. Earliest start time (EST): It is the earliest possible time at which an activity can start and is calculated by moving from the first to the last event in a network diagram.

6.4 Project Scheduling and Progress Control

189

Earliest finish time (EFT): It is the earliest possible time at which an activity can finish. EFT = EST + duration of that activity. Latest finish time (LFT): It is calculated by moving backwards that is from the last event to the first event of the network diagram. The latest finish time (LFT) is calculated by backward computation from the maximum time required obtained from the forward pass computation. It should start from the last event. From the LFT of the end event, the duration of the activity should be subtracted to get the latest start time (LST) of the activity. If more than one activities are incoming to that particular event, a minimum of all the finishing times should be considered to calculate the latest start time from it. Latest start time (LST): It is the latest possible time by which an activity can start. LST = LFT- duration of that activity. Float or slack: It means spare time; a margin of extra time over and above its duration which a non-critical activity can consume without delaying the project. When there is a gap between the earliest start time and the latest start time, or the earliest finish time and the latest finish time, a certain extra time is available for the work. The time in hand to complete an activity may become more than the time required to finish the particular activity. The difference between the two times is called float. Slack refers to an event, and float refers to an activity. ‘Slack’ is used in PERT whereas ‘Float’ is used in CPM [2, 3].

6.4.3.1

Critical Path Method (CPM)

Example 6.1: On critical path method Draw the network diagram and calculate EST, LST, EFT, LET and the floats. Mark the critical path and find the total project duration (Fig. 6.6).

Fig. 6.6 Network for Example 6.1

190

6 Management Aspects of Project Engineering

Start node

End node

Activity

Duration (days)

1

2

A

4

2

3

B

6

3

5

C

5

2

4

D

4

4

5

E

3

5

6

F

3

Activity

Days

EST

LST

EFT

LFT

Total float

Free float

A

4

0

0

4

B

6

4

4

10

C

5

10

10

15

D

4

4

4

E

3

8

12

F

3

15

15

Independent float

4

0

0

0

10

0

0

0

15

0

0

0

8

12

4

0

0

11

15

4

4

0

18

18

0

0

0

Total float = LST-EST or LFT-EFT. Free float = EST of tail event-EST of the head event—duration. Independent float = EST of tail event-LFT of the head event—duration. Critical path = path of longest duration = 1–2-3–5-6 = 4 + 6 + 5 + 3 = 18 days Example 6.2: On comparison of Gantt chart and Critical path method The following table indicates the activities, their predecessors and durations for a project. Determine the Critical path, Duration of the project and respective floats using both Gantt chart and Critical Path Method. Activity

Predecessor

Duration (months)

A

-

6

B

-

10

C

-

11

D

-

9

E

B

5

F

E

8

G

C

12

H

A,F

8

I

E

7

J

D,G

4

6.4 Project Scheduling and Progress Control

191

Solution: A Gantt chart is constructed as in Fig. 6.7: It is observed from the above figure that the critical path is B-E–F-H resulting in the duration of the project as 10 + 5 + 8 + 8 = 31 months. Any delay of any activity in this pathway will cause a delay in the project. Other activities may be delayed according to the floats allowable. For example, activity A is the predecessor of H along with activity F. Unless both A and F is complete H cannot start. So A can end simultaneously with F without hampering the critical path. The latest possible position of the activity is shown with and an alternative name. For alternative of A, A’ is used. It may be understood that with the change in position, critical path and project duration are not affected. The same problem is solved by the Critical Path method as in Fig. 6.8: Activity

Months

EST

LST

EFT

LFT

Total float

A

6

0

B

10

0

Free float

Independent float

17

6

23

17

17

17

0

10

10

0

0

0 4

C

11

0

3

11

15

4

0

D

9

0

18

9

27

18

14

18

E

5

10

10

15

15

0

0

0 0

F

8

15

15

23

23

0

0

G

12

11

15

23

27

4

0

4

H

8

23

23

31

31

0

0

0

I

7

15

24

22

31

9

9

9

J

4

23

27

27

31

4

4

4

Fig. 6.7 Gantt chart for Example 6.2

192

6 Management Aspects of Project Engineering

Fig. 6.8 Critical path network for Example 6.2

Total float = LST-EST or LFT-EFT. Free float = EST of the tail (starting) event-EST of (end) head event – duration. Independent float = EST of tail (starting) event-LFT of head(end) event- duration. Critical path = path of longest duration = 1–2-4–3-7 = 10 + 5 + 8 + 8 = 31 months.

6.4.3.2

Programme Evaluation and Review Technique (PERT) Network

In the case of the Critical path method, it was observed that the durations of the activities are exactly known, that is they are deterministic. But in actual practice, in many projects, the exact duration of activities cannot be ascertained. The durations are estimated as optimistic time t O (if nothing runs wrong), pessimistic time t p (if nothing runs right) and most likely time t m (generally observed duration). So the duration is probabilistic. The data generally follow a beta distribution. Mean (μ) and variance (σ) for the beta distribution are as below: μ=

(t O + 4tm + t P ) 6

and  σ2 =

t p − tO 6

2 .

6.4 Project Scheduling and Progress Control

193

 So, the expected completion time of the project is μ and the variance of the i  2 σi . It is often necessary to know the probability of project completion time is i

completion of the project before a given date (C). It is also sometimes necessary to estimate the project completion time if the probability is given. For that beta distribution has to be converted into the normal distribution whereby, z = x−μ . σ x is the actual time of completion of the project. P(x < C) represents the probability of completion of the project within C. In terms of the normal distribution [3]: 

 P{

x −μ σ